Module 6

Réussis tes devoirs et examens dès maintenant avec Quizwiz!

A patient has been diagnosed with hypoparathyroidism. What manifestations should the nurse expect to observe (select all that apply)? a. Skeletal pain b. Dry, scaly skin c. Personality changes d. Abdominal cramping e. Cardiac dysrhythmias f. Muscle spasms and stiffness

28. b, c, d, e, f. In hypoparathyroidism the patient has inadequate circulating parathyroid hormone (PTH) that leads to hypocalcemia from the inability to maintain serum calcium levels. With hypocalcemia there is muscle stiffness and spasms, which can lead to cardiac dysrhythmias and abdominal cramps. There can also be personality and visual changes and dry, scaly skin.

When teaching a patient with chronic SIADH about long-term management of the disorder, the nurse determines that additional instruction is needed when the patient says, a. "I need to shop for foods that are low in sodium and avoid adding salt to foods." b. "I should weigh myself daily and report any sudden weight loss or gain." c. "I need to limit my fluid intake to no more than 1 quart of liquids a day." d. "I will eat foods high in potassium because the diuretics cause potassium loss."

A Rationale: Pts with SIADH are at risk for hyponatremia, and a sodium supplement may be prescribed. The other pt statements are correct and indicate successful teaching has occurred.

The nurse is caring for a 68-year-old woman after a parathyroidectomy related to hyperparathyroidism. The nurse should administer IV calcium gluconate if the patient exhibits which clinical manifestations? A. Facial muscle spasms or laryngospasms B. Decreased muscle tone or muscle weakness C. Tingling in the hands and around the mouth D. Shortened QT interval on the electrocardiogram

A. Facial muscle spasms or laryngospasms Nursing care for the patient following a parathyroidectomy includes monitoring for a sudden decrease in serum calcium levels causing tetany, a condition of neuromuscular hyperexcitability. If tetany is severe (e.g., muscular spasms or laryngospasms develop), IV calcium gluconate should be administered. Mild tetany, characterized by unpleasant tingling of the hands and around the mouth, may be present but should decrease over time without treatment. Decreased muscle tone, muscle weakness, and shortened QT interval are clinical manifestations of hyperparathyroidism.

When a patient is hospitalized with acute adrenal insufficiency, which assessment finding by the nurse indicates that the prescribed therapies are effective? A. Increasing serum sodium levels B. Decreasing blood glucose levels C. Decreasing serum chloride levels D. Increasing serum potassium levels

A. Increasing serum sodium levels

Which nursing action will be included in the postoperative plan of care for a patient who has had a transsphenoidal resection of a pituitary tumor? A. Monitor urine output every hour. B. Palpate extremities for dependent edema. C. Check hematocrit hourly for first 12 hours. D. Obtain continuous pulse oximetry for 24 hours.

A. Monitor urine output every hour

The nurse is caring for a 56-year-old man receiving high-dose oral corticosteroid therapy to prevent organ rejection after a kidney transplant. What is most important for the nurse to observe related to this medication? A. Signs of infection B. Low blood pressure C. Increased urine output D. Decreased blood glucose

A. Signs of infection Side effects of corticosteroid therapy include increased susceptibility to infection, edema related to sodium and water retention (decreased urine output), hypertension, and hyperglycemia.

The nurse is caring for a 40-year-old man who has begun taking levothyroxine (Synthroid) for recently diagnosed hypothyroidism. What information reported by the patient is most important for the nurse to further assess? A. Weight gain or weight loss B. Chest pain and palpitations C. Muscle weakness and fatigue D. Decreased appetite and constipation

B. Chest pain and palpitations Levothyroxine (Synthroid) is used to treat hypothyroidism. Any chest pain or heart palpitations or heart rate greater than 100 beats/minute experienced by a patient starting thyroid replacement should be reported immediately, and an electrocardiogram (ECG) and serum cardiac enzyme tests should be performed.

A patient admitted to the hospital with hypertension is diagnosed with a pheochromocytoma. The nurse will plan to monitor the patient for A. flushing. B. headache. C. bradycardia. D. hypoglycemia.

B. Headache

A college student is newly diagnosed with type 1 diabetes. She now has a headache, changes in her vision, and is anxious, but does not have her portable blood glucose monitor with her. Which action should the campus nurse advise her to take? A) Eat a piece of pizza. B) Drink some diet pop. C) Eat 15 g of simple carbohydrates. D) Take an extra dose of rapid-acting insulin.

C) Eat 15 g of simple carbohydrates. When the patient with type 1 diabetes is unsure about the meaning of the symptoms she is experiencing, she should treat herself for hypoglycemia to prevent seizures and coma from occurring. She should also be advised to check her blood glucose as soon as possible. The fat in the pizza and the diet pop would not allow the blood glucose to increase to eliminate the symptoms. The extra dose of rapid-acting insulin would further decrease her blood glucose.

The nurse is caring for a 36-year-old woman with possible hypoparathyroidism after a thyroidectomy. It is most appropriate for the nurse to assess for which clinical manifestations? A. Polyuria, polydipsia, and weight loss Incorrect B. Cardiac dysrhythmias and hypertension C. Muscle spasms and hyperactive deep tendon reflexes Correct D. Hyperpigmentation, skin ulcers, and peripheral edema

C. Muscle spasms and hyperactive deep tendon reflexes Common assessment abnormalities associated with hypoparathyroidism include tetany (muscle spasms) and increased deep tendon reflexes. Hyperpigmentation is associated with Addison's disease. Skin ulcers occur in patient with diabetes. Edema is associated with hypothyroidism. Polyuria and polydipsia occur in patients with diabetes mellitus or diabetes insipidus. Weight loss occurs in hyperthyroidism or diabetic ketoacidosis. Hypertension and cardiac dysrhythmias may be caused by hyperthyroidism, hyperparathyroidism, or pheochromocytoma.

The nurse is assisting a patient with newly diagnosed type 2 diabetes to learn dietary planning as part of the initial management of diabetes. The nurse would encourage the patient to limit intake of which foods to help reduce the percent of fat in the diet? A) Cheese B) Broccoli C) Chicken D) Oranges

A) Cheese Cheese is a product derived from animal sources and is higher in fat and calories than vegetables, fruit, and poultry. Excess fat in the diet is limited to help avoid macrovascular changes.

The nurse instructs a 28-year-old man with acromegaly resulting from an unresectable benign pituitary tumor about octreotide (Sandostatin). The nurse should intervene if the patient makes which statement? A. "I will come in to receive this medication IV every 2 to 4 weeks." B. "I will inject the medication in the subcutaneous layer of the skin." C. "The medication will decrease the growth hormone production to normal." D. "If radiation treatment is not effective, I may need to take the medication."

A. "I will come in to receive this medication IV every 2 to 4 weeks Drugs are most commonly used in patients who have had an inadequate response to or cannot be treated with surgery and/or radiation therapy. The most common drug used for acromegaly is octreotide (Sandostatin), a somatostatin analog that reduces growth hormone levels to within the normal range in many patients. Octreotide is given by subcutaneous injection three times a week. Two long-acting analogs, octreotide (Sandostatin LAR) and lanreotide SR (Somatuline Depot), are available as intramuscular (IM) injections given every 2 to 4 weeks.

A patient is to receive methylprednisolone (Solu-Medrol) 100 mg. The label on the medication states: methylprednisolone 125 mg in 2 mL. How many milliliters will the nurse administer?

ANS: 1.6 A concentration of 125 mg in 2 mL will result in 100 mg in 1.6 mL.

In which order will the nurse take these steps to prepare NPH 20 units and regular insulin 2 units using the same syringe? (Put a comma and a space between each answer choice [A, B, C, D, E]). a. Rotate NPH vial. b. Withdraw regular insulin. c. Withdraw 20 units of NPH. d. Inject 20 units of air into NPH vial. e. Inject 2 units of air into regular insulin vial

ANS: A, D, E, B, C When mixing regular insulin with NPH, it is important to avoid contact between the regular insulin and the additives in the NPH that slow the onset, peak, and duration of activity in the longer-acting insulin.

A patient with Graves' disease has exophthalmos. Which nursing action will be included in the plan of care? A. Apply eye patches to protect the cornea from irritation. B. Place cold packs on the eyes to relieve pain and swelling. C. Elevate the head of the patient's bed to reduce periorbital fluid. D. Teach the patient to blink every few seconds to lubricate the cornea.

C. Elevate the head of the pt's bed to reduce periorbital fluid.

The surgeon was unable to spare a patient's parathyroid gland during a thyroidectomy. Which assessments should the nurse prioritize when providing postoperative care for this patient? A. Assessing the patient's white blood cell levels and assessing for infection B. Monitoring the patient's hemoglobin, hematocrit, and red blood cell levels C. Monitoring the patient's serum calcium levels and assessing for signs of hypocalcemia D. Monitoring the patient's level of consciousness and assessing for acute delirium or agitation

C. Monitoring the patient's serum calcium levels and assessing for signs of hypocalcemia Loss of the parathyroid gland is associated with hypocalcemia. Infection and anemia are not associated with loss of the parathyroid gland, whereas cognitive changes are less pronounced than the signs and symptoms of hypocalcemia.

When providing postoperative care for a patient who had a bilateral adrenalectomy, which assessment information requires the most rapid action by the nurse? A. The blood glucose is 176 mg/dL. B. The lungs have bibasilar crackles. C. The patient's BP is 88/50 mm Hg. D. The patient has 5/10 incisional pain.

C. the pt's BP is 88/50

The nurse is planning the care of a client diagnosed with Addison's disease. Which intervention should be included? 1. Administer steroid medications. 2. Place the client on fluid restriction. 3. Provide frequent stimulation. 4. Consult physical therapy for gait training.

1 . Clients diagnosed with Addison's disease have adrenal gland hypofunction.

The client diagnosed with a pituitary tumor developed syndrome of inappropriate antidiuretic hormone (SIADH). Which interventions should the nurse implement? 1. Assess for dehydration and monitor blood glucose levels. 2. Assess for nausea and vomiting and weigh daily. 3. Monitor potassium levels and encourage fluid intake. 4. Administer vasopressin IV and conduct a fluid deprivation test.

2. Early signs and symptoms are nausea and vomiting.

The client is admitted to rule out Cushing's syndrome. Which laboratory tests should the nurse anticipate being ordered? 1. Plasma drug levels of quinidine, digoxin, and hydralazine. 2. Plasma levels of ACTH and cortisol. 3. A 24-hour urine for metanephrine and catecholamine. 4. Spot urine for creatinine and white blood cells.

2. The adrenal gland secretes cortisol and the pituitary gland secretes adrenocorticotropic hormone (ACTH), a hormone used by the body to stimulate the production of cortisol.

The nurse is admitting a client to the neurological intensive care unit who is postoperative transsphenoidal hypophysectomy. Which data warrant immediate intervention? 1. The client is alert to name but is unable to tell the nurse the location. 2. The client has an output of 2,500 mL since surgery and an intake of 1,000 mL. 3. The client's vital signs are T 97.6 F, P 88, R 20, and BP 130/80. 4. The client has a 3-cm amount of dark-red drainage on the turban dressing.

2. The output is more than double the intake in a short time.

Which sign/symptom indicates to the nurse the client is experiencing hyperparathyroidism? 1. A negative Trousseau's sign. 2. A positive Chvostek's sign. 3. Nocturnal muscle cramps. 4. Tented skin turgor.

2. When a sharp tapping over the facial nerve elicits a spasm or twitching of the mouth, nose, or eyes, the client is hypocalcemic

The client diagnosed with Cushing's disease has undergone a unilateral adrenalectomy. Which discharge instructions should the nurse discuss with the client? 1. Instruct the client to take the glucocorticoid and mineralocorticoid medications as prescribed. 2. Teach the client regarding sexual functioning and androgen replacement therapy. 3. Explain the signs and symptoms of infection and when to call the health-care provider. 4. Demonstrate turn, cough, and deep-breathing exercises the client should perform every (2) hours.

3. Notifying the HCP if signs/symptoms of infection develop is an instruction given to all surgical clients on discharge.

The nurse is discussing the endocrine system with the client. Which endocrine gland secretes epinephrine and norepinephrine? 1. The pancreas. 2. The adrenal cortex. 3. The adrenal medulla. 4. The anterior pituitary gland.

3. The adrenal medulla secretes the catecholamines epinephrine and norepinephrine.

The client diagnosed with hypothyroidism is prescribed the thyroid hormone levothyroxine (Synthroid). Which assessment data indicate the medication has been effective? 1. The client has a three (3)-pound weight gain. 2. The client has a decreased pulse rate. 3. The client's temperature is WNL. 4. The client denies any diaphoresis.

3. The client with hypothyroidism frequently has a subnormal temperature, so a temperature WNL indicates the medication is effective.

The nurse is assessing a client in an outpatient clinic. Which assessment data are a risk factor for developing pheochromocytoma? 1. A history of skin cancer. 2. A history of high blood pressure. 3. A family history of adrenal tumors. 4. A family history of migraine headaches.

3. There is a high incidence of pheochromocytomas in family members with adrenal tumors,

The charge nurse of an intensive care unit is making assignments for the night shift. Which client should be assigned to the most experienced intensive care nurse? 1. The client diagnosed with respiratory failure who is on a ventilator and requires frequent sedation. 2. The client diagnosed with lung cancer and iatrogenic Cushing's disease with ABGs of pH 7.35, PaO2 88, PaCO2 44, and HCO3 22. 3. The client diagnosed with Addison's disease who is lethargic and has a BP of 80/45, P 124, and R 28. 4. The client diagnosed with hyperthyroidism who has undergone a thyroidectomy two (2) days ago and has a negative Trousseau's sign.

3. This client has a low blood pressure and tachycardia. This client may be experiencing an addisonian crisis

The nurse is preparing to administer the following medications. Which medication should the nurse question administering? 1. The thyroid hormone to the client who does not have a T3, T4 level. 2. The regular insulin to the client with a blood glucose level of 210 mg/dL. 3. The loop diuretic to the client with a potassium level of 3.3 mEq/L. 4. The cardiac glycoside to the client who has a digoxin level of 1.4 mg/dL.

3. This potassium level is below normal, which is 3.5 to 5.5 mEq/L.

The nurse is providing an in-service on thyroid disorders. One of the attendees asks the nurse, "Why don't the people in the United States get goiters as often?" Which statement by the nurse is the best response? 1. "It is because of the screening techniques used in the United States." 2. "It is a genetic predisposition rare in North Americans." 3. "The medications available in the United States decrease goiters." 4. "Iodized salt helps prevent the development of goiters in the United States."

4. Almost all of the iodine entering the body is retained in the thyroid gland. A deficiency in iodine will cause the thyroid gland to work hard and enlarge, which is called a goiter. Goiters are commonly seen in geographical regions having an iodine deficiency. Most table salt in the United States has iodine added.

The client is three (3) days postoperative unilateral adrenalectomy. Which discharge instructions should the nurse teach? 1. Discuss the need for lifelong steroid replacement. 2. Instruct the client on administration of vasopressin. 3. Teach the client to care for the suprapubic Foley catheter. 4. Tell the client to notify the HCP if the incision is inflamed.

4. Any inflammation of the incision indicates an infection

The nurse is admitting a client to rule out aldosteronism. Which assessment data support the client's diagnosis? 1. Temperature. 2. Pulse. 3. Respirations. 4. Blood pressure.

4. Blood pressure is affected by aldosteronism, with hypertension being the most prominent and universal sign of aldosteronism.

Which statement made by the client makes the nurse suspect the client is experiencing hyperthyroidism? 1. "I just don't seem to have any appetite anymore." 2. "I have a bowel movement about every 3 to 4 days." 3. "My skin is really becoming dry and coarse." 4. "I have noticed all my collars are getting tighter."

4. The thyroid gland (in the neck) en larges as a result of the increased need for thyroid hormone production; an enlarged gland is called a goiter.

Following a transsphenoidal resection of a pituitary tumor, an important N assessment is a. monitoring hourly urine output. b. checking the dressings for serous drainage. c. palpating for dependent pitting edema. d. obtaining continuous pulse oximetry.

A R: After pituitary surgery, the pt is at risk for diabetes insipidus caused by cerebral edema and monitoring of urine output and urine specific gravity is essential. There will be no dressing when transsphenoidal approach is used. The pt is at risk for dehydration, not volume overload. The pt is not at high risk for problems with oxygenation, and continuous pulse oximetry is not needed.

RN observes a nursing assistant (NA) caring for a patient after a hypophysectomy. Which action by the NA requires that the RN intervene? a. The NA lowers the HOB to the flat position. b. The NA cautions the patient to avoid coughing. c. The NA cleans the patient's mouth with a swab. d. NA collects a urine specimen for specific gravity

A R: HOB should be elevated about 30 degrees to decrease pressure on the sella turcica and avoid headaches. The other actions by the NA are appropriate after this surgery. (Cognitive Level: Application Text : p. 1293 NProcess: Implementation NCLEX: Safe and Effective Care Enviro)

A patient with symptoms of DI is admitted to the hospital for evaluation and treatment of the condition. An appropriate nursing diagnosis for the patient is a. insomnia related to waking at night to void. b. risk for impaired skin integrity related to generalized edema. c. excess fluid volume related to intake greater than output. d. activity intolerance related to muscle cramps and weakness.

A R: Nocturia occurs as a result of the polyuria caused by diabetes insipidus. Edema will not be expected because dehydration is a concern with polyuria. The pt drinks large amnts of fluid to compensate for losses experienced from diuresis. The pt's fluid and electrolyte status remain normal as long as the patient's oral intake can keep up w fluid losses, muscle cramps and weakness arent concerns.

After neck surgery, a patient develops hypoparathyroidism. The nurse should plan to teach the patient about a. calcium supplementation to normalize serum calcium levels. b. including whole grains in the diet to prevent constipation. c. use of bisphosphonates to reduce bone demineralization. d. having a high fluid intake to decrease risk for nephrolithiasis.

A R: Oral calcium supplements are used to maintain the serum calcium in normal range and prevent the complications of hypocalcemia. Whole-grain foods decrease calcium absorption and will not be recommended. Bisphosphonates will lower serum calcium level further by preventing calcium from being reabsorbed from bone. Kidney stones are not a complication of hypoparathyroidism and low calcium levels.

A patient with hyperthyroidism is treated with radioactive iodine (RAI) at a clinic. Before the patient is discharged, the nurse instructs the pt a. to monitor for symptoms of hypothyroidism, such as easy bruising and cold intolerance. b. to discontinue the antithyroid medications taken before the radioactive therapy. c. that symptoms of hyperthyroidism should be relieved in about a week. d. about radioactive precautions to take with urine, stool, and other body secretions.

A R: There is a high incidence of post-radiation hypothyroidism after RAI, and the pt should be monitored for symptoms of hypothyroidism. RAI has a delayed response, with maximum effect not seen for 2-3 months, and pt will continue to take antithyroid medications during this time. The therapeutic dose of radioactive iodine is low enough that no radiation safety precautions are needed.

A patient is hospitalized with possible SIADH. The patient is confused and reports a headache, muscle cramps, and twitching. The nurse would expect the initial laboratory results to include a a. serum sodium of 125 mEq/L (125 mmol/L). b. hematocrit of 52%. c. blood urea nitrogen (BUN) of 22 mg/dl (11.5 mmol/L). d. serum chloride of 110 mEq/L (110 mmol/L).

A R: When water is retained, the serum sodium level will drop below normal, causing the CMs reported by the patient. The hematocrit will decrease because of the dilution caused by water retention. The BUN is not helpful in diagnosis of SIADH and this BUN value is increased. The serum chloride level will usually decrease along with the sodium level. This chloride value is elevated.

A 63-year-old patient is newly diagnosed with type 2 diabetes. When developing an education plan, the nurse's first action should be to a. assess the patient's perception of what it means to have type 2 diabetes. b. demonstrate how to check glucose using capillary blood glucose monitoring. c. ask the patient's family to participate in the diabetes education program. d. discuss the need for the patient to actively participate in diabetes management.

A Rationale: Before planning education, the nurse should assess the patient's interest in and ability to self-manage the diabetes. After assessing the patient, the other nursing actions may be appropriate, but planning needs to be individualized to each patient. Cognitive Level: Application Text Reference: p.1264 Nursing Process: Planning NCLEX: Health Promotion and Maintenance

A patient with type 2 diabetes that is controlled with diet and metformin (Glucophage) also has severe rheumatoid arthritis (RA). During an acute exacerbation of the patient's arthritis, the health care provider prescribes prednisone (Deltasone) to control inflammation. The nurse will anticipate that the patient may a. require administration of insulin while taking prednisone. b. develop acute hypoglycemia during the RA exacerbation. c. have rashes caused by metformin-prednisone interactions. d. need a diet higher in calories while receiving prednisone.

A Rationale: Glucose levels increase when patients are taking corticosteroids, and insulin may be required to control blood glucose. Hypoglycemia is not a complication of RA exacerbation or prednisone use. Rashes are not an adverse effect caused by taking metformin and prednisone simultaneously. The patient is likely to have an increased appetite when taking prednisone, but it will be important to avoid weight gain for the patient with RA. Cognitive Level: Application Text Reference: pp. 1258, 1267 Nursing Process: Planning NCLEX: Physiological Integrity

After the home health nurse has taught a patient and family about how to use glargine and regular insulin safely, which action by the patient indicates that the teaching has been successful? a. The patient disposes of the open insulin vials after 4 weeks. b. The patient draws up the regular insulin in the syringe and then draws up the glargine. c. The patient stores extra vials of both types of insulin in the freezer until needed. d. The patient's family prefills the syringes weekly and stores them in the refrigerator.

A Rationale: Insulin can be stored at room temperature for 4 weeks. Glargine should not be mixed with other insulins or prefilled and stored. Freezing alters the insulin molecule and should not be done. Cognitive Level: Application Text Reference: p. 1261 Nursing Process: Evaluation NCLEX: Physiological Integrity

A patient using a split mixed-dose insulin regimen asks the nurse about the use of intensive insulin therapy to achieve tighter glucose control. The nurse should teach the patient that a. intensive insulin therapy requires three or more injections a day in addition to an injection of a basal long-acting insulin. b. intensive insulin therapy is indicated only for newly diagnosed type 1 diabetics who have never experienced ketoacidosis. c. studies have shown that intensive insulin therapy is most effective in preventing the macrovascular complications characteristic of type 2 diabetes. d. an insulin pump provides the best glucose control and requires about the same amount of attention as intensive insulin therapy.

A Rationale: Patients using intensive insulin therapy must check their glucose level four to six times daily and administer insulin accordingly. A previous episode of ketoacidosis is not a contraindication for intensive insulin therapy. Intensive insulin therapy is not confined to type 2 diabetics and would prevent microvascular changes as well as macrovascular changes. Intensive insulin therapy and an insulin pump are comparable in glucose control. Cognitive Level: Application Text Reference: p. 1263 Nursing Process: Implementation NCLEX: Physiological Integrity

Intramuscular glucagon is administered to an unresponsive patient for treatment of hypoglycemia. Which action should the nurse take after the patient regains consciousness? a. Give the patient a snack of cheese and crackers. b. Have the patient drink a glass of orange juice or nonfat milk. c. Administer a continuous infusion of 5% dextrose for 24 hours. d. Assess the patient for symptoms of hyperglycemia.

A Rationale: Rebound hypoglycemia can occur after glucagon administration, but having a meal containing complex carbohydrates plus protein and fat will help prevent hypoglycemia. Orange juice and nonfat milk will elevate blood sugar rapidly, but the cheese and crackers will stabilize blood sugar. Administration of glucose intravenously might be used in patients who were unable to take in nutrition orally. The patient should be assessed for symptoms of hypoglycemia after glucagon administration. Cognitive Level: Application Text Reference: p. 1282 Nursing Process: Implementation NCLEX: Physiological Integrity

A patient is hospitalized with possible syndrome of inappropriate antidiuretic hormone (SIADH). The patient is confused and reports a headache, muscle cramps, and twitching. The nurse would expect the initial laboratory results to include A. An elevated hematocrit. B. A decreased serum sodium. C. An increased serum chloride. D. A low urine specific gravity

A decreased serum sodium. Feedback When water is retained, the serum sodium level will drop below normal, causing the clinical manifestations reported by the patient. The hematocrit will decrease because of the dilution caused by water retention. Urine will be more concentrated with a higher specific gravity. The serum chloride level will usually decrease along with the sodium level.

The nurse has been teaching the patient to administer a dose of 10 units of regular insulin and 28 units of NPH insulin. The statement by the patient that indicates a need for additional instruction is, a. "I need to rotate injection sites among my arms, legs, and abdomen each day." b. "I will buy the 0.5-ml syringes because the line markings will be easier to see." c. "I should draw up the regular insulin first after injecting air into the NPH bottle." d. "I do not need to aspirate the plunger to check for blood before I inject the insulin."

A Rationale: Rotating sites is no longer necessary because all insulin is now purified human insulin, and the risk for lipodystrophy is low. The other patient statements are accurate and indicate that no additional instruction is needed. Cognitive Level: Application Text Reference: p. 1262 Nursing Process: Evaluation NCLEX: Health Promotion and Maintenance

Cardiac monitoring is initiated for a patient in diabetic ketoacidosis (DKA). The nurse recognizes that this measure is important to identify a. electrocardiographic (ECG) changes and dysrhythmias related to hypokalemia. b. fluid overload resulting from aggressive fluid replacement. c. the presence of hypovolemic shock related to osmotic diuresis. d. cardiovascular collapse resulting from the effects of hyperglycemia.

A Rationale: The hypokalemia associated with metabolic acidosis can lead to potentially fatal dysrhythmias such as ventricular tachycardia and ventricular fibrillation, which would be detected with ECG monitoring. Fluid overload, hypovolemia, and cardiovascular collapse are possible complications of DKA, but cardiac monitoring would not detect theses. Cognitive Level: Application Text Reference: p. 1281 Nursing Process: Assessment NCLEX: Physiological Integrity

When teaching a patient with type 2 diabetes about taking glipizide (Glucotrol), the nurse determines that additional teaching about the medication is needed when the patient says, a. "Since I can take oral drugs rather than insulin, my diabetes is not serious and won't cause many complications." b. "If I overeat at a meal, I will still take just the usual dose of medication." c. "If I become ill, I may have to take insulin to control my blood sugar." d. "I should check with my doctor before taking any other medications because there are many that will affect glucose levels."

A Rationale: The patient should understand that type 2 diabetes places the patient at risk for many complications and that good glucose control is as important when taking oral agents as when using insulin. The other statements are accurate and indicate good understanding of the use of glipizide. Cognitive Level: Application Text Reference: p. 1275 Nursing Process: Evaluation NCLEX: Health Promotion and Maintenance

While hospitalized and recovering from an episode of diabetic ketoacidosis, the patient calls the nurse and reports feeling anxious, nervous, and sweaty. Based on the patient's report, the nurse should a. obtain a glucose reading using a finger stick. b. administer 1 mg glucagon subcutaneously. c. have the patient eat a candy bar. d. have the patient drink 4 ounces of orange juice.

A Rationale: The patient's clinical manifestations are consistent with hypoglycemia and the initial action should be to check the patient's glucose with a finger stick or order a stat blood glucose. If the glucose is low, the patient should ingest a rapid-acting carbohydrate, such as orange juice. Glucagon might be given if the patient's symptoms become worse or if the patient is unconscious. Candy bars contain fat, which would slow down the absorption of sugar and delay the response to treatment. Cognitive Level: Application Text Reference: p. 1282 Nursing Process: Implementation NCLEX: Physiological Integrity

Glyburide (Micronase, DiaBeta, Glynase) is prescribed for a patient whose type 2 diabetes has not been controlled with diet and exercise. When teaching the patient about glyburide, the nurse explains that a. glyburide stimulates insulin production and release from the pancreas. b. the patient should not take glyburide for 48 hours after receiving IV contrast media. c. glyburide should be taken even when the blood glucose level is low in the morning. d. glyburide decreases glucagon secretion.

A Rationale: The sulfonylureas stimulate the production and release of insulin from the pancreas. If the glucose level is low, the patient should contact the health care provider before taking the glyburide, since hypoglycemia can occur with this category of medication. Metformin should be held for 48 hours after administration of IV contract, but this is not necessary for glyburide. Cognitive Level: Application Text Reference: pp. 1265-1266 Nursing Process: Implementation NCLEX: Physiological Integrity

Amitriptyline (Elavil) is prescribed for a diabetic patient with peripheral neuropathy who has burning foot pain occurring mostly at night. Which information should the nurse include when teaching the patient about the new medication? a. Amitriptyline will help prevent the transmission of pain impulses to the brain. b. Amitriptyline will improve sleep and make you less aware of nighttime pain. c. Amitriptyline will decrease the depression caused by the pain. d. Amitriptyline will correct some of the blood vessel changes that cause pain.

A Rationale: Tricyclic antidepressants decrease the transmission of pain impulses to the spinal cord and brain. Tricyclics also improve sleep quality and are used for depression, but that is not the major purpose for their use in diabetic neuropathy. The blood vessel changes that contribute to neuropathy are not affected by tricyclics. Cognitive Level: Application Text Reference: p. 1285 Nursing Process: Implementation NCLEX: Physiological Integrity

When assessing the patient experiencing the onset of symptoms of type 1 diabetes, which question should the nurse ask? a. "Have you lost any weight lately?" b. "Do you crave fluids containing sugar?" c. "How long have you felt anorexic?" d. "Is your urine unusually dark-colored?"

A Rationale: Weight loss occurs because the body is no longer able to absorb glucose and starts to break down protein and fat for energy. The patient is thirsty but does not necessarily crave sugar- containing fluids. Increased appetite is a classic symptom of type 1 diabetes. With the classic symptom of polyuria, urine will be very dilute. Cognitive Level: Application Text Reference: pp. 1255, 1258 Nursing Process: Assessment NCLEX: Physiological Integrity

The client is admitted to the intensive care department diagnosed with myxedema coma. Which assessment data warrant immediate intervention by the nurse? 1. Serum blood glucose level of 74 mg/dL. 2. Pulse oximeter reading of 90%. 3. Telemetry reading showing sinus bradycardia. 4. The client is lethargic and sleeps all the time.

A pulse oximeter reading of less than 93% is significant.

As a precaution for vocal cord paralysis from damage to the recurrent laryngeal nerve during thyroidectomy surgery, what equipment should be in the room in case it is needed for this emergency situation? a. Tracheostomy tray b. Oxygen equipment c. IV calcium gluconate d. Paper and pencil for communication

A tracheostomy tray is in the room to use if vocal cord paralysis occurs from recurrent laryngeal nerve damage or for laryngeal stridor from tetany. The oxygen equipment may be useful but will not improve oxygenation with vocal cord paralysis without a tracheostomy. IV calcium salts will be used if hypocalcemia occurs from parathyroid damage. The paper and pencil for communication may be helpful, especially if a tracheostomy is performed, but will not aid in emergency oxygenation of the patient.

The nurse is evaluating a 45-year-old patient diagnosed with type 2 diabetes mellitus. Which symptom reported by the patient is considered one of the classic clinical manifestations of diabetes? A) Excessive thirst B) Gradual weight gain C) Overwhelming fatigue D) Recurrent blurred vision

A) Excessive thirst The classic symptoms of diabetes are polydipsia (excessive thirst), polyuria, (excessive urine output), and polyphagia (increased hunger). Weight gain, fatigue, and blurred vision may all occur with type 2 diabetes, but are NOT classic manifestations.

Laboratory results have been obtained for a 50-year-old patient with a 15-year history of type 2 diabetes. Which result reflects the expected pattern accompanying macrovascular disease as a complication of diabetes? A) Increased triglyceride levels B) Increased high-density lipoproteins (HDL) C) Decreased low-density lipoproteins (LDL) D) Decreased very-low-density lipoproteins (VLDL)

A) Increased triglyceride levels Macrovascular complications of diabetes include changes to large- and medium-sized blood vessels. They include cerebrovascular, cardiovascular, and peripheral vascular disease. Increased triglyceride levels are associated with these macrovascular changes. Increased HDL, decreased LDL, and decreased VLDL are positive in relation to atherosclerosis development.

A patient is admitted with diabetes mellitus, malnutrition, and cellulitis. The patient's potassium level is 5.6 mEq/L. The nurse understands that what could be contributing factors for this laboratory result (select all that apply)? A) The level may be increased as a result of dehydration that accompanies hyperglycemia. B) The patient may be excreting extra sodium and retaining potassium because of malnutrition. C) The level is consistent with renal insufficiency that can develop with renal nephropathy. D) The level may be raised as a result of metabolic ketoacidosis caused by hyperglycemia. Correct E) This level demonstrates adequate treatment of the cellulitis and effective serum glucose control.

A) The level may be increased as a result of dehydration that accompanies hyperglycemia. C) The level is consistent with renal insufficiency that can develop with renal nephropathy. E) This level demonstrates adequate treatment of the cellulitis and effective serum glucose control. The additional stress of cellulitis may lead to an increase in the patient's serum glucose levels. Dehydration may cause hemoconcentration, resulting in elevated serum readings. Kidneys may have difficulty excreting potassium if renal insufficiency exists. Finally, the nurse must consider the potential for metabolic ketoacidosis since potassium will leave the cell when hydrogen enters in an attempt to compensate for a low pH.

Which question during the assessment of a diabetic patient will help the nurse identify autonomic neuropathy? a. "Do you feel bloated after eating?" b. "Have you seen any skin changes?" c. "Do you need to increase your insulin dosage when you are stressed?" d. "Have you noticed any painful new ulcerations or sores on your feet?"

ANS: A Autonomic neuropathy can cause delayed gastric emptying, which results in a bloated feeling for the patient. The other questions are also appropriate to ask but would not help in identifying autonomic neuropathy

In order to assist an older diabetic patient to engage in moderate daily exercise, which action is most important for the nurse to take? a. Determine what type of activities the patient enjoys. b. Remind the patient that exercise will improve self-esteem. c. Teach the patient about the effects of exercise on glucose level. d. Give the patient a list of activities that are moderate in intensity.

ANS: A Because consistency with exercise is important, assessment for the types of exercise that the patient finds enjoyable is the most important action by the nurse in ensuring adherence to an exercise program. The other actions will also be implemented but are not the most important in improving compliance

Which finding by the nurse when assessing a patient with Hashimoto's thyroiditis and a goiter will require the most immediate action? a. New-onset changes in the patient's voice b. Apical pulse rate at rest 112 beats/minute c. Elevation in the patient's T3 and T4 levels d. Bruit audible bilaterally over the thyroid gland

ANS: A Changes in the patient's voice indicate that the goiter is compressing the laryngeal nerve and may lead to airway compression. The other findings will also be reported but are expected with Hashimoto's thyroiditis and do not require immediate action.

Which finding indicates to the nurse that the current therapies are effective for a patient with acute adrenal insufficiency? a. Increasing serum sodium levels b. Decreasing blood glucose levels c. Decreasing serum chloride levels d. Increasing serum potassium levels

ANS: A Clinical manifestations of Addison's disease include hyponatremia and an increase in sodium level indicates improvement. The other values indicate that treatment has not been effective.

A 56-year-old female patient has an adrenocortical adenoma, causing hyperaldosteronism. The nurse providing care should a. monitor the blood pressure every 4 hours. b. elevate the patient's legs to relieve edema. c. monitor blood glucose level every 4 hours. d. order the patient a potassium-restricted diet.

ANS: A Hypertension caused by sodium retention is a common complication of hyperaldosteronism. Hyperaldosteronism does not cause an elevation in blood glucose. The patient will be hypokalemic and require potassium supplementation before surgery. Edema does not usually occur with hyperaldosteronism.

A 27-year-old patient admitted with diabetic ketoacidosis (DKA) has a serum glucose level of 732 mg/dL and serum potassium level of 3.1 mEq/L. Which action prescribed by the health care provider should the nurse take first? a. Place the patient on a cardiac monitor. b. Administer IV potassium supplements. c. Obtain urine glucose and ketone levels. d. Start an insulin infusion at 0.1 units/kg/hr.

ANS: A Hypokalemia can lead to potentially fatal dysrhythmias such as ventricular tachycardia and ventricular fibrillation, which would be detected with electrocardiogram (ECG) monitoring. Because potassium must be infused over at least 1 hour, the nurse should initiate cardiac monitoring before infusion of potassium. Insulin should not be administered without cardiac monitoring because insulin infusion will further decrease potassium levels. Urine glucose and ketone levels are not urgently needed to manage the patient's care.

Which patient action indicates a good understanding of the nurse's teaching about the use of an insulin pump? a. The patient programs the pump for an insulin bolus after eating. b. The patient changes the location of the insertion site every week. c. The patient takes the pump off at bedtime and starts it again each morning. d. The patient plans for a diet that is less flexible when using the insulin pump.

ANS: A In addition to the basal rate of insulin infusion, the patient will adjust the pump to administer a bolus after each meal, with the dosage depending on the oral intake. The insertion site should be changed every 2 or 3 days. There is more flexibility in diet and exercise when an insulin pump is used. The pump will deliver a basal insulin rate 24 hours a day.

A female patient is scheduled for an oral glucose tolerance test. Which information from the patient's health history is most important for the nurse to communicate to the health care provider? a. The patient uses oral contraceptives. b. The patient runs several days a week. c. The patient has been pregnant three times. d. The patient has a family history of diabetes.

ANS: A Oral contraceptive use may falsely elevate oral glucose tolerance test (OGTT) values. Exercise and a family history of diabetes both can affect blood glucose but will not lead to misleading information from the OGTT. History of previous pregnancies may provide informational about gestational glucose tolerance, but will not lead to misleading information from the OGTT

A 32-year-old patient with diabetes is starting on intensive insulin therapy. Which type of insulin will the nurse discuss using for mealtime coverage? a. Lispro (Humalog) b. Glargine (Lantus) c. Detemir (Levemir) d. NPH (Humulin N)

ANS: A Rapid- or short-acting insulin is used for mealtime coverage for patients receiving intensive insulin therapy. NPH, glargine, or detemir will be used as the basal insulin.

The nurse has administered 4 oz of orange juice to an alert patient whose blood glucose was 62 mg/dL. Fifteen minutes later, the blood glucose is 67 mg/dL. Which action should the nurse take next? a. Give the patient 4 to 6 oz more orange juice. b. Administer the PRN glucagon (Glucagon) 1 mg IM. c. Have the patient eat some peanut butter with crackers. d. Notify the health care provider about the hypoglycemia.

ANS: A The "rule of 15" indicates that administration of quickly acting carbohydrates should be done 2 to 3 times for a conscious patient whose glucose remains less than 70 mg/dL before notifying the health care provider. More complex carbohydrates and fats may be used once the glucose has stabilized. Glucagon should be used if the patient's level of consciousness decreases so that oral carbohydrates can no longer be given

Which finding indicates a need to contact the health care provider before the nurse administers metformin (Glucophage)? a. The patient's blood glucose level is 174 mg/dL. b. The patient has gained 2 lb (0.9 kg) since yesterday. c. The patient is scheduled for a chest x-ray in an hour. d. The patient's blood urea nitrogen (BUN) level is 52 mg/dL.

ANS: D The BUN indicates possible renal failure, and metformin should not be used in patients with renal failure. The other findings are not contraindications to the use of metformin.

A 38-year-old patient who has type 1 diabetes plans to swim laps daily at 1:00 PM. The clinic nurse will plan to teach the patient to a. check glucose level before, during, and after swimming. b. delay eating the noon meal until after the swimming class. c. increase the morning dose of neutral protamine Hagedorn (NPH) insulin. d. time the morning insulin injection so that the peak occurs while swimming.

ANS: A The change in exercise will affect blood glucose, and the patient will need to monitor glucose carefully to determine the need for changes in diet and insulin administration. Because exercise tends to decrease blood glucose, patients are advised to eat before exercising. Increasing the morning NPH or timing the insulin to peak during exercise may lead to hypoglycemia, especially with the increased exercise.

A 54-year-old patient is admitted with diabetic ketoacidosis. Which admission order should the nurse implement first? a. Infuse 1 liter of normal saline per hour. b. Give sodium bicarbonate 50 mEq IV push. c. Administer regular insulin 10 U by IV push. d. Start a regular insulin infusion at 0.1 units/kg/hr.

ANS: A The most urgent patient problem is the hypovolemia associated with diabetic ketoacidosis (DKA), and the priority is to infuse IV fluids. The other actions can be done after the infusion of normal saline is initiated

The nurse is caring for a patient admitted with diabetes insipidus (DI). Which information is most important to report to the health care provider? a. The patient is confused and lethargic. b. The patient reports a recent head injury. c. The patient has a urine output of 400 mL/hr. d. The patient's urine specific gravity is 1.003.

ANS: A The patient's confusion and lethargy may indicate hypernatremia and should be addressed quickly. In addition, patients with DI compensate for fluid losses by drinking copious amounts of fluids, but a patient who is lethargic will be unable to drink enough fluids and will become hypovolemic. A high urine output, low urine specific gravity, and history of a recent head injury are consistent with diabetes insipidus, but they do not require immediate nursing action to avoid life-threatening complications.

A patient receives aspart (NovoLog) insulin at 8:00 AM. Which time will it be most important for the nurse to monitor for symptoms of hypoglycemia? a. 10:00 AM b. 12:00 AM c. 2:00 PM d. 4:00 PM

ANS: A The rapid-acting insulins peak in 1 to 3 hours. The patient is not at a high risk for hypoglycemia at the other listed times, although hypoglycemia may occur

A patient who was admitted with myxedema coma and diagnosed with hypothyroidism is improving and expected to be discharged in 2 days. Which teaching strategy will be best for the nurse to use? a. Provide written reminders of self-care information. b. Offer multiple options for management of therapies. c. Ensure privacy for teaching by asking visitors to leave. d. Delay teaching until patient discharge date is confirmed.

ANS: A Written instructions will be helpful to the patient because initially the hypothyroid patient may be unable to remember to take medications and other aspects of self-care. Because the treatment regimen is somewhat complex, teaching should be initiated well before discharge. Family members or friends should be included in teaching because the hypothyroid patient is likely to forget some aspects of the treatment plan. A simpler regimen will be easier to understand until the patient is euthyroid.

A 45-year-old male patient with suspected acromegaly is seen at the clinic. To assist in making the diagnosis, which question should the nurse ask? a. "Have you had a recent head injury?" b. "Do you have to wear larger shoes now?" c. "Is there a family history of acromegaly?" d. "Are you experiencing tremors or anxiety?"

ANS: B Acromegaly causes an enlargement of the hands and feet. Head injury and family history are not risk factors for acromegaly. Tremors and anxiety are not clinical manifestations of acromegaly.

The nurse is planning postoperative care for a patient who is being admitted to the surgical unit form the recovery room after transsphenoidal resection of a pituitary tumor. Which nursing action should be included? a. Palpate extremities for edema. b. Measure urine volume every hour. c. Check hematocrit every 2 hours for 8 hours. d. Monitor continuous pulse oximetry for 24 hours.

ANS: B After pituitary surgery, the patient is at risk for diabetes insipidus caused by cerebral edema. Monitoring of urine output and urine specific gravity is essential. Hemorrhage is not a common problem. There is no need to check the hematocrit hourly. The patient is at risk for dehydration, not volume overload. The patient is not at high risk for problems with oxygenation, and continuous pulse oximetry is not needed.

A 34-year-old has a new diagnosis of type 2 diabetes. The nurse will discuss the need to schedule a dilated eye exam a. every 2 years. b. as soon as possible. c. when the patient is 39 years old. d. within the first year after diagnosis.

ANS: B Because many patients have some diabetic retinopathy when they are first diagnosed with type 2 diabetes, a dilated eye exam is recommended at the time of diagnosis and annually thereafter. Patients with type 1 diabetes should have dilated eye exams starting 5 years after they are diagnosed and then annually.

After change-of-shift report, which patient should the nurse assess first? a. 19-year-old with type 1 diabetes who has a hemoglobin A1C of 12% b. 23-year-old with type 1 diabetes who has a blood glucose of 40 mg/dL c. 40-year-old who is pregnant and whose oral glucose tolerance test is 202 mg/dL d. 50-year-old who uses exenatide (Byetta) and is complaining of acute abdominal pain

ANS: B Because the brain requires glucose to function, untreated hypoglycemia can cause unconsciousness, seizures, and death. The nurse will rapidly assess and treat the patient with low blood glucose. The other patients also have symptoms that require assessments and/or interventions, but they are not at immediate risk for life-threatening complications

The nurse is preparing to teach a 43-year-old man who is newly diagnosed with type 2 diabetes about home management of the disease. Which action should the nurse take first? a. Ask the patient's family to participate in the diabetes education program. b. Assess the patient's perception of what it means to have diabetes mellitus. c. Demonstrate how to check glucose using capillary blood glucose monitoring. d. Discuss the need for the patient to actively participate in diabetes management.

ANS: B Before planning teaching, the nurse should assess the patient's interest in and ability to self-manage the diabetes. After assessing the patient, the other nursing actions may be appropriate, but planning needs to be individualized to each patient.

The nurse is planning care for a client diagnosed with Addisons disease. Which intervention should be included? a. administer steroid medications b. place the client on fluid restriction c. provide frequent stimulation d. consult physical therapy for gait training

a. administer steroid medications

The nurse is assessing a 41-year-old African American male patient diagnosed with a pituitary tumor causing panhypopituitarism. Assessment findings consistent with panhypopituitarism include a. high blood pressure. b. decreased facial hair. c. elevated blood glucose. d. tachycardia and cardiac palpitations.

ANS: B Changes in male secondary sex characteristics such as decreased facial hair, testicular atrophy, diminished spermatogenesis, loss of libido, impotence, and decreased muscle mass are associated with decreases in follicle stimulating hormone (FSH) and luteinizing hormone (LH). Fasting hypoglycemia and hypotension occur in panhypopituitarism as a result of decreases in adrenocorticotropic hormone (ACTH) and cortisol. Bradycardia is likely due to the decrease in thyroid stimulating hormone (TSH) and thyroid hormones associated with panhypopituitarism.

The nurse determines that demeclocycline (Declomycin) is effective for a patient with syndrome of inappropriate antidiuretic hormone (SIADH) based on finding that the patient's a. weight has increased. b. urinary output is increased. c. peripheral edema is decreased. d. urine specific gravity is increased.

ANS: B Demeclocycline blocks the action of antidiuretic hormone (ADH) on the renal tubules and increases urine output. An increase in weight or an increase in urine specific gravity indicates that the SIADH is not corrected. Peripheral edema does not occur with SIADH. A sudden weight gain without edema is a common clinical manifestation of this disorder.

The cardiac telemetry unit charge nurse receives status reports from other nursing units about four patients who need cardiac monitoring. Which patient should be transferred to the cardiac unit first? a. Patient with Hashimoto's thyroiditis and a heart rate of 102 b. Patient with tetany who has a new order for IV calcium chloride c. Patient with Cushing syndrome and a blood glucose of 140 mg/dL d. Patient with Addison's disease who takes hydrocortisone twice daily

ANS: B Emergency treatment of tetany requires IV administration of calcium; ECG monitoring will be required because cardiac arrest may occur if high calcium levels result from too-rapid administration. The information about the other patients indicates that they are more stable than the patient with tetany.

An unresponsive patient with type 2 diabetes is brought to the emergency department and diagnosed with hyperosmolar hyperglycemic syndrome (HHS). The nurse will anticipate the need to a. give a bolus of 50% dextrose. b. insert a large-bore IV catheter. c. initiate oxygen by nasal cannula. d. administer glargine (Lantus) insulin.

ANS: B HHS is initially treated with large volumes of IV fluids to correct hypovolemia. Regular insulin is administered, not a long-acting insulin. There is no indication that the patient requires oxygen. Dextrose solutions will increase the patient's blood glucose and would be contraindicated

Which nursing assessment of a 69-year-old patient is most important to make during initiation of thyroid replacement with levothyroxine (Synthroid)? a. Fluid balance b. Apical pulse rate c. Nutritional intake d. Orientation and alertness

ANS: B In older patients, initiation of levothyroxine therapy can increase myocardial oxygen demand and cause angina or dysrhythmias. The medication also is expected to improve mental status and fluid balance and will increase metabolic rate and nutritional needs, but these changes will not result in potentially life-threatening complications.

After receiving change-of-shift report about the following four patients, which patient should the nurse assess first? a. A 31-year-old female with Cushing syndrome and a blood glucose level of 244 mg/dL b. A 70-year-old female taking levothyroxine (Synthroid) who has an irregular pulse of 134 c. A 53-year-old male who has Addison's disease and is due for a scheduled dose of hydrocortisone (Solu-Cortef). d. A 22-year-old male admitted with syndrome of inappropriate antidiuretic hormone (SIADH) who has a serum sodium level of 130 mEq/L

ANS: B Initiation of thyroid replacement in older adults may cause angina and cardiac dysrhythmias. The patient's high pulse rate needs rapid investigation by the nurse to assess for and intervene with any cardiac problems. The other patients also require nursing assessment and/or actions but are not at risk for life-threatening complications.

Which question will the nurse in the endocrine clinic ask to help determine a patient's risk factors for goiter? a. "How much milk do you drink?" b. "What medications are you taking?" c. "Are your immunizations up to date?" d. "Have you had any recent neck injuries?"

ANS: B Medications that contain thyroid-inhibiting substances can cause goiter. Milk intake, neck injury, and immunization history are not risk factors for goiter.

A patient who had radical neck surgery to remove a malignant tumor developed hypoparathyroidism. The nurse should plan to teach the patient about a. bisphosphonates to reduce bone demineralization. b. calcium supplements to normalize serum calcium levels. c. increasing fluid intake to decrease risk for nephrolithiasis. d. including whole grains in the diet to prevent constipation.

ANS: B Oral calcium supplements are used to maintain the serum calcium in normal range and prevent the complications of hypocalcemia. Whole grain foods decrease calcium absorption and will not be recommended. Bisphosphonates will lower serum calcium levels further by preventing calcium from being reabsorbed from bone. Kidney stones are not a complication of hypoparathyroidism and low calcium levels.

The nurse is taking a health history from a 29-year-old pregnant patient at the first prenatal visit. The patient reports no personal history of diabetes but has a parent who is diabetic. Which action will the nurse plan to take first? a. Teach the patient about administering regular insulin. b. Schedule the patient for a fasting blood glucose level. c. Discuss an oral glucose tolerance test for the twenty-fourth week of pregnancy. d. Provide teaching about an increased risk for fetal problems with gestational diabetes.

ANS: B Patients at high risk for gestational diabetes should be screened for diabetes on the initial prenatal visit. An oral glucose tolerance test may also be used to check for diabetes, but it would be done before the twenty-fourth week. The other actions may also be needed (depending on whether the patient develops gestational diabetes), but they are not the first actions that the nurse should take

Which finding by the nurse when assessing a patient with a large pituitary adenoma is most important to report to the health care provider? a. Changes in visual field b. Milk leaking from breasts c. Blood glucose 150 mg/dL d. Nausea and projectile vomiting

ANS: D Nausea and projectile vomiting may indicate increased intracranial pressure, which will require rapid actions for diagnosis and treatment. Changes in the visual field, elevated blood glucose, and galactorrhea are common with pituitary adenoma, but these do not require rapid action to prevent life-threatening complications.

Which action should the nurse take after a 36-year-old patient treated with intramuscular glucagon for hypoglycemia regains consciousness? a. Assess the patient for symptoms of hyperglycemia. b. Give the patient a snack of peanut butter and crackers. c. Have the patient drink a glass of orange juice or nonfat milk. d. Administer a continuous infusion of 5% dextrose for 24 hours.

ANS: B Rebound hypoglycemia can occur after glucagon administration, but having a meal containing complex carbohydrates plus protein and fat will help prevent hypoglycemia. Orange juice and nonfat milk will elevate blood glucose rapidly, but the cheese and crackers will stabilize blood glucose. Administration of IV glucose might be used in patients who were unable to take in nutrition orally. The patient should be assessed for symptoms of hypoglycemia after glucagon administration.

Which information will the nurse include when teaching a 50-year-old male patient about somatropin (Genotropin)? a. The medication will be needed for 3 to 6 months. b. Inject the medication subcutaneously every day. c. Blood glucose levels may decrease when taking the medication. d. Stop taking the medication if swelling of the hands or feet occurs.

ANS: B Somatropin is injected subcutaneously on a daily basis, preferably in the evening. The patient will need to continue on somatropin for life. If swelling or other common adverse effects occur, the health care provider should be notified. Growth hormone will increase blood glucose levels.

The nurse will plan to monitor a patient diagnosed with a pheochromocytoma for a. flushing. b. headache. c. bradycardia. d. hypoglycemia.

ANS: B The classic clinical manifestations of pheochromocytoma are hypertension, tachycardia, severe headache, diaphoresis, and abdominal or chest pain. Elevated blood glucose may also occur because of sympathetic nervous system stimulation. Bradycardia and flushing would not be expected.

A 37-year-old patient has just arrived in the postanesthesia recovery unit (PACU) after a thyroidectomy. Which information is most important to communicate to the surgeon? a. The patient reports 7/10 incisional pain. b. The patient has increasing neck swelling. c. The patient is sleepy and difficult to arouse. d. The patient's cardiac rate is 112 beats/minute.

ANS: B The neck swelling may lead to respiratory difficulty, and rapid intervention is needed to prevent airway obstruction. The incisional pain should be treated but is not unusual after surgery. A heart rate of 112 is not unusual in a patient who has been hyperthyroid and has just arrived in the PACU from surgery. Sleepiness in the immediate postoperative period is expected.

A 38-year-old male patient is admitted to the hospital in Addisonian crisis. Which patient statement supports a nursing diagnosis of ineffective self-health management related to lack of knowledge about management of Addison's disease? a. "I frequently eat at restaurants, and my food has a lot of added salt." b. "I had the stomach flu earlier this week, so I couldn't take the hydrocortisone." c. "I always double my dose of hydrocortisone on the days that I go for a long run." d. "I take twice as much hydrocortisone in the morning dose as I do in the afternoon."

ANS: B The need for hydrocortisone replacement is increased with stressors such as illness, and the patient needs to be taught to call the health care provider because medication and IV fluids and electrolytes may need to be given. The other patient statements indicate appropriate management of the Addison's disease.

Which laboratory value reported to the nurse by the unlicensed assistive personnel (UAP) indicates the most urgent need for the nurse's assessment of the patient? a. Bedtime glucose of 140 mg/dL b. Noon blood glucose of 52 mg/dL c. Fasting blood glucose of 130 mg/dL d. 2-hr postprandial glucose of 220 mg/dL

ANS: B The nurse should assess the patient with a blood glucose level of 52 mg/dL for symptoms of hypoglycemia and give the patient a carbohydrate-containing beverage such as orange juice. The other values are within an acceptable range or not immediately dangerous for a diabetic patient

The nurse identifies a need for additional teaching when the patient who is self-monitoring blood glucose a. washes the puncture site using warm water and soap. b. chooses a puncture site in the center of the finger pad. c. hangs the arm down for a minute before puncturing the site. d. says the result of 120 mg indicates good blood sugar control.

ANS: B The patient is taught to choose a puncture site at the side of the finger pad because there are fewer nerve endings along the side of the finger pad. The other patient actions indicate that teaching has been effective

Which nursing action will be included in the plan of care for a 55-year-old patient with Graves' disease who has exophthalmos? a. Place cold packs on the eyes to relieve pain and swelling. b. Elevate the head of the patient's bed to reduce periorbital fluid. c. Apply alternating eye patches to protect the corneas from irritation. d. Teach the patient to blink every few seconds to lubricate the corneas.

ANS: B The patient should sit upright as much as possible to promote fluid drainage from the periorbital area. With exophthalmos, the patient is unable to close the eyes completely to blink. Lubrication of the eyes, rather than eye patches, will protect the eyes from developing corneal scarring. The swelling of the eye is not caused by excessive blood flow to the eye, so cold packs will not be helpful.

A 63-year-old patient with primary hyperparathyroidism has a serum phosphorus level of 1.7 mg/dL (0.55 mmol/L) and calcium of 14 mg/dL (3.5 mmol/L). Which nursing action should be included in the plan of care? a. Restrict the patient to bed rest. b. Encourage 4000 mL of fluids daily. c. Institute routine seizure precautions. d. Assess for positive Chvostek's sign.

ANS: B The patient with hypercalcemia is at risk for kidney stones, which may be prevented by a high fluid intake. Seizure precautions and monitoring for Chvostek's or Trousseau's sign are appropriate for hypocalcemic patients. The patient should engage in weight-bearing exercise to decrease calcium loss from bone.

A patient who had a subtotal thyroidectomy earlier today develops laryngeal stridor and a cramp in the right hand upon returning to the surgical nursing unit. Which collaborative action will the nurse anticipate next? a. Suction the patient's airway. b. Administer IV calcium gluconate. c. Plan for emergency tracheostomy. d. Prepare for endotracheal intubation.

ANS: B The patient's clinical manifestations of stridor and cramping are consistent with tetany caused by hypocalcemia resulting from damage to the parathyroid glands during surgery. Endotracheal intubation or tracheostomy may be needed if the calcium does not resolve the stridor. Suctioning will not correct the stridor.

Which information will the nurse include when teaching a 50-year-old patient who has type 2 diabetes about glyburide (Micronase, DiaBeta, Glynase)? a. Glyburide decreases glucagon secretion from the pancreas. b. Glyburide stimulates insulin production and release from the pancreas. c. Glyburide should be taken even if the morning blood glucose level is low. d. Glyburide should not be used for 48 hours after receiving IV contrast media.

ANS: B The sulfonylureas stimulate the production and release of insulin from the pancreas. If the glucose level is low, the patient should contact the health care provider before taking the glyburide, because hypoglycemia can occur with this class of medication. Metformin should be held for 48 hours after administration of IV contrast media, but this is not necessary for glyburide. Glucagon secretion is not affected by glyburide

An active 28-year-old male with type 1 diabetes is being seen in the endocrine clinic. Which finding may indicate the need for a change in therapy? a. Hemoglobin A1C level 6.2% b. Blood pressure 146/88 mmHg c. Heart rate at rest 58 beats/minute d. High density lipoprotein (HDL) level 65 mg/dL

ANS: B To decrease the incidence of macrovascular and microvascular problems in patients with diabetes, the goal blood pressure is usually 130/80. An A1C less than 6.5%, a low resting heart rate (consistent with regular aerobic exercise in a young adult), and an HDL level of 65 mg/dL all indicate that the patient's diabetes and risk factors for vascular disease are well controlled.

A diabetic patient who has reported burning foot pain at night receives a new prescription. Which information should the nurse teach the patient about amitriptyline (Elavil)? a. Amitriptyline decreases the depression caused by your foot pain. b. Amitriptyline helps prevent transmission of pain impulses to the brain. c. Amitriptyline corrects some of the blood vessel changes that cause pain. d. Amitriptyline improves sleep and makes you less aware of nighttime pain.

ANS: B Tricyclic antidepressants decrease the transmission of pain impulses to the spinal cord and brain. Tricyclic antidepressants also improve sleep quality and are used for depression, but that is not the major purpose for their use in diabetic neuropathy. The blood vessel changes that contribute to neuropathy are not affected by tricyclic antidepressants

A 56-year-old patient who is disoriented and reports a headache and muscle cramps is hospitalized with possible syndrome of inappropriate antidiuretic hormone (SIADH). The nurse would expect the initial laboratory results to include a(n) a. elevated hematocrit. b. decreased serum sodium. c. low urine specific gravity. d. increased serum chloride.

ANS: B When water is retained, the serum sodium level will drop below normal, causing the clinical manifestations reported by the patient. The hematocrit will decrease because of the dilution caused by water retention. Urine will be more concentrated with a higher specific gravity. The serum chloride level will usually decrease along with the sodium level.

To monitor for complications in a patient with type 2 diabetes, which tests will the nurse in the diabetic clinic schedule at least annually (select all that apply)? a. Chest x-ray b. Blood pressure c. Serum creatinine d. Urine for microalbuminuria e. Complete blood count (CBC) f. Monofilament testing of the foot

ANS: B, C, D, F Blood pressure, serum creatinine, urine testing for microalbuminuria, and monofilament testing of the foot are recommended at least annually to screen for possible microvascular and macrovascular complications of diabetes. Chest x-ray and CBC might be ordered if the diabetic patient presents with symptoms of respiratory or infectious problems but are not routinely included in screening

The nurse is caring for a patient following an adrenalectomy. The highest priority in the immediate postoperative period is to a. protect the patient's skin. b. monitor for signs of infection. c. balance fluids and electrolytes. d. prevent emotional disturbances.

ANS: C After adrenalectomy, the patient is at risk for circulatory instability caused by fluctuating hormone levels, and the focus of care is to assess and maintain fluid and electrolyte status through the use of IV fluids and corticosteroids. The other goals are also important for the patient but are not as immediately life threatening as the circulatory collapse that can occur with fluid and electrolyte disturbances.

A patient has just arrived on the unit after a thyroidectomy. Which action should the nurse take first? a. Observe the dressing for bleeding. b. Check the blood pressure and pulse. c. Assess the patient's respiratory effort. d. Support the patient's head with pillows.

ANS: C Airway obstruction is a possible complication after thyroidectomy because of swelling or bleeding at the site or tetany. The priority nursing action is to assess the airway. The other actions are also part of the standard nursing care postthyroidectomy but are not as high of a priority.

After a 22-year-old female patient with a pituitary adenoma has had a hypophysectomy, the nurse will teach about the need for a. sodium restriction to prevent fluid retention. b. insulin to maintain normal blood glucose levels. c. oral corticosteroids to replace endogenous cortisol. d. chemotherapy to prevent malignant tumor recurrence.

ANS: C Antidiuretic hormone (ADH), cortisol, and thyroid hormone replacement will be needed for life after hypophysectomy. Without the effects of adrenocorticotropic hormone (ACTH) and cortisol, the blood glucose and serum sodium will be low unless cortisol is replaced. An adenoma is a benign tumor, and chemotherapy will not be needed.

Which statement by a nurse to a patient newly diagnosed with type 2 diabetes is correct? a. Insulin is not used to control blood glucose in patients with type 2 diabetes. b. Complications of type 2 diabetes are less serious than those of type 1 diabetes. c. Changes in diet and exercise may control blood glucose levels in type 2 diabetes. d. Type 2 diabetes is usually diagnosed when the patient is admitted with a hyperglycemic coma.

ANS: C For some patients with type 2 diabetes, changes in lifestyle are sufficient to achieve blood glucose control. Insulin is frequently used for type 2 diabetes, complications are equally severe as for type 1 diabetes, and type 2 diabetes is usually diagnosed with routine laboratory testing or after a patient develops complications such as frequent yeast infections.

The nurse is interviewing a new patient with diabetes who receives rosiglitazone (Avandia) through a restricted access medication program. What is most important for the nurse to report immediately to the health care provider? a. The patient's blood pressure is 154/92. b. The patient has a history of emphysema. c. The patient's blood glucose is 86 mg/dL. d. The patient has chest pressure when walking.

ANS: D Rosiglitazone can cause myocardial ischemia. The nurse should immediately notify the health care provider and expect orders to discontinue the medication. There is no urgent need to discuss the other data with the health care provider

When a patient who takes metformin (Glucophage) to manage type 2 diabetes develops an allergic rash from an unknown cause, the health care provider prescribes prednisone (Deltasone). The nurse will anticipate that the patient may a. need a diet higher in calories while receiving prednisone. b. develop acute hypoglycemia while taking the prednisone. c. require administration of insulin while taking prednisone. d. have rashes caused by metformin-prednisone interactions.

ANS: C Glucose levels increase when patients are taking corticosteroids, and insulin may be required to control blood glucose. Hypoglycemia is not a side effect of prednisone. Rashes are not an adverse effect caused by taking metformin and prednisone simultaneously. The patient may have an increased appetite when taking prednisone, but will not need a diet that is higher in calories

A 26-year-old female with type 1 diabetes develops a sore throat and runny nose after caring for her sick toddler. The patient calls the clinic for advice about her symptoms and a blood glucose level of 210 mg/dL despite taking her usual glargine (Lantus) and lispro (Humalog) insulin. The nurse advises the patient to a. use only the lispro insulin until the symptoms are resolved. b. limit intake of calories until the glucose is less than 120 mg/dL. c. monitor blood glucose every 4 hours and notify the clinic if it continues to rise. d. decrease intake of carbohydrates until glycosylated hemoglobin is less than 7%.

ANS: C Infection and other stressors increase blood glucose levels and the patient will need to test blood glucose frequently, treat elevations appropriately with lispro insulin, and call the health care provider if glucose levels continue to be elevated. Discontinuing the glargine will contribute to hyperglycemia and may lead to diabetic ketoacidosis (DKA). Decreasing carbohydrate or caloric intake is not appropriate because the patient will need more calories when ill. Glycosylated hemoglobin testing is not used to evaluate short-term alterations in blood glucose.

An expected nursing diagnosis for a 30-year-old patient admitted to the hospital with symptoms of diabetes insipidus is a. excess fluid volume related to intake greater than output. b. impaired gas exchange related to fluid retention in lungs. c. sleep pattern disturbance related to frequent waking to void. d. risk for impaired skin integrity related to generalized edema.

ANS: C Nocturia occurs as a result of the polyuria caused by diabetes insipidus. Edema, excess fluid volume, and fluid retention are not expected.

A 37-year-old patient is being admitted with a diagnosis of Cushing syndrome. Which findings will the nurse expect during the assessment? a. Chronically low blood pressure b. Bronzed appearance of the skin c. Purplish streaks on the abdomen d. Decreased axillary and pubic hair

ANS: C Purplish-red striae on the abdomen are a common clinical manifestation of Cushing syndrome. Hypotension and bronzed-appearing skin are manifestations of Addison's disease. Decreased axillary and pubic hair occur with androgen deficiency.

After the nurse has finished teaching a patient who has a new prescription for exenatide (Byetta), which patient statement indicates that the teaching has been effective? a. "I may feel hungrier than usual when I take this medicine." b. "I will not need to worry about hypoglycemia with the Byetta." c. "I should take my daily aspirin at least an hour before the Byetta." d. "I will take the pill at the same time I eat breakfast in the morning."

ANS: C Since exenatide slows gastric emptying, oral medications should be taken at least an hour before the exenatide to avoid slowing absorption. Exenatide is injected and increases feelings of satiety. Hypoglycemia can occur with this medication

Which assessment finding of a 42-year-old patient who had a bilateral adrenalectomy requires the most rapid action by the nurse? a. The blood glucose is 176 mg/dL. b. The lungs have bibasilar crackles. c. The blood pressure (BP) is 88/50 mm Hg. d. The patient reports 5/10 incisional pain.

ANS: C The decreased BP indicates possible adrenal insufficiency. The nurse should immediately notify the health care provider so that corticosteroid medications can be administered. The nurse should also address the elevated glucose, incisional pain, and crackles with appropriate collaborative or nursing actions, but prevention and treatment of acute adrenal insufficiency is the priority after adrenalectomy.

A patient who was admitted with diabetic ketoacidosis secondary to a urinary tract infection has been weaned off an insulin drip 30 minutes ago. The patient reports feeling lightheaded and sweaty. Which action should the nurse take first? a. Infuse dextrose 50% by slow IV push. b. Administer 1 mg glucagon subcutaneously. c. Obtain a glucose reading using a finger stick. d. Have the patient drink 4 ounces of orange juice.

ANS: C The patient's clinical manifestations are consistent with hypoglycemia and the initial action should be to check the patient's glucose with a finger stick or order a stat blood glucose. If the glucose is low, the patient should ingest a rapid-acting carbohydrate, such as orange juice. Glucagon or dextrose 50% might be given if the patient's symptoms become worse or if the patient is unconscious

After change-of-shift report, which patient will the nurse assess first? a. 19-year-old with type 1 diabetes who was admitted with possible dawn phenomenon b. 35-year-old with type 1 diabetes whose most recent blood glucose reading was 230 mg/dL c. 60-year-old with hyperosmolar hyperglycemic syndrome who has poor skin turgor and dry oral mucosa d. 68-year-old with type 2 diabetes who has severe peripheral neuropathy and complains of burning foot pain

ANS: C The patient's diagnosis of HHS and signs of dehydration indicate that the nurse should rapidly assess for signs of shock and determine whether increased fluid infusion is needed. The other patients also need assessment and intervention but do not have life-threatening complications

A 48-year-old male patient screened for diabetes at a clinic has a fasting plasma glucose level of 120 mg/dL (6.7 mmol/L). The nurse will plan to teach the patient about a. self-monitoring of blood glucose. b. using low doses of regular insulin. c. lifestyle changes to lower blood glucose. d. effects of oral hypoglycemic medications.

ANS: C The patient's impaired fasting glucose indicates prediabetes, and the patient should be counseled about lifestyle changes to prevent the development of type 2 diabetes. The patient with prediabetes does not require insulin or oral hypoglycemics for glucose control and does not need to self-monitor blood glucose.

When caring for a patient who has an adrenocortical adenoma, causing hyperaldosteronism, the nurse should A. provide a potassium-restricted diet. B. monitor the blood pressure every 4 hours. C. monitor blood glucose level every 4 hours. D. relieve edema by elevating the extremities.

B. Monitor the blood pressure every 4 hours.

A patient develops carpopedal spasms and tingling of the lips following a parathyroidectomy. Which action should the nurse take first? a. Administer the ordered muscle relaxant. b. Give the ordered oral calcium supplement. c. Have the patient rebreathe from a paper bag. d. Start the PRN oxygen at 2 L/min per cannula.

ANS: C The patient's symptoms suggest mild hypocalcemia. The symptoms of hypocalcemia will be temporarily reduced by having the patient breathe into a paper bag, which will raise the PaCO2 and create a more acidic pH. The muscle relaxant will have no impact on the ionized calcium level. Although severe hypocalcemia can cause laryngeal stridor, there is no indication that this patient is experiencing laryngeal stridor or needs oxygen. Calcium supplements will be given to normalize calcium levels quickly, but oral supplements will take time to be absorbed.

A 62-year-old patient with hyperthyroidism is to be treated with radioactive iodine (RAI). The nurse instructs the patient a. about radioactive precautions to take with all body secretions. b. that symptoms of hyperthyroidism should be relieved in about a week. c. that symptoms of hypothyroidism may occur as the RAI therapy takes effect. d. to discontinue the antithyroid medications taken before the radioactive therapy.

ANS: C There is a high incidence of postradiation hypothyroidism after RAI, and the patient should be monitored for symptoms of hypothyroidism. RAI has a delayed response, with the maximum effect not seen for 2 to 3 months, and the patient will continue to take antithyroid medications during this time. The therapeutic dose of radioactive iodine is low enough that no radiation safety precautions are needed.

A 42-year-old female patient is scheduled for transsphenoidal hypophysectomy to treat a pituitary adenoma. During preoperative teaching, the nurse instructs the patient about the need to a. cough and deep breathe every 2 hours postoperatively. b. remain on bed rest for the first 48 hours after the surgery. c. avoid brushing teeth for at least 10 days after the surgery. d. be positioned flat with sandbags at the head postoperatively.

ANS: C To avoid disruption of the suture line, the patient should avoid brushing the teeth for 10 days after surgery. It is not necessary to remain on bed rest after this surgery. Coughing is discouraged because it may cause leakage of cerebrospinal fluid (CSF) from the suture line. The head of the bed should be elevated 30 degrees to reduce pressure on the sella turcica and decrease the risk for headaches.

Which information is most important for the nurse to report to the health care provider before a patient with type 2 diabetes is prepared for a coronary angiogram? a. The patient's most recent HbA1C was 6.5%. b. The patient's admission blood glucose is 128 mg/dL. c. The patient took the prescribed metformin (Glucophage) today. d. The patient took the prescribed captopril (Capoten) this morning.

ANS: C To avoid lactic acidosis, metformin should be discontinued a day or 2 before the coronary arteriogram and should not be used for 48 hours after IV contrast media are administered. The other patient data will also be reported but do not indicate any need to reschedule the procedure

The nurse is assessing a 22-year-old patient experiencing the onset of symptoms of type 1 diabetes. Which question is most appropriate for the nurse to ask? a. "Are you anorexic?" b. "Is your urine dark colored?" c. "Have you lost weight lately?" d. "Do you crave sugary drinks?"

ANS: C Weight loss occurs because the body is no longer able to absorb glucose and starts to break down protein and fat for energy. The patient is thirsty but does not necessarily crave sugar-containing fluids. Increased appetite is a classic symptom of type 1 diabetes. With the classic symptom of polyuria, urine will be very dilute

After obtaining the information shown in the accompanying figure regarding a patient with Addison's disease, which prescribed action will the nurse take first a. Give 4 oz of fruit juice orally. b. Recheck the blood glucose level. c. Infuse 5% dextrose and 0.9% saline. d. Administer oxygen therapy as needed.

ANS: C The patient's poor skin turgor, hypotension, and hyponatremia indicate an Addisonian crisis. Immediate correction of the hypovolemia and hyponatremia is needed. The other actions may also be needed but are not the initial action for the patient

Which information is most important for the nurse to communicate rapidly to the health care provider about a patient admitted with possible syndrome of inappropriate antidiuretic hormone (SIADH)? a. The patient has a recent weight gain of 9 lb. b. The patient complains of dyspnea with activity. c. The patient has a urine specific gravity of 1.025. d. The patient has a serum sodium level of 118 mEq/L.

ANS: D A serum sodium of less than 120 mEq/L increases the risk for complications such as seizures and needs rapid correction. The other data are not unusual for a patient with SIADH and do not indicate the need for rapid action

A hospitalized diabetic patient received 38 U of NPH insulin at 7:00 AM. At 1:00 PM, the patient has been away from the nursing unit for 2 hours, missing the lunch delivery while awaiting a chest x-ray. To prevent hypoglycemia, the best action by the nurse is to a. save the lunch tray for the patient's later return to the unit. b. ask that diagnostic testing area staff to start a 5% dextrose IV. c. send a glass of milk or orange juice to the patient in the diagnostic testing area. d. request that if testing is further delayed, the patient be returned to the unit to eat.

ANS: D Consistency for mealtimes assists with regulation of blood glucose, so the best option is for the patient to have lunch at the usual time. Waiting to eat until after the procedure is likely to cause hypoglycemia. Administration of an IV solution is unnecessarily invasive for the patient. A glass of milk or juice will keep the patient from becoming hypoglycemic but will cause a rapid rise in blood glucose because of the rapid absorption of the simple carbohydrate in these items

Which action by a patient indicates that the home health nurse's teaching about glargine and regular insulin has been successful? a. The patient administers the glargine 30 minutes before each meal. b. The patient's family prefills the syringes with the mix of insulins weekly. c. The patient draws up the regular insulin and then the glargine in the same syringe. d. The patient disposes of the open vials of glargine and regular insulin after 4 weeks.

ANS: D Insulin can be stored at room temperature for 4 weeks. Glargine should not be mixed with other insulins or prefilled and stored. Short-acting regular insulin is administered before meals, while glargine is given once daily.

A patient with type 2 diabetes is scheduled for a follow-up visit in the clinic several months from now. Which test will the nurse schedule to evaluate the effectiveness of treatment for the patient? a. Urine dipstick for glucose b. Oral glucose tolerance test c. Fasting blood glucose level d. Glycosylated hemoglobin level

ANS: D The glycosylated hemoglobin (A1C or HbA1C) test shows the overall control of glucose over 90 to 120 days. A fasting blood level indicates only the glucose level at one time. Urine glucose testing is not an accurate reflection of blood glucose level and does not reflect the glucose over a prolonged time. Oral glucose tolerance testing is done to diagnose diabetes, but is not used for monitoring glucose control once diabetes has been diagnosed

A 44-year-old female patient with Cushing syndrome is admitted for adrenalectomy. Which intervention by the nurse will be most helpful for a nursing diagnosis of disturbed body image related to changes in appearance? a. Reassure the patient that the physical changes are very common in patients with Cushing syndrome. b. Discuss the use of diet and exercise in controlling the weight gain associated with Cushing syndrome. c. Teach the patient that the metabolic impact of Cushing syndrome is of more importance than appearance. d. Remind the patient that most of the physical changes caused by Cushing syndrome will resolve after surgery.

ANS: D The most reassuring communication to the patient is that the physical and emotional changes caused by the Cushing syndrome will resolve after hormone levels return to normal postoperatively. Reassurance that the physical changes are expected or that there are more serious physiologic problems associated with Cushing syndrome are not therapeutic responses. The patient's physiological changes are caused by the high hormone levels, not by the patient's diet or exercise choices.

A 28-year-old male patient with type 1 diabetes reports how he manages his exercise and glucose control. Which behavior indicates that the nurse should implement additional teaching? a. The patient always carries hard candies when engaging in exercise. b. The patient goes for a vigorous walk when his glucose is 200 mg/dL. c. The patient has a peanut butter sandwich before going for a bicycle ride. d. The patient increases daily exercise when ketones are present in the urine.

ANS: D When the patient is ketotic, exercise may result in an increase in blood glucose level. Type 1 diabetic patients should be taught to avoid exercise when ketosis is present. The other statements are correct

Which information will the nurse include when teaching a patient who has been newly diagnosed with Graves' disease? A. Exercise is contraindicated to avoid increasing metabolic rate. B. Restriction of iodine intake is needed to reduce thyroid activity. C. Surgery will eventually be required to remove the thyroid gland. D. Antithyroid medications may take several weeks to have an effect.

Antithyroid medications may take several weeks to have an effect. Feedback Medications used to block the synthesis of thyroid hormones may take several weeks before an effect is seen. Large doses of iodine are used to inhibit the synthesis of thyroid hormones. Exercise using large muscle groups is encouraged to decrease the irritability and hyperactivity associated with high levels of thyroid hormones. Radioactive iodine is the most common treatment for Graves' disease, although surgery may be used

A 72-year-old patient is diagnosed with hypothyroidism and levothyroxine (Synthroid) is prescribed. Which assessment is most important for the nurse to make during initiation of thyroid replacement? A. Apical pulse rate B. Nutritional intake C. Intake and output D. Orientation and alertness

Apical pulse rate Feedback In older patients, initiation of levothyroxine therapy can increase myocardial oxygen demand and cause angina or dysrhythmias. The medication also is expected to improve mental status and fluid balance and will increase metabolic rate and nutritional needs, but these changes will not result in potentially life-threatening complications.

Which action should the nurse take first when caring for a patient who has just arrived on the unit after a thyroidectomy? A. Check the dressing for bleeding. B. Assess respiratory rate and effort. C. Take the blood pressure and pulse. D. Support the patient's head with pillows.

Assess respiratory rate and effort. ANS: B Airway obstruction is a possible complication after thyroidectomy because of swelling or bleeding at the site or tetany, and the priority nursing action is to assess the airway. The other actions also are part of the standard nursing care postthyroidectomy but are not as high in priority.

During preoperative teaching for a patient scheduled for transsphenoidal hypophysectomy for treatment of a pituitary adenoma, the nurse instructs the patient about the need to A. Cough and deep breathe every 2 hours postoperatively. B. Remain on bed rest for the first 48 hours after the surgery. C. Be positioned flat with sandbags at the head postoperatively. D. Avoid brushing the teeth for at least 10 days after the surgery.

Avoid brushing the teeth for at least 10 days after the surgery. Feedback To avoid disruption of the suture line, the patient should avoid brushing the teeth for 10 days after surgery. It is not necessary to remain on bed rest after this surgery. Coughing is discouraged because it may cause leakage of cerebrospinal fluid (CSF) from the suture line. The head of the bed should be elevated 30 degrees to reduce pressure on the sella turcica and

The first nursing action indicated when a patient returns to the surgical nursing unit following a thyroidectomy is to a. check the dressing for bleeding. b. assess respiratory rate and effort. c. support the patient's head with pillows. d. take the blood pressure and pulse.

B R: Airway obstruction is a possible complication after thyroidectomy because of swelling or bleeding at the site or tetany, and priority nursing action is to assess airway. The other actions are also part of the standard nursing care post-thyroidectomy but are not as high in priority.

A patient is hospitalized with acute adrenal insufficiency. The nurse determines that the pt is responding favorably to treatment upon finding a. decreasing serum sodium. b. decreasing serum potassium. c. decreasing blood glucose. d. increasing urinary output.

B R: CMs of Addison's disease include hyperkalemia and a decrease in potassium level indicates improvement. Decreasing serum sodium and decreasing blood glucose indicate that treatment has not been effective. Changes in urinary output are not an effective way of monitoring treatment for Addison's disease.

Following a thyroidectomy, a patient develops carpal spasm while the nurse is taking a blood pressure on the left arm. Which action by the nurse is appropriate? a. Administer the ordered muscle relaxant. b. Have the patient rebreathe using a paper bag. c. Start oxygen at 2 to 3 L/min per cannula. d. Give the ordered oral calcium supplement.

B R: Carpal spasm after a thyroidectomy suggests that pt has hypocalcaemia caused by damage to the parathyroid glands. The symptoms of hypocalcemia will be temporarily reduced by having the patient breath into a paper bag, which will raise the PaCO2 and create a more acidic pH. The muscle relaxant will not impact on ionized calcium level. There is no indication that the patient is experiencing laryngeal stridor or needs oxygen. IV calcium supplements will be given to normalize calcium level quickly.

When providing postoperative care for a patient who has had bilateral adrenalectomy, which assessment information obtained by the nurse is most important to communicate to HCP? a. The blood glucose is 156 mg/dl. b. The patient's blood pressure is 102/50. c. The patient has 5/10 incisional pain. d. The lungs have bibasilar crackles.

B R: During immediate postoperative period, marked fluctuation in cortisol levels may occur and the nurse must be alert for signs of acute adrenal insufficiency such as hypotension. nurse should also address elevated glucose, incisional pain, and crackles with appropriate collaborative or nursing actions, but prevention and treatment of acute adrenal insufficiency is the priority after adrenalectomy.

When developing a plan of care for a pt with SIADH, which interventions will the nurse include? a. Encourage fluids to 2000 ml/day. b. Offer patient hard candies to suck on. c. Monitor for increased peripheral edema. d. Keep head of bed elevated to 30 degrees.

B R: Sucking on hard candies decreases thirst for patient on a fluid restriction. Pts with SIADH are on fluid restrictions of 800-1000 ml/day. Peripheral edema isnt seen w SIADH. HOB is elevated no more than 10 degrees to increase left atrial filling pressure and decrease ADH release. (Cognitive Level: Application Text Reference: p. 1296 NProcess: Planning NCLEX: Physiological Integrity)

A patient is admitted to the hospital in addisonian crisis 1 month after a diagnosis of Addison's disease. The nurse identifies the nursing diagnosis of ineffective therapeutic regimen management related to lack of knowledge of management of condition when the patient says, a. "I double my dose of hydrocortisone on the days that I go for a run." b. "I had the stomach flu earlier this week and couldn't take the hydrocortisone." c. "I frequently eat at restaurants, and so my food has a lot of added salt." d. "I do yoga exercises almost every day to help me reduce stress and relax."

B R: The need for hydrocortisone replacement is increased with stressors such as illness, and the patient needs to be taught to call the health care provider because medication and IV fluids and electrolytes may need to be given. The other patient statements indicate appropriate management of the Addison's disease.

The nurse identifies a nursing dx of risk for injury: corneal ulceration related to inability to close the eyelids secondary to exophthalmos for a patient with Graves' disease. An appropriate nursing intervention for this problem is to a. teach the patient to blink every few seconds to lubricate the cornea. b. elevate the head of the patient's bed to reduce periorbital fluid. c. apply eye patches to protect the cornea from irritation. d. place cold packs on the eyes to relieve pain and swelling.

B R: The patient should sit upright as much as possible to promote fluid drainage from the periorbital area. With exophthalmos, the pt is unable to close eyes completely. Lubrication of the eyes, rather than eye patches, will protect the eyes from developing corneal scarring. The swelling of the eye is not caused by excessive blood flow to the eye, so cold packs will not be helpful.

Which clinical manifestations may be seen in a client experiencing diabetic ketoacidosis (hyperglycemia)? (Select all that apply.) a. thirst b. polyuria c. bradycardia d. Kussmaul's sign e. dry mucous membranes f. fruity breath odor

a. thirst b. polyuria d. Kussmaul's sign e. dry mucous membranes f. fruity breath odor

A few hours after returning to the surgical nursing unit, a patient who has undergone a subtotal thyroidectomy develops laryngeal stridor and a cramp in the right hand. The nurse anticipates that intervention will include a. administration of IV morphine. b. administration of IV calcium gluconate. c. endotracheal intubation with mechanical ventilation. d. immediate tracheostomy and manual ventilation.

B R: The pt's CMs are consistent with tetany caused by hypocalcemia resulting from damage to the parathyroid glands during surgery. Tracheostomy may be needed if the calcium does not resolve the stridor. There is no indication that morphine is needed. Endotracheal intubation may be done, but only if calcium is not effective in correcting stridor

A newly diagnosed type 1 diabetic patient likes to run 3 miles several mornings a week. Which teaching will the nurse implement about exercise for this patient? a. "You should not take the morning NPH insulin before you run." b. "Plan to eat breakfast about an hour before your run." c. "Afternoon running is less likely to cause hypoglycemia." d. "You may want to run a little farther if your glucose is very high."

B Rationale: Blood sugar increases after meals, so this will be the best time to exercise. NPH insulin will not peak until mid-afternoon and is safe to take before a morning run. Running can be done in either the morning or afternoon. If the glucose is very elevated, the patient should postpone the run. Cognitive Level: Application Text Reference: p. 1269 Nursing Process: Implementation NCLEX: Physiological Integrity

A patient is suspected of having a pituitary tumor causing panhypopituitarism. During assessment of the pt, the nurse would expect to find a. elevated blood glucose. b. changes in secondary sex characteristics. c. high blood pressure. d. tachycardia and cardiac palpitations.

B Rationale: Changes in secondary sex characteristics are associated with decreases in FSH and LH. Fasting hypoglycemia and hypotension occur in panhypopituitarism as a result of decreases in ACTH and cortisol. Bradycardia is likely due to the decrease in TSH and thyroid hormones associated with panhypopituitarism. (Cognitive Level: Application Text Reference: p. 1294 NProcess: Assessment NCLEX: Physiological Integrity)

A 1200-calorie diet and exercise are prescribed for a patient with newly diagnosed type 2 diabetes. The patient tells the nurse, "I hate to exercise! Can't I just follow the diet to keep my glucose under control?" The nurse teaches the patient that the major purpose of exercise for diabetics is to a. increase energy and sense of well-being, which will help with body image. b. facilitate weight loss, which will decrease peripheral insulin resistance. c. improve cardiovascular endurance, which is important for diabetics. d. set a successful pattern, which will help in making other needed changes.

B Rationale: Exercise is essential to decrease insulin resistance and improve blood glucose control. Increased energy, improved cardiovascular endurance, and setting a pattern of success are secondary benefits of exercise, but they are not the major reason. Cognitive Level: Application Text Reference: p. 1269 Nursing Process: Implementation NCLEX: Physiological Integrity

A patient with type 1 diabetes has an unusually high morning glucose measurement, and the health care provider wants the patient evaluated for possible Somogyi effect. The nurse will plan to a. administer an increased dose of NPH insulin in the evening. b. obtain the patient's blood glucose at 3:00 in the morning. c. withhold the nighttime snack and check the glucose at 6:00 AM. d. check the patient for symptoms of hypoglycemia at 2:00 to 4:00 AM.

B Rationale: In the Somogyi effect, the patient's blood glucose drops in the early morning hours (in response to excess insulin administration), which causes the release of hormones that result in a rebound hyperglycemia. It is important to check the blood glucose in the early morning hours to detect the initial hypoglycemia. An increased evening NPH dose or holding the nighttime snack will further increase the risk for early morning hypoglycemia. Information about symptoms of hypoglycemia will not be as accurate as checking the patient's blood glucose in determining whether the patient has the Somogyi effect. Cognitive Level: Application Text Reference: pp. 1263-1264 Nursing Process: Planning NCLEX: Physiological Integrity

A patient with newly diagnosed type 2 diabetes mellitus asks the nurse what "type 2" means in relation to diabetes. The nurse explains to the patient that type 2 diabetes differs from type 1 diabetes primarily in that with type 2 diabetes a. the patient is totally dependent on an outside source of insulin. b. there is decreased insulin secretion and cellular resistance to insulin that is produced. c. the immune system destroys the pancreatic insulin-producing cells. d. the insulin precursor that is secreted by the pancreas is not activated by the liver.

B Rationale: In type 2 diabetes, the pancreas produces insulin, but the insulin is insufficient for the body's needs or the cells do not respond to the insulin appropriately. The other information describes the physiology of type 1 diabetes. Cognitive Level: Application Text Reference: p. 1255 Nursing Process: Implementation NCLEX: Physiological Integrity

A patient with type 1 diabetes who uses glargine (Lantus) and lispro (Humalog) insulin develops a sore throat, cough, and fever. When the patient calls the clinic to report the symptoms and a blood glucose level of 210 mg/dl, the nurse advises the patient to a. use only the lispro insulin until the symptoms of infection are resolved. b. monitor blood glucose every 4 hours and notify the clinic if it continues to rise. c. decrease intake of carbohydrates until glycosylated hemoglobin is less than 7%. d. limit intake to non-calorie-containing liquids until the glucose is within the usual range.

B Rationale: Infection and other stressors increase blood glucose levels and the patient will need to test blood glucose frequently, treat elevations appropriately with insulin, and call the health care provider if glucose levels continue to be elevated. Discontinuing the glargine will contribute to hyperglycemia and may lead to DKA. Decreasing carbohydrate or caloric intake is not appropriate as the patient will need more calories when ill. Glycosylated hemoglobins are not used to test for short-term alterations in blood glucose. Cognitive Level: Application Text Reference: p. 1272 Nursing Process: Implementation NCLEX: Health Promotion and Maintenance

During a diabetes screening program, a patient tells the nurse, "My mother died of complications of type 2 diabetes. Can I inherit diabetes?" The nurse explains that a. as long as the patient maintains normal weight and exercises, type 2 diabetes can be prevented. b. the patient is at a higher than normal risk for type 2 diabetes and should have periodic blood glucose level testing. c. there is a greater risk for children developing type 2 diabetes when the father has type 2 diabetes. d. although there is a tendency for children of people with type 2 diabetes to develop diabetes, the risk is higher for those with type 1 diabetes.

B Rationale: Offspring of people with type 2 diabetes are at higher risk for developing type 2 diabetes. The risk can be decreased, but not prevented, by maintenance of normal weight and exercising. The risk for children of a person with type 1 diabetes to develop diabetes is higher when it is the father who has the disease. Offspring of people with type 2 diabetes are more likely to develop diabetes than offspring of those with type 1 diabetes. Cognitive Level: Application Text Reference: p. 1256 Nursing Process: Implementation NCLEX: Physiological Integrity

The health care provider orders oral glucose tolerance testing for a patient seen in the clinic. Which information from the patient's health history is most important for the nurse to communicate to the health care provider? a. The patient had a viral illness 2 months ago. b. The patient uses oral contraceptives. c. The patient runs several days a week. d. The patient has a family history of diabetes.

B Rationale: Oral contraceptive use may falsely elevate oral glucose tolerance test (OGTT) values. A viral 2 months previously illness may be associated with the onset of type 1 diabetes but will not falsely impact the OGTT. Exercise and a family history of diabetes both can affect blood glucose but will not lead to misleading information from the OGTT. Cognitive Level: Application Text Reference: p. 1267 Nursing Process: Assessment NCLEX: Physiological Integrity

A diabetic patient is started on intensive insulin therapy. The nurse will plan to teach the patient about mealtime coverage using _____ insulin. a. NPH b. lispro c. detemir d. glargine

B Rationale: Rapid or short acting insulin is used for mealtime coverage for patients receiving intensive insulin therapy. NPH, glargine, or detemir will be used as the basal insulin. Cognitive Level: Application Text Reference: p. 1260 Nursing Process: Planning NCLEX: Physiological Integrity

A patient receives a daily injection of 70/30 NPH/regular insulin premix at 7:00 AM. The nurse expects that a hypoglycemic reaction is most likely to occur between a. 8:00 and 10:00 AM. b. 4:00 and 6:00 PM. c. 7:00 and 9:00 PM. d. 10:00 PM and 12:00 AM.

B Rationale: The greatest insulin effect with this combination occurs mid afternoon. The patient is not at a high risk at the other listed times, although hypoglycemia may occur. Cognitive Level: Comprehension Text Reference: p. 1260 Nursing Process: Evaluation NCLEX: Physiological Integrity

Which of these laboratory values noted by the nurse when reviewing the chart of a diabetic patient indicates the need for further assessment of the patient? a. Fasting blood glucose of 130 mg/dl b. Noon blood glucose of 52 mg/dl c. Glycosylated hemoglobin of 6.9% d. Hemoglobin A1C of 5.8%

B Rationale: The nurse should assess the patient with a blood glucose level of 52 mg/dl for symptoms of hypoglycemia, and give the patient some carbohydrate-containing beverage such as orange juice. The other values are within an acceptable range for a diabetic patient. Cognitive Level: Application Text Reference: pp. 1281-1282 Nursing Process: Assessment NCLEX: Physiological Integrity

A patient with type 2 diabetes has sensory neuropathy of the feet and legs and peripheral vascular disease evidenced by decreased peripheral pulses and dependent rubor. The nurse teaches the patient that a. the feet should be soaked in warm water on a daily basis. b. flat-soled leather shoes are the best choice to protect the feet from injury. c. heating pads should always be set at a very low temperature. d. over-the-counter (OTC) callus remover may be used to remove callus and prevent pressure.

B Rationale: The patient is taught to avoid high heels and that leather shoes are preferred. The feet should be washed, but not soaked, in warm water daily. Heating pad use should be avoided. Commercial callus and corn removers should be avoided; the patient should see a specialist to treat these problems. Cognitive Level: Application Text Reference: p. 1287 Nursing Process: Implementation NCLEX: Health Promotion and Maintenance

A patient with type 2 diabetes is scheduled for an outpatient coronary arteriogram. Which information obtained by the nurse when admitting the patient indicates a need for a change in the patient's regimen? a. The patient's most recent hemoglobin A1C was 6%. b. The patient takes metformin (Glucophage) every morning. c. The patient uses captopril (Capoten) for hypertension. d. The patient's admission blood glucose is 128 mg/dl.

B Rationale: To avoid lactic acidosis, metformin should not be used for 48 hours after IV contrast media are administered. The other patient data indicate that the patient is managing the diabetes appropriately. Cognitive Level: Application Text Reference: p. 1266 Nursing Process: Assessment NCLEX: Physiological Integrity

The nurse has taught a patient admitted with diabetes, cellulitis, and osteomyelitis about the principles of foot care. The nurse evaluates that the patient understands the principles of foot care if the patient makes what statement? A) "I should only walk barefoot in nice dry weather." B) "I should look at the condition of my feet every day." C) "I am lucky my shoes fit so nice and tight because they give me firm support." D) "When I am allowed up out of bed, I should check the shower water with my toes."

B) "I should look at the condition of my feet every day." Patients with diabetes mellitus need to inspect their feet daily for broken areas that are at risk for infection and delayed wound healing. Properly fitted (not tight) shoes should be worn at all times. Water temperature should be tested with the hands first.

The nurse receives a phone call from a 36-year-old woman taking cyclophosphamide (Cytoxan) for treatment of non-Hodgkin's lymphoma. The patient tells the nurse that she has muscle cramps and weakness and very little urine output. Which response by the nurse is best? A. "Start taking supplemental potassium, calcium, and magnesium." B. "Stop taking the medication now and call your health care provider." C. "These symptoms will decrease with continued use of the medication." D. "Increase fluids to 3000 mL per 24 hours to improve your urine output."

B. Stop taking the medication now and call your health care provider." Cyclophosphamide may cause syndrome of inappropriate antidiuretic hormone (SIADH). Medications that stimulate the release of ADH should be avoided or discontinued. Treatment may include restriction of fluids to 800 to 1000 mL per day. If a loop diuretic such as furosemide (Lasix) is used to promote diuresis, supplements of potassium, calcium, and magnesium may be needed.

The nurse is beginning to teach a diabetic patient about vascular complications of diabetes. What information is appropriate for the nurse to include? A) Macroangiopathy does not occur in type 1 diabetes but rather in type 2 diabetics who have severe disease. B) Microangiopathy is specific to diabetes and most commonly affects the capillary membranes of the eyes, kidneys, and skin. C) Renal damage resulting from changes in large- and medium-sized blood vessels can be prevented by careful glucose control. D) Macroangiopathy causes slowed gastric emptying and the sexual impotency experienced by a majority of patients with diabetes.

B) Microangiopathy is specific to diabetes and most commonly affects the capillary membranes of the eyes, kidneys, and skin. Microangiopathy occurs in diabetes mellitus. When it affects the eyes, it is called diabetic retinopathy. When the kidneys are affected, the patient has nephropathy. When the skin is affected, it can lead to diabetic foot ulcers. Macroangiopathy can occur in either type 1 or type 2 diabetes and contributes to cerebrovascular, cardiovascular, and peripheral vascular disease.

A 51-year-old patient with diabetes mellitus is scheduled for a fasting blood glucose level at 8:00 AM. The nurse instructs the patient to only drink water after what time? A) 6:00 PM on the evening before the test B) Midnight before the test C) 4:00 AM on the day of the test D)7:00 AM on the day of the test

B) Midnight before the test Typically, a patient is ordered to be NPO for 8 hours before a fasting blood glucose level. For this reason, the patient who has a lab draw at 8:00 AM should not have any food or beverages containing any calories after midnight.

A patient with diabetes mellitus who has multiple infections every year needs a mitral valve replacement. What is the most important preoperative teaching the nurse should provide to prevent a cardiac infection postoperatively? A) Avoid sick people and wash hands. B) Obtain comprehensive dental care. C) Maintain hemoglobin A1c below 7%. D) Coughing and deep breathing with splinting

B) Obtain comprehensive dental care. A person with diabetes is at high risk for postoperative infections. The most important preoperative teaching to prevent a postoperative infection in the heart is to have the patient obtain comprehensive dental care because the risk of septicemia and infective endocarditis increases with poor dental health. Avoiding sick people, hand washing, maintaining hemoglobin A1c below 7%, and coughing and deep breathing with splinting would be important for any type of surgery, but not the priority with mitral valve replacement for this patient.

The nurse is providing discharge instructions to a patient with diabetes insipidus. Which instructions regarding desmopressin acetate (DDAVP) would be most appropriate? A) The patient can expect to experience weight loss resulting from increased diuresis. B) The patient should alternate nostrils during administration to prevent nasal irritation. C) The patient should monitor for symptoms of hypernatremia as a side effect of this drug. D) The patient should report any decrease in urinary elimination to the health care provider.

B) The patient should alternate nostrils during administration to prevent nasal irritation. DDAVP is used to treat diabetes insipidus by replacing the antidiuretic hormone that the patient is lacking. Inhaled DDAVP can cause nasal irritation, headache, nausea, and other signs of hyponatremia. Diuresis will be decreased and is expected, and hypernatremia should not occur.

The nurse is caring for a group of older patients in a long-term care setting. Which physical changes in the patients should the nurse investigate as signs of possible endocrine dysfunction? A. Absent reflexes, diarrhea, and hearing loss B. Hypoglycemia, delirium, and incontinence Incorrect C. Fatigue, constipation, and mental impairment Correct D. Hypotension, heat intolerance, and bradycardia

B. Hypoglycemia, delirium, and incontinence Changes of aging often mimic clinical manifestations of endocrine disorders. Clinical manifestations of endocrine dysfunction such as fatigue, constipation, or mental impairment in the older adult are often missed because they are attributed solely to aging.

The patient is brought to the ED following a car accident and is wearing medical identification that says she has Addison's disease. What should the nurse expect to be included in the collaborative care of this patient? A. Low sodium diet B. Increased glucocorticoid replacement C. Suppression of pituitary ACTH synthesis D. Elimination of mineralocorticoid replacement

B. Increased glucocorticoid replacement The patient with Addison's disease needs lifelong glucocorticoid and mineralocorticoid replacement and has an increased need with illness, injury, or stress, as this patient is experiencing. The patient with Addison's may also need a high sodium diet. Suppression of pituitary ACTH synthesis is done for Cushing syndrome. Elimination of mineralocorticoid replacement cannot be done for Addison's disease.

The patient with systemic lupus erythematosus had been diagnosed with syndrome of inappropriate antidiuretic hormone (SIADH). What should the nurse expect to include in this patient's plan of care (select all that apply)? A. Obtain weekly weights. B. Limit fluids to 1000 mL per day.* C. Monitor for signs of hypernatremia. D. Minimize turning and range of motion. E. Keep the head of the bed at 10 degrees or less elevation.

B. Limit fluids to 1000 mL per day.* E. Keep the head of the bed at 10 degrees or less elevation. The care for the patient with SIADH will include limiting fluids to 1000 mL per day or less to decrease weight, increase osmolality, and improve symptoms; and keeping the head of the bed elevated at 10 degrees or less to enhance venous return to the heart and increase left atrial filling pressure, thereby reducing the release of ADH. The weights should be done daily along with intake and output. Signs of hyponatremia should be monitored, and frequent turning, positioning, and range-of-motion exercises are important to maintain skin integrity and joint mobility.

During the nursing assessment of a patient with Graves' disease, the nurse notes a bounding, rapid pulse and systolic hypertension. Based on these assessment data, which question is important for the nurse to ask the patient? a. "Do you have any problem with frequent constipation?" b. "Have you noticed any recent decrease in your appetite?" c. "Do you ever have any chest pain?" d. "Have you had recent muscle aches?"

C R: Angina is a possible complication of Graves' disease, especially for a patient with tachycardia and hypertension. The other CMs are associated with hypothyroidism.

A patient with Graves' disease is prepared for surgery with drug therapy consisting of 4 weeks of propylthiouracil (PTU) and 10 days of iodine before surgery. When teaching the patient about the drugs, the nurse explains that the drugs are given preoperatively to a. eliminate the risk for tetany during the postoperative period. b. decrease the risk of hypometabolism during and after the surgery. c. normalize metabolism and decrease the size and vascularity of the gland. d. assist in differentiating the thyroid and parathyroid glands during surgery.

C R: Antithyroid drugs and iodine decrease the levels of thyroid hormone and the vascularity of the thyroid gland prior to surgery and lower the risk for postoperative thyrotoxicosis and hemorrhage. Postoperative tetany might be caused by removal of the parathyroid gland during thyroidectomy. The medications will tend to decrease metabolic rate. The medications will not help in differentiating the tissues of the thyroid and parathyroid glands.

A patient with an antidiuretic hormone (ADH)-secreting small-cell cancer of the lung is treated with demeclocycline (Declomycin) to control the symptoms of syndrome of inappropriate secretion of antidiuretic hormone (SIADH). The nurse determines that the demeclocycline is effective upon finding that the a. patient's daily weight is stable. b. urine specific gravity is increased. c. patient's urinary output is increased. d. peripheral edema is decreased.

C R: Demeclocycline blocks the action of ADH on the renal tubules and increases urine output. A stable body weight and an increase in urine specific gravity indicate that the SIADH is not corrected. Peripheral edema does not occur with SIADH; a sudden weight gain without edema is a common clinical manifestation of this disorder.

A patient has an adrenocortical adenoma causing hyperaldosteronism and is scheduled for laparoscopic surgery to remove the tumor. During care before surgery, the nurse should a. monitor blood glucose level every 4 hours. b. provide a potassium-restricted diet. c. monitor the blood pressure every 4 hours. d. relieve edema by elevating the extremities.

C R: HTN caused by Na retention is a common complication of hyperaldosteronism. Hyperaldosteronism does not cause elevation in blood glucose. pt will be hypokalemic and require potassium supplementation prior to surgery. Edema does not usually occur with hyperaldosteronism. (Cognitive Level: Application Text : pp. 1319-1320 NProcess: Implementation NCLEX: Physiological Integrity)

A 72-year-old patient is diagnosed with hypothyroidism, and levothyroxine (Synthroid) is prescribed. During initiation of thyroid replacement for the patient, it is most important for the nurse to assess a. mental status. b. nutritional level. c. cardiac function. d. fluid balance.

C R: In older patients, initiation of levothyroxine therapy can increase myocardial oxygen demand and cause angina or dysrhythmias. The medication is also expected to improve mental status and fluid balance and will increase metabolic rate and nutritional needs, but these changes do not indicate a need to change the therapy.

A patient seen at clinic for an upper respiratory infection reports receiving subcutaneous somatotropin (Genotropin) when asked by the nurse about current medications. The nurse questions the pt further about a hx of a. adrenal disease. b. untreated acromegaly. c. a pituitary tumor. d. diabetes insipidus (DI).

C R: Somatotropin is a recombinant growth hormone product used for adults with growth hormone deficiency, such as that caused by a pituitary tumor. The med is not used in adrenal disease or DI. The patient with untreated acromegaly will have an excess of growth hormone.

A patient with a possible pheochromocytoma is admitted to the hospital for evaluation and diagnostic testing. During an attack, the nurse will monitor for hypertension and a. hypoglycemia. b. bradycardia. c. headache. d. flushing.

C R: The classic CMs of pheochromocytoma are hypertension, tachycardia, severe headache, diaphoresis, and abdominal or chest pain. Elevated blood glucose may also occur due to sympathetic nervous system stimulation. Bradycardia and flushing would not be expected. (Cognitive Level: Application Text Reference: p. 1320 NProcess: Assessment NCLEX: Physiological Integrity)

While assessing a patient who has just arrived in the postanesthesia recovery unit (PACU) after a thyroidectomy, the nurse obtains these data. Which information is most important to communicate to the surgeon? a. The pt is complaining of 7/10 incisional pain. b. The pt's cardiac monitor shows a HR of 112. c. The patient has increasing swelling of the neck. d. The pat's voice is weak and hoarse sounding.

C R: The neck swelling may lead to respiratory difficulty, and rapid intervention is needed to prevent airway obstruction. The incisional pain should be treated but is not unusual after surgery. A heart rate of 112 is not unusual in a pt who has been hyperthyroid and has just arrived in the PACU from surgery. Vocal hoarseness is expected after surgery due to edema.

A diabetic patient has a new order for inhaled insulin (Exubera). Which information about the patient indicates that the nurse should contact the patient before administering the Exubera? a. The patient has a history of a recent myocardial infarction. b. The patient's blood glucose is 224 mg/dl. c. The patient uses a bronchodilator to treat emphysema. d. The patient's temperature is 101.4° F.

C Rationale: Exubera is not recommended for patients with emphysema. The other data do not indicate any contraindication to using Exubera. Cognitive Level: Application Text Reference: p. 1263 Nursing Process: Assessment NCLEX: Physiological Integrity

A diagnosis of hyperglycemic hyperosmolar nonketotic coma (HHNC) is made for a patient with type 2 diabetes who is brought to the emergency department in an unresponsive state. The nurse will anticipate the need to a. administer glargine (Lantus) insulin. b. initiate oxygen by nasal cannula. c. insert a large-bore IV catheter. d. give 50% dextrose as a bolus.

C Rationale: HHNC is initially treated with large volumes of IV fluids to correct hypovolemia. Regular insulin is administered, not a long-acting insulin. There is no indication that the patient requires oxygen. Dextrose solutions will increase the patient's blood glucose and would be contraindicated. Cognitive Level: Application Text Reference: p. 1281 Nursing Process: Planning NCLEX: Physiological Integrity

A type 1 diabetic patient who was admitted with severe hypoglycemia and treated tells the nurse, "I did not have any of the usual symptoms of hypoglycemia." Which question by the nurse will help identify a possible reason for the patient's hypoglycemic unawareness? a. "Do you use any calcium-channel blocking drugs for blood pressure?" b. "Have you observed any recent skin changes?" c. "Do you notice any bloating feeling after eating?" d. "Have you noticed any painful new ulcerations or sores on your feet?"

C Rationale: Hypoglycemic unawareness is caused by autonomic neuropathy, which would also cause delayed gastric emptying. Calcium-channel blockers are not associated with hypoglycemic unawareness, although -adrenergic blockers can prevent patients from having symptoms of hypoglycemia. Skin changes can occur with diabetes, but these are not associated with autonomic neuropathy. If the patient can feel painful areas on the feet, neuropathy has not occurred. Cognitive Level: Application Text Reference: p. 1281 Nursing Process: Assessment NCLEX: Physiological Integrity

A nursing assessment of a patient with Cushing syndrome reveals that the patient has truncal obesity and thin arms and legs. An additional manifestation of Cushing syndrome that the nurse would expect to find is a. chronically low blood pressure. b. decreased axillary and pubic hair. c. purplish red streaks on the abdomen. d. bronzed appearance of the skin.

C Rationale: Purplish-red striae on the abdomen are a common clinical manifestation of Cushing syndrome. Hypotension and bronzed-appearing skin are manifestations of Addison's disease. Decreased axillary and pubic hair occur with androgen deficiency.

A patient who has just been diagnosed with type 2 diabetes is 5 ft 4 in (160 cm) tall and weighs 182 pounds (82 kg). A nursing diagnosis of imbalanced nutrition: more than body requirements is developed. Which patient outcome is most important for this patient? a. The patient will have a diet and exercise plan that results in weight loss. b. The patient will state the reasons for eliminating simple sugars in the diet. c. The patient will have a glycosylated hemoglobin level of less than 7%. d. The patient will choose a diet that distributes calories throughout the day.

C Rationale: The complications of diabetes are related to elevated blood glucose, and the most important patient outcome is the reduction of glucose to near-normal levels. The other outcomes are also appropriate but are not as high in priority. Cognitive Level: Application Text Reference: p. 1273 Nursing Process: Planning NCLEX: Physiological Integrity

During a clinic visit 3 months following a diagnosis of type 2 diabetes, the patient reports following a reduced-calorie diet. The patient has not lost any weight and did not bring the glucose-monitoring record. The nurse will plan to obtain a(n) a. fasting blood glucose level. b. urine dipstick for glucose. c. glycosylated hemoglobin level. d. oral glucose tolerance test.

C Rationale: The glycosylated hemoglobin (Hb A1C) test shows the overall control of glucose over 90 to 120 days. A fasting blood level indicates only the glucose level at one time. Urine glucose testing is not an accurate reflection of blood glucose level and does not reflect the glucose over a prolonged time. Oral glucose tolerance testing is done to diagnose diabetes, but is not used for monitoring glucose control once diabetes has been diagnosed. Cognitive Level: Application Text Reference: pp. 1258-1259 Nursing Process: Planning NCLEX: Physiological Integrity

A patient with Cushing syndrome is admitted to the hospital to have laparoscopic adrenalectomy. During the admission assessment, the patient tells the nurse, "The worst thing about this disease is how terrible I look. I feel awful about it." best response by the nurse is a. "Let me show you how to dress so that the changes are not so noticeable." b. "I do not think you look bad. Your appearance is just altered by your disease." c. "Most of the physical and mental changes caused by the disease will gradually improve after surgery." d. "You really should not worry about how you look in the hospital. We see many worse things."

C Rationale: The most reassuring communication to the patient is that the physical and emotional changes caused by the Cushing syndrome will resolve after hormone levels return to normal postoperatively. The response beginning "Let me show you how to dress" indicates that the changes are permanent and that the patient's appearance needs disguising. The response beginning, "I do not think you look bad" does not acknowledge the patient's feelings and also fails to communicate that the changes will be resolved after surgery. And the response beginning "You really should not worry about how you look in the hospital" implies that the pt's appearance is not good.

A diabetic patient is admitted with ketoacidosis and the health care provider writes all of the following orders. Which order should the nurse implement first? a. Start an infusion of regular insulin at 50 U/hr. b. Give sodium bicarbonate 50 mEq IV push. c. Infuse 1 liter of normal saline per hour. d. Administer regular IV insulin 30 U.

C Rationale: The most urgent patient problem is the hypovolemia associated with DKA, and the priority is to infuse IV fluids. The other actions can be accomplished after the infusion of normal saline is initiated. Cognitive Level: Application Text Reference: p. 1280 Nursing Process: Implementation NCLEX: Physiological Integrity

The patient received regular insulin 10 units subcutaneously at 8:30 PM for a blood glucose level of 253 mg/dL. The nurse plans to monitor this patient for signs of hypoglycemia at which time related to the insulin's peak action? A) 8:40 PM to 9:00 PM B) 9:00 PM to 11:30 PM C) 10:30 PM to 1:30 AM D) 12:30 AM to 8:30 AM

C) 10:30 PM to 1:30 AM Regular insulin exerts peak action in 2 to 5 hours, making the patient most at risk for hypoglycemia between 10:30 PM and 1:30 AM. Rapid-acting insulin's onset is between 10-30 minutes with peak action and hypoglycemia most likely to occur between 9:00 PM and 11:30 PM. With intermediate acting insulin, hypoglycemia may occur from 12:30 AM to 8:30 AM.

A 65-year-old patient with type 2 diabetes has a urinary tract infection (UTI). The unlicensed assistive personnel (UAP) reported to the nurse that the patient's blood glucose is 642 mg/dL and the patient is hard to arouse. When the nurse assesses the urine, there are no ketones present. What collaborative care should the nurse expect for this patient? A) Routine insulin therapy and exercise B) Administer a different antibiotic for the UTI. C) Cardiac monitoring to detect potassium changes D) Administer IV fluids rapidly to correct dehydration.

C) Cardiac monitoring to detect potassium changes This patient has manifestations of hyperosmolar hyperglycemic syndrome (HHS). Cardiac monitoring will be needed because of the changes in the potassium level related to fluid and insulin therapy and the osmotic diuresis from the elevated serum glucose level. Routine insulin would not be enough, and exercise could be dangerous for this patient. Extra insulin will be needed. The type of antibiotic will not affect HHS. There will be a large amount of IV fluid administered, but it will be given slowly because this patient is older and may have cardiac or renal compromise requiring hemodynamic monitoring to avoid fluid overload during fluid replacement.

A patient, who is admitted with diabetes mellitus, has a glucose level of 380 mg/dL and a moderate level of ketones in the urine. As the nurse assesses for signs of ketoacidosis, which respiratory pattern would the nurse expect to find? A) Central apnea B) Hypoventilation C) Kussmaul respirations D) Cheyne-Stokes respirations

C) Kussmaul respirations In diabetic ketoacidosis, the lungs try to compensate for the acidosis by blowing off volatile acids and carbon dioxide. This leads to a pattern of Kussmaul respirations, which are deep and nonlabored.

The newly diagnosed patient with type 2 diabetes has been prescribed metformin (Glucophage). What should the nurse tell the patient to best explain how this medication works? A) Increases insulin production from the pancreas. B) Slows the absorption of carbohydrate in the small intestine. C) Reduces glucose production by the liver and enhances insulin sensitivity. D) Increases insulin release from the pancreas, inhibits glucagon secretion, and decreases gastric emptying.

C) Reduces glucose production by the liver and enhances insulin sensitivity. Metformin is a biguanide that reduces glucose production by the liver and enhances the tissue's insulin sensitivity. Sulfonylureas and meglitinides increase insulin production from the pancreas. α-glucosidase inhibitors slow the absorption of carbohydrate in the intestine. Glucagon-like peptide receptor agonists increase insulin synthesis and release from the pancreas, inhibit glucagon secretion, and decrease gastric emptying.

A patient is admitted to the hospital in Addisonian crisis. Which patient statement supports the nursing diagnosis of ineffective self-health management related to lack of knowledge about management of Addison's disease? A. "I double my dose of hydrocortisone on the days that I go for a run." B. "I frequently eat at restaurants, and so my food has a lot of added salt." C. "I had the stomach flu earlier this week and couldn't take the hydrocortisone." D. "I take twice as much hydrocortisone in the morning as I do in the afternoon."

C. "I had the stomach flu earlier this week and couldn't take the hydrocortisone."

What is a nursing priority in the care of a patient with a diagnosis of hypothyroidism? A. Providing a dark, low-stimulation environment B. Closely monitoring the patient's intake and output C. Patient teaching related to levothyroxine (Synthroid) D. Patient teaching related to radioactive iodine therapy

C. Patient teaching related to levothyroxine (Synthroid) A euthyroid state is most often achieved in patients with hypothyroidism by the administration of levothyroxine (Synthroid). It is not necessary to carefully monitor intake and output, and low stimulation and radioactive iodine therapy are indicated in the treatment of hyperthyroidism.

A patient has been taking oral prednisone for the past several weeks after having a severe reaction to poison ivy. The nurse has explained the procedure for gradual reduction rather than sudden cessation of the drug. What is the rationale for this approach to drug administration? A. Prevention of hypothyroidism B. Prevention of diabetes insipid us C. Prevention of adrenal insufficiency D. Prevention of cardiovascular complications

C. Prevention of adrenal insufficiency Sudden cessation of corticosteroid therapy can precipitate life-threatening adrenal insufficiency. Diabetes insipidus, hypothyroidism, and cardiovascular complications are not common consequences of suddenly stopping corticosteroid therapy.

After a hypophysectomy for acromegaly, postoperative nursing care should focus on a. frequent monitoring of serum and urine osmolarity. b. parenteral administration of a GH-receptor antagonist. c. keeping the patient in a recumbent position at all times. d. patient teaching regarding the need for lifelong hormone therapy.

Correct answer: a Rationale: A possible postoperative complication after a hypophysectomy is transient diabetes insipidus (DI). It may occur because of the loss of antidiuretic hormone (ADH), which is stored in the posterior lobe of the pituitary gland, or because of cerebral edema related to manipulation of the pituitary gland during surgery. To assess for DI, urine output and serum and urine osmolarity should be monitored closely.

To control the side effects of corticosteroid therapy, the nurse teaches the patient who is taking corticosteroids to a. increase calcium intake to 1500 mg/day. b. perform glucose monitoring for hypoglycemia. c. obtain immunizations due to high risk of infections. d. avoid abrupt position changes because of orthostatic hypotension.

Correct answer: a Rationale: Because patients often receive corticosteroid treatment for prolonged periods (more than 3 months), corticosteroid-induced osteoporosis is an important concern. Therapies to reduce the resorption of bone may include increased calcium intake, vitamin D supplementation, bisphosphonates (e.g., alendronate [Fosamax]), and institution of a low-impact exercise program.

The health care provider prescribes levothyroxine (Synthroid) for a patient with hypothyroidism. After teaching regarding this drug, the nurse determines that further instruction is needed when the patient says a. "I can expect the medication dose may need to be adjusted." b. "I only need to take this drug until my symptoms are improved." c. "I can expect to return to normal function with the use of this drug." d. "I will report any chest pain or difficulty breathing to the doctor right away."

Correct answer: b Rationale: Levothyroxine (Synthroid) is the drug of choice to treat hypothyroidism. The need for thyroid replacement therapy is usually lifelong.

The nurse teaches the patient that the best time to take corticosteroids for replacement purposes is a. once a day at bedtime. b. every other day on awakening. c. on arising and in the late afternoon. d. at consistent intervals every 6 to 8 hours.

Correct answer: c Rationale: As replacement therapy, glucocorticoids are usually administered in divided doses: two thirds in the morning and one third in the afternoon. This dosage schedule reflects normal circadian rhythm in endogenous hormone secretion and decreases the side effects associated with corticosteroid replacement therapy.

An important preoperative nursing intervention before an adrenalectomy for hyperaldosteronism is to a. monitor blood glucose levels. b. restrict fluid and sodium intake. c. administer potassium-sparing diuretics. d. advise the patient to make postural changes slowly.

Correct answer: c Rationale: Before surgery, patients should be treated with potassium-sparing diuretics (spironolactone [Aldactone], eplerenone [Inspra]) to normalize serum potassium levels. Spironolactone and eplerenone block the binding of aldosterone to the mineralocorticoid receptor in the terminal distal tubules and collecting ducts of the kidney, thus increasing sodium excretion, water excretion, and potassium retention. Oral potassium supplements may also be necessary.

A patient with a head injury develops SIADH. Manifestations the nurse would expect to find include a. hypernatremia and edema. b. muscle spasticity and hypertension. c. low urine output and hyponatremia. d. weight gain and decreased glomerular filtration rate.

Correct answer: c Rationale: Excess ADH increases the permeability of the renal distal tubule and collecting ducts, which leads to the reabsorption of water into the circulation. Consequently, extracellular fluid volume expands, plasma osmolality declines, the glomerular filtration rate increases, and sodium levels decline (i.e., dilutional hyponatremia). Hyponatremia causes muscle cramping, pain, and weakness. Initially, the patient displays thirst, dyspnea on exertion, and fatigue. Patients with the syndrome of inappropriate antidiuretic hormone secretion (SIADH) experience low urinary output and increased body weight. As the serum sodium level falls (usually to less than 120 mEq/L), manifestations become more severe and include vomiting, abdominal cramps, muscle twitching, and seizures. As plasma osmolality and serum sodium levels continue to decline, cerebral edema may occur, leading to lethargy, anorexia, confusion, headache, seizures, and coma.

After thyroid surgery, the nurse suspects damage or removal of the parathyroid glands when the patient develops a. muscle weakness and weight loss. b. hyperthermia and severe tachycardia. c. hypertension and difficulty swallowing. d. laryngospasms and tingling in the hands and feet.

Correct answer: d Rationale: Painful tonic spasms of smooth and skeletal muscles can cause laryngospasms that may compromise breathing. These spasms may be related to tetany, which occurs if the parathyroid glands are removed or damaged during surgery, which leads to hypocalcemia.

Important nursing intervention(s) when caring for a patient with Cushing syndrome include (select all that apply) a. restricting protein intake. b. monitoring blood glucose levels. c. observing for signs of hypotension. d. administering medication in equal doses. e. protecting patient from exposure to infection.

Correct answers: b, e Rationale: Hyperglycemia occurs with Cushing disease because of glucose intolerance (associated with cortisol-induced insulin resistance) and increased gluconeogenesis by the liver. High levels of corticosteroids increase susceptibility to infection and delay wound healing.

A patient who uses every-other-day prednisone therapy for rheumatoid arthritis complains of not feeling as well on the non-prednisone days and asks nurse about taking prednisone daily instead. The best response to the pt is that a. an every-other-day schedule mimics the normal pattern of cortisol secretion from the adrenal gland. b. glucocorticoids are taken on a daily basis only when theyre being used for replacement therapy. c. if it improves the symptoms, it would be acceptable to take half the usual dose every day. d. there is less effect on normal adrenal function when prednisone is taken every other day.

D R: An alternate-day regimen is given to minimize the impact of exogenous glucocorticoids on adrenal gland function. The normal pattern of cortisol secretion is diurnal. Glucocorticoids are taken daily when being used for replacement therapy, but this is not the only indication for daily use. Taking half the usual dose would not achieve the goal of minimizing adrenal gland suppression.

When teaching a patient newly diagnosed with Graves' disease about the disorder, the nurse explains that a. restriction of iodine intake is needed to reduce thyroid activity. b. exercise is contraindicated to avoid increasing metabolic rate. c. surgery will eventually be required to remove the thyroid gland. d. antithyroid medications may take several weeks to have an effect.

D R: Improvement usually begins in 1-2 wks w good results at 4-6 weeks. Large doses of iodine are used to inhibit the synthesis of thyroid hormones. Exercise using large muscle groups is encouraged to decrease irritability and hyperactivity associated with high levels of thyroid hormones. Radioactive iodine is the most common trtmt for Graves' disease, although surgery may be used.

Which information obtained when caring for a pt who has just been admitted for evaluation of DI will be of greatest concern to the nurse? a. The patient has a urine output of 800 ml/hr. b. The patient's urine specific gravity is 1.003. c. The patient had a recent head injury. d. The patient is confused and lethargic.

D R: Pts with diabetes insipidus compensate for fluid losses by drinking copious amounts of fluids, but a patient who is lethargic will be unable to drink enough fluids and will become hypovolemic. A high urine output, low urine specific gravity, and history of a recent head injury are consistent with DI, but they do not require immediate nursing action to avoid life-threatening complications.

A patient with hypoparathyroidism receives instructions from the nurse regarding symptoms of hypocalcemia and hypercalcemia. The nurse teaches the patient that if mild symptoms of hypocalcemia occur, the patient should a. increase daily fluid intake to twice usual amount b. self-administer IM calcium before calling doctor. c. call an ambulance because the symptoms will progress to seizures. d. rebreathe with a paper bag and then seek medical assistance.

D R: Rebreathing may help alleviate mild sx, but it will only temporarily increase ionized calcium level, so the pt should call HCP. There is no need to increase fluid intake. Calcium is not given IM but given slowly through IV route. Mild hypocalcemia is unlikely to progress to seizures.

A patient with primary hyperparathyroidism has a serum calcium level of 14 mg/dl (3.5 mmol/L), phosphorus of 1.7 mg/dl (0.55 mmol/L), serum creatinine of 2.2 mg/dl (194 mmol/L), and a high urine calcium. While the patient awaits surgery, the nurse should a. institute seizure precautions such as padded siderails. b. assist the patient to perform range-of-motion exercises QID. c. monitor the patient for positive Chvostek's or Trousseau's sign. d. encourage the pt to drink 4000 ml of fluid daily.

D R: The pt with hypercalcemia is at risk for kidney stones, which may be prevented by a high fluid intake. Seizure precautions and monitoring for Chvostek's or Trousseau's sign are appropriate for hypocalcemic patients. The pt should engage in weight-bearing exercise rather than range-of-motion because weight-bearing decreases calcium loss from bone.

Which class of oral glucose-lowering agents is most commonly used for people with type 2 diabetes because is reduces hepatic glucose production and enhances tissue uptake of glucose? a. insulin b. biguanide c. meglitinide d. sulfonylurea

b. biguanide

A patient recovering from DKA asks the nurse how acidosis occurs. The best response by the nurse is that a. insufficient insulin leads to cellular starvation, and as cells rupture they release organic acids into the blood. b. when an insulin deficit causes hyperglycemia, then proteins are deaminated by the liver, causing acidic by-products. c. excess glucose in the blood is metabolized by the liver into acetone, which is acidic. d. an insulin deficit promotes metabolism of fat stores, which produces large amounts of acidic ketones.

D Rationale: Ketoacidosis is caused by the breakdown of fat stores when glucose is not available for intracellular metabolism. The other responses are inaccurate. Cognitive Level: Application Text Reference: pp. 1278-1279 Nursing Process: Implementation NCLEX: Physiological Integrity

A patient with type 1 diabetes has received diet instruction as part of the treatment plan. The nurse determines a need for additional instruction when the patient says, a. "I may have an occasional alcoholic drink if I include it in my meal plan." b. "I will need a bedtime snack because I take an evening dose of NPH insulin." c. "I will eat meals as scheduled, even if I am not hungry, to prevent hypoglycemia." d. "I may eat whatever I want, as long as I use enough insulin to cover the calories."

D Rationale: Most patients with type 1 diabetes need to plan diet choices very carefully. Patients who are using intensified insulin therapy have considerable flexibility in diet choices but still should restrict dietary intake of items such as fat, protein, and alcohol. The other patient statements are correct and indicate good understanding of the diet instruction. Cognitive Level: Application Text Reference: p. 1268 Nursing Process: Evaluation NCLEX: Physiological Integrity

The nurse teaches the diabetic patient who rides a bicycle to work every day to administer morning insulin into the a. thigh. b. buttock. c. arm. d. abdomen.

D Rationale: Patients should be taught not to administer insulin into a site that will be exercised because exercise will increase the rate of absorption. The thigh, buttock, and arm are all exercised by riding a bicycle. Cognitive Level: Application Text Reference: p. 1262 Nursing Process: Implementation NCLEX: Physiological Integrity

A college student who has type 1 diabetes normally walks each evening as part of an exercise regimen. The student now plans to take a swimming class every day at 1:00 PM. The clinic nurse teaches the patient to a. delay eating the noon meal until after the swimming class. b. increase the morning dose of neutral protamine Hagedorn (NPH) insulin on days of the swimming class. c. time the morning insulin injection so that the peak occurs while swimming. d. check glucose level before, during, and after swimming.

D Rationale: The change in exercise will affect blood glucose, and the patient will need to monitor glucose carefully to determine the need for changes in diet and insulin administration. Because exercise tends to decrease blood glucose, patients are advised to eat before exercising. Increasing the morning NPH or timing the insulin to peak during exercise may lead to hypoglycemia, especially with the increased exercise. Cognitive Level: Application Text Reference: p. 1269 Nursing Process: Implementation NCLEX: Health Promotion and Maintenance

A program of weight loss and exercise is recommended for a patient with impaired fasting glucose (IFG). When teaching the patient about the reason for these lifestyle changes, the nurse will tell the patient that a. the high insulin levels associated with this syndrome damage the lining of blood vessels, leading to vascular disease. b. although the fasting plasma glucose levels do not currently indicate diabetes, the glycosylated hemoglobin will be elevated. c. the liver is producing excessive glucose, which will eventually exhaust the ability of the pancreas to produce insulin, and exercise will normalize glucose production. d. the onset of diabetes and the associated cardiovascular risks can be delayed or prevented by weight loss and exercise.

D Rationale: The patient with IFG is at risk for developing type 2 diabetes, but this risk can be decreased with lifestyle changes. Glycosylated hemoglobin levels will not be elevated in IFG and the Hb A1C test is not included in prediabetes testing. Elevated insulin levels do not cause the damage to blood vessels that can occur with IFG. The liver does not produce increased levels of glucose in IFG. Cognitive Level: Application Text Reference: p. 1255 Nursing Process: Implementation NCLEX: Physiological Integrity

A patient screened for diabetes at a clinic has a fasting plasma glucose level of 120 mg/dl (6.7 mmol/L). The nurse will plan to teach the patient about a. use of low doses of regular insulin. b. self-monitoring of blood glucose. c. oral hypoglycemic medications. d. maintenance of a healthy weight.

D Rationale: The patient's impaired fasting glucose indicates prediabetes and the patient should be counseled about lifestyle changes to prevent the development of type 2 diabetes. The patient with prediabetes does not require insulin or the oral hypoglycemics for glucose control and does not need to self-monitor blood glucose. Cognitive Level: Application Text Reference: p. 1255 Nursing Process: Planning NCLEX: Physiological Integrity

The nurse caring for a patient hospitalized with diabetes mellitus would look for which laboratory test result to obtain information on the patient's past glucose control? A) Prealbumin level B) Urine ketone level C) Fasting glucose level D) Glycosylated hemoglobin level

D) Glycosylated hemoglobin level A glycosylated hemoglobin level detects the amount of glucose that is bound to red blood cells (RBCs). When circulating glucose levels are high, glucose attaches to the RBCs and remains there for the life of the blood cell, which is approximately 120 days. Thus the test can give an indication of glycemic control over approximately 2 to 3 months. The prealbumin level is used to establish nutritional status and is unrelated to past glucose control. The urine ketone level will only show that hyperglycemia or starvation is probably currently occurring. The fasting glucose level only indicates current glucose control.

A patient with a severe pounding headache has been diagnosed with hypertension. However, the hypertension is not responding to traditional treatment. What should the nurse expect as the next step in management of this patient? A. Administration of β-blocker medications B. Abdominal palpation to search for a tumor C. Administration of potassium-sparing diuretics D. A 24-hour urine collection for fractionated metanephrines

D. A 24-hour urine collection for fractionated metanephrines Pheochromocytoma should be suspected when hypertension does not respond to traditional treatment. The 24-hour urine collection for fractionated metanephrines is simple and reliable with elevated values in 95% of people with pheochromocytoma. In a patient with pheochromocytoma preoperatively an α-adrenergic receptor blocker is used to reduce BP. Abdominal palpation is avoided to avoid a sudden release of catecholamines and severe hypertension. Potassium-sparing diuretics are not needed. Most likely they would be used for hyperaldosteronism, which is another cause of hypertension.

Following a parathyroidectomy, a patient develops tingling of the lips and a positive Trousseau's sign. Which action should the nurse take first? A. Administer the ordered muscle relaxant. B. Give the ordered oral calcium supplement. C. Start the PRN oxygen at 2 L/min per cannula. D. Have the patient rebreathe using a paper bag.

D. Have the patient rebreathe using a paper bag.

The nurse is caring for a patient admitted with suspected hyperparathyroidism. Because of the potential effects of this disease on electrolyte balance, the nurse should assess this patient for what manifestation? A. Neurologic irritability B. Declining urine output D. Lethargy and weakness C. Hyperactive bowel sounds

D. Lethargy and weakness Hyperparathyroidism can cause hypercalcemia. Signs of hypercalcemia include muscle weakness, polyuria, constipation, nausea and vomiting, lethargy, and memory impairment. Neurologic irritability, declining urine output, and hyperactive bowel sounds do not occur with hypercalcemia.

The patient with an adrenal hyperplasia is returning from surgery for an adrenalectomy. For what immediate postoperative risk should the nurse plan to monitor the patient? A. Vomiting B. Infection C. Thomboembolism D. Rapid BP changes

D. Rapid BP changes The risk of hemorrhage is increased with surgery on the adrenal glands as well as large amounts of hormones being released in the circulation, which may produce hypertension and cause fluid and electrolyte imbalances to occur for the first 24 to 48 hours after surgery. Vomiting, infection, and thromboembolism may occur postoperatively with any surgery.

A 50-year-old female patient smokes, is getting a divorce, and is reporting eye problems. On assessment of this patient, the nurse notes exophthalmos. What other abnormal assessments should the nurse expect to find in this patient? A. Puffy face, decreased sweating, and dry hair B. Muscle aches and pains and slow movements C. Decreased appetite, increased thirst, and pallor D. Systolic hypertension and increased heart rate

D. Systolic hypertension and increased heart rate The patient's manifestations point to Graves' disease or hyperthyroidism, which would also include systolic hypertension and increased heart rate and increased thirst. Puffy face, decreased sweating; dry, coarse hair; muscle aches and pains and slow movements; decreased appetite and pallor are all manifestations of hypothyroidism.

An 18-year-old male patient is undergoing a growth hormone stimulation test. The nurse should monitor the patient for A. hypothermia. Incorrect B. hypertension. C. hyperreflexia. D. hypoglycemia.

D. hypoglycemia Insulin or arginine (agent that stimulates insulin secretion) is administered for a growth hormone stimulation test. The nurse should monitor the patient closely for hypoglycemia. Hypothermia and hypertension are not expected in response to insulin or arginine. Hyperreflexia is an autonomic complication of spinal cord injury.

A patient is admitted with possible syndrome of inappropriate antidiuretic hormone (SIADH). Which information obtained by the nurse is most important to communicate rapidly to the health care provider? A. The patient complains of dyspnea with activity. B. The patient has a urine specific gravity of 1.025. C. The patient has a recent weight gain of 8 lb. D. The patient has a serum sodium level of 119 mEq/L.

D. the pt has a serum sodium level of 119 mEq/L R: A serum sodium of less than 120 mEq/L increases risk for complications such as seizures and needs rapid correction. The other data are not unusual for a pt with SIADH and do not indicate the need for rapid action.

A patient with Cushing syndrome returns to the surgical unit following an adrenalectomy. During the initial postoperative period, the nurse gives the highest priority to A. Monitoring for infection. B. Protecting the patient's skin. C. Maintaining fluid and electrolyte status. D. Preventing severe emotional disturbances.

Feedback ANS: C After adrenalectomy, the patient is at risk for circulatory instability caused by fluctuating hormone levels, and the focus of care is to assess and maintain fluid and electrolyte status through the use of IV fluids and corticosteroids. The other goals also are important for the patient but are not as immediately life-threatening as the circulatory collapse that can occur with fluid and electrolyte disturbances.

The client diagnosed with Cushing's disease has developed 1 peripheral edema. Theclient has received intravenous fluids at 100 mL/hr via IV pump for the past 79 hours. The client received IVPB medication in 50 mL of fluid every 6 hours for 15 doses. How many mL of fluid did the client receive? ________

The client has received 8,650 mL of intravenous fluid.

Which assessment finding for a patient who takes levothyroxine (Synthroid) to treat hypothyroidism indicates that the nurse should contact the health care provider before administering the medication? A. Increased thyroxine (T4) level B. Blood pressure 102/62 mm Hg C. Distant and difficult to hear heart sounds D. Elevated thyroid stimulating hormone level

Increased thyroxine (T4) level Feedback An increased thyroxine level indicates the levothyroxine dose needs to be decreased. The other data are consistent with hypothyroidism and the nurse should administer the Synthroid.

What is the priority action for the nurse to take if the patient with type 2 diabetes complains of blurred vision and irritability? a. Call the physician. b. Administer insulin as ordered. c. Check the patients blood glucose level. d. Assess for other neurologic symptoms.

c. Check the patients blood glucose level.

A patient is treated with demeclocycline (Declomycin) to control the symptoms of syndrome of inappropriate antidiuretic hormone (SIADH). The nurse determines that the demeclocycline is effective upon finding that the A. Peripheral edema is decreased. B. Patient's weight has increased. C. Urine specific gravity is increased. D. Patient's urinary output is increased.

Patient's urinary output is increased. Feedback Demeclocycline blocks the action of ADH on the renal tubules and increases urine output. An increase in weight or an increase in urine specific gravity indicates that the SIADH is not corrected. Peripheral edema does not occur with SIADH. A sudden weight gain without edema is a common clinical manifestation of this disorder.

Which assessment finding for a 24-year-old patient admitted with Graves' disease requires the most rapid intervention by the nurse? A. BP 166/100 mm Hg B. Bilateral exophthalmos C. Heart rate 136 beats/minute D. Temperature 104.8° F (40.4°

Temperature 104.8° F (40.4° Feedback ANS: D The patient's temperature indicates that the patient may have thyrotoxic crisis and that interventions to lower the temperature are needed immediately. The other findings also require intervention but do not indicate potentially life-threatening complications

A patient with hyperthyroidism is treated with radioactive iodine (RAI) at a clinic. Before the patient is discharged, the nurse instructs the patient A. That symptoms of hyperthyroidism should be relieved in about a week. B. That symptoms of hypothyroidism may occur as the RAI therapy takes effect. C. To discontinue the antithyroid medications taken before the radioactive therapy. D. About radioactive precautions to take with urine, stool, and other body secretions.

That symptoms of hypothyroidism may occur as the RAI therapy takes effect. Feedback There is a high incidence of postradiation hypothyroidism after RAI, and the patient should be monitored for symptoms of hypothyroidism. RAI has a delayed response, with the maximum effect not seen for 2 to 3 months, and the patient will continue to take antithyroid medications during this time. The therapeutic dose of radioactive iodine is low enough that no radiation safety precautions are needed.

Which are appropriate therapies for patients with diabetes mellitus (select all that apply)? a. Use of statins to treat dyslipidemia b. Use of diuretics to treat nephropathy c. Use of ACE inhibitors to treat nephropathy d. Use of laser photocoagulation to treat retinopathy

a. Use of statins to treat dylipidemia c. Use of ACE inhibitors to treat nephropathy d. Use of laser photocoagulation to treat retinopathy

When caring for the patient with a traumatic brain injury (TBI), the nurse knows that damage to which endocrine gland can affect the hormone secretion from some of the other endocrine glands? ANTERIOR PITUITARY ADRENAL HYPOTHALAMUS PARATHYROID

With a TBI, the anterior pituitary is likely to be damaged. The anterior pituitary gland secrets tropic hormones that control the secretion of hormones by other endocrine glands (the thyroid, adrenal cortex, and reproductive organs). The parathyroids secrete parathyroid hormone that regulates serum calcium level by acting on bone, the kidneys, and indirectly the gastrointestinal tract. The pineal gland secretes melatonin that helps regulate circadian rhythm and reproduction. The thyroid glands secrete thyroxine (T4), triiodothyronine (T3) that regulates the cell processes of cell growth and tissue differentiation, and calcitonin that affects bone tissue to regulate serum calcium and phosphorus levels.

Which characteristics describe the use of RAI (select all that apply)? a. Often causes hypothyroidism over time b. Decreases release of thyroid hormones c. Blocks peripheral conversion of T4 to T3 d. Treatment of choice in nonpregnant adults e. Decreases thyroid secretion by damaging thyroid gland f. Often used with iodine to produce euthyroid before surgery

a, d, e. RAI causes hypothyroidism over time by damaging thyroid tissue and is the treatment of choice for nonpregnant adults. Potassium iodide decreases the release of thyroidhormones and decreases the size of the thyroid gland preoperatively. Propylthiouracil (PTU) blocks peripheral conversion of T4 to T3 and may be used with iodine to produce a euthyroid state before surgery.

Which statement by the patient with type 2 diabetes is accurate? a. "I am supposed to have a meal or snack if I drink alcohol." b. "I am not allowed to eat any sweets because of my diabetes." c. "I do not need to watch what I eat because my diabetes is not the bad kind." d. "The amount of fat in my diet is not important; it is just the carbohydrates that raise my blood sugar."

a. "I am supposed to have a meal or snack if I drink alcohol."

what characterizes type 2 diabetes? select all that apply a. B cell exhaustion b. insulin resistance c. genetic predisposition d. altered production of adipokines e. inherited defect in insulin receptors f. inappropraite glucose production by the liver

a. B cell exhaustion b. insulin resistance c. genetic predisposition d. altered production of adipokines e. inherited defect in insulin receptors f. inappropraite glucose production by the liver

Antidiabetic drugs are designed to control signs and symptoms of diabetes mellitus. The nurse primarily expects a decrease in which? a. Blood glucose b. Fat metabolism c. Glycogen storage d. Protein mobilization

a. Blood glucose

Which statement accurately describes Graves' disease? a. Exophthalmos occurs in Graves' disease. b. It is an uncommon form of hyperthyroidism. c. Manifestations of hyperthyroidism occur from tissue desensitization to the sympathetic nervous system. d. Diagnostic testing in the patient with Graves' disease will reveal an increased thyroid-stimulating hormone (TSH) level.

a. Exophthalmos or protrusion of the eyeballs may occur in Graves' disease from increased fat deposits and fluid in the orbital tissues and ocular muscles, forcing the eyeballs outward. Graves' disease is the most common form of hyperthyroidism. Increased metabolic rate and sensitivity of the sympathetic nervous system lead to the clinical manifestations. Thyroid-stimulating hormone (TSH) level is decreased in Graves' disease.

A nurse who is teaching a client how to recognize symptoms of hypoglycemia should include which symptoms in the teaching? (Select all that apply.) a. Headache b. Nervousness c. Bradycardia d. Sweating e. Thirst f. Sweet breath odor

a. Headache b. Nervousness d. Sweating

During discharge teaching of a patient with Addisons disease, which statement by the patient indicates that the nurse needs to do additional teaching? a. I should always call the doctor if I develop vomiting or diarrhea b. if my weight goes down, my dosage of steroid is probably too high c. i should double or trip my steroid dose if i undergo rigorous physical activity d. i need to carry an emergency kit with injectable hydrocortisone in case i cant take my medication by mouth

a. I should always call the doctor if I develop vomiting or diarrhea

You are caring for a patient with newly diagnosed type 1 diabetes. What information is essential to include in your patient teaching before discharge from the hospital (select all that apply)? a. Insulin administration b. Elimination of sugar from diet c. Need to reduce physical activity d. Hypoglycemia prevention, symptoms, and treatment

a. Insulin administration d. Hypoglycemia prevention, symptoms, and treatment

When caring for a patient with primary hyperaldosteronism, the nurse would question a physician's order for the use of a. Lasix b. amiloride (midamor) c. spironolactone (aldactone) d. aminoglutethimide (cytadren)

a. Lasix (R- hyperaldosteronism is an excess of aldosterone, which is manifested by sodium and water retention and potassium excretion. Lasix is a potassium-wasting diuretic that would increase the potassium deficiency. Aminoglutethimide blocks aldosterone synthesis; amiloride is apotassium-sparing diuretic; and spironolactone blocks mineralocorticoid receptors in the kidney, increasing secretion of sodium and water and retention of potassium.)

A 72-year-old woman is diagnosed with diabetes. The nurse recognizes that management of diabetes in the older adult a. does not require as tight glucose control as in younger diabetics. b. is usually not treated unless the patient becomes severely hyperglycemic. c. does not include treatment with insulin because of limited dexterity and vision. d. usually requires that a younger family member be responsible for care of the patient.

a. Rationale: Because the clinical manifestations of long-term complications of diabetes take 10 to 20 years to develop, and because tight glucose control in the older patient is associated with an increase frequency of hypoglycemia, the goals for glycemic control are not as rigid as in the younger population. Treatment is indicated, and insulin may be used if the patient does not respond to oral agents. The patient's needs, rather than age, determine the responsibility of other in care.

When caring for a patient with metabolic syndrome, what should the nurse give the highest priority to teach the patient about? a. achieving a normal weight b. performing daily aerobic exercise c. eliminating red meat from the diet d. monitoring blood glucose periodically

a. achieving a normal weight

The client diagnosed with type 1 diabetes is found lying unconscious on the floor of the bathroom. Which intervention should the nurse implement first? a. administer 50% dextrose IVP b. notify the HCP c. move the client to the ICU d. check the serum glucose level

a. administer 50% dextrose IVP dextrose is only given if the client is found unconscious and the nurse suspect hypoglycemia. This will arouse the patient immediately

Two days following a self-managed hypoglycemic episode at home, the patient tells the nurse that his blood glucose levels since the episode have been between 80 and 90mg/dL. The best response by the nurse is, a. "That is a good range for your glucose levels." b. "You should call your health care provider because you need to have your insulin increased." c. "That level is too low in view of your recent hypoglycemia, and you should increase your food intake." d. "You should only take half your insulin dosage for the next few days to get your glucose level back to normal."

a. Rationale: Blood glucose levels of 80 to 90mg/dL (4.5-5 mmol/L) are within the normal range and are desired in the patient with diabetes, even following a recent hypoglycemic episode. Hypoglycemia is often caused by a single event, such as skipping a meal or taking too much insulin or vigorous exercise; once corrected, normal control should be maintained.

A patient with diabetes calls the clinic because she is experiencing nausea and flulike symptoms. The nurse advises the patient to a. administer the usual insulin dosage. b. hold fluid intake until the nausea subsides. c. come to the clinic immediately for evaluation and treatment. d. monitor the blood glucose every 1 to 2 hours and call if the glucose rises over 150 mg/dL (8.3 mmol/L)

a. Rationale: During minor illnesses, the patient with diabetes should continue drug therapy and food intake. Insulin is important because counter regulatory hormones may raise blood glucose during the stress of illness, and food or a carbohydrate liquid substitution is important because during illness the body requires extra energy to deal with the stress of the illness. Blood glucose monitoring should be done every 4 hour, and the health care provider should be notified if the level is >240mg/dL (13.9 mmol/L) or if fever, ketonuria, or nausea and vomiting occur.

The major advantage of using an insulin pump is that a. tight glycemic control can be maintained. b. errors in insulin dosing are less likely to happen. c. complications of insulin therapy are prevented. d. frequent blood glucose monitoring is unnecessary.

a. Rationale: Insulin pumps provide tight glycemic control by continuous subcutaneous insulin infusion based on the patient's basal profile, with bolus doses at mealtime at the patient's discretion. Errors in insulin dosing and complications of insulin therapy are still potential risks with insulin pumps.

When caring for a patient with metabolic syndrome, the nurse gives the highest priority to teaching the patient about a. maintaining a normal weight. b. performing daily aerobic exercise. c. eliminating red mean from the diet. d. monitoring the blood glucose periodically.

a. Rationale: Metabolic syndrome is a cluster of abnormalities that include elevated insulin levels, elevated triglycerides and low-density lipoproteins (LDL), and decreased high-density lipoproteins (HDL). These abnormalities greatly increase the risk for cardiovascular disease associated with diabetes that can be prevented or delayed with weight loss. Exercise is also important, but normal weight is most important.

In nutritional management of all types of diabetes, it is important for the patient to a. eat regular meals at regular times. b. restrict calories to promote moderate weight loss. c. eliminate sucrose and other simple sugars from the diet. d. limit saturated fat intake to 30% of dietary intake.

a. Rationale: The body requires food at regularly spaced intervals throughout the day, and omission or delay of meals can result in hypoglycemia, especially for the patient taking insulin or oral hypoglycemic agents. Weight loss may be recommended in type 2 diabetes if the individual is overweight, but many patients with type 1 diabetes are thin and require an increase in caloric intake. Fewer than 7% of total calories should be from saturated fats, and simple sugars should be limited, but moderate amounts can used if counted as a part of total carbohydrate intake.

A diabetic patient is learning to mix regular insulin and NPH insulin in the same syringe. The nurse determines that additional teaching is needed when the patient a. withdraws the NPH dose in the syringe first b. injects air equal to the NPH dose into the NPH vial first. c. removes any air bubbles after withdrawing the first insulin. d. adds air equal to the insulin dose into the regular vial and withdraws the dose.

a. Rationale: When mixing regular with a longer-acting insulin, regular insulin should always be drawn in the syringe first to prevent contamination of the regular insulin vial with longer-acting insulin additives. Air is added in the neutral protamine Hagedorn (NPH) vial; then air is added to the regular vial, and the regular insulin is withdrawn, bubbles are removed, and the dose of NPH is withdrawn.

Which information should be included in health teaching for clients taking insulin? (Select all that apply.) a. Recognize signs of hypoglycemic reaction. b. Adhere to the prescribed diet. c. Take insulin as prescribed. d. Monitor blood glucose level. e. Be sure to exercise. f. Keep appointments with health care provider. g. Alter insulin dose based on how you're feeling.

a. Recognize signs of hypoglycemic reaction. b. Adhere to the prescribed diet. c. Take insulin as prescribed. d. Monitor blood glucose level.

A patient is admitted to the hospital with a diagnosis of Cushing syndrome. On physical assessment of the patient, what should the nurse expect to find? a. Hypertension, peripheral edema, and petechiae b. Weight loss, buffalo hump, and moon face with acne c. Abdominal and buttock striae, truncal obesity, and hypotension d. Anorexia, signs of dehydration, and hyperpigmentation of the skin

a. The effects of adrenocortical hormone excess, especially glucocorticoid excess, include weight gain from accumulation and redistribution of adipose tissue, sodium and water retention, glucose intolerance, protein wasting,loss of bone structure, loss of collagen, and capillary fragility leading to petechiae. Clinical manifestations of adrenocortical hormone deficiency include hypotension, dehydration, weight loss, and hyperpigmentation of the skin.

What preoperative instruction should the nurse give to the patient scheduled for a subtotal thyroidectomy? a. How to support the head with the hands when turning in bed b. Coughing should be avoided to prevent pressure on the incision c. Head and neck will need to remain immobile until the incision heals d. Any tingling around the lips or in the fingers after surgery is expected and temporary

a. To prevent strain on the suture line postoperatively, the patient's head must be manually supported while turning and moving in bed but range-of-motion exercises for the head and neck are also taught preoperatively to be gradually implemented after surgery. There is no contraindication for coughing and deep breathing and these should be carried out postoperatively. Tingling around the lips or fingers is a sign of hypocalcemia, which may occur if the parathyroid glands are inadvertently removed during surgery. This sign should be reported immediately.

A patient with diabetes calls the clinic because she is experiencing nausea and flu like symptoms. Which advice from the nurse will be the best for this patient? a. administer the usual insulin dosage b. hold fluid intake until the nausea subsides c. come to the clinic immediately for evaluation and treatment d. monitor the blood glucose everyone one to two hours and call if it rises over 150 mg/dL (8.3 mmol/L)

a. administer the usual insulin dosage

the home health nurse is completing the admission assessment for a 76 year old client diagnosed with type 2 diabetes controlled with 70/30 insulin. Which intervention should be included in the plan of care a. assess the clients ability to read small print b. monitor the clients serum pt level c. teach the client how to perform a hemoglobin A1C test daily d. instruct the client to check the feet weekly

a. assess the clients ability to read small print

The client is diagnosed with hypothyroidism. Which s/s should the nurse expect the client to exhibit? a. complaints of extreme fatigue and hair loss b. exophthalmos and complaints of nervousness c. complaints of profused sweating and flushed skin d. tetany and complaints of stiffness of the hands

a. complaints of extreme fatigue and hair loss

The nurse identifies the client problem "risk for imbalance body temp" for a client diagnosed with hypothyroidism. which intervention should be included in the plan of care? a. discourage the use of an electric blanket b. assess the clients temp q2h c. keep the room temp cool d. space activities to promote rest

a. discourage the use of an electric blanket

individualized nutrition therapy for patients using conventional, fixed insulin regimens should include teaching the patient to a. eat regular meals at regular times b. restrict calories to promote moderate weight loss c. eliminate sucrose and other simple sugars from the diet d. limit saturated fat intake to 30% of dietary calorie intake

a. eat regular meals at regular times

The nurses adminstered 28 units of Humulin N, an intermediate-acting insulin, to a client diagnosed with type 1 diabetes at 1600. Which intervention should the nurse implement? a. ensure the client eats the bedtime snack b. determine how much food the client ate at lunch c. perform a glucometer reading at 0700 d. offer the client protein after administering insulin

a. ensure the client eats the bedtime snack

A patient with acromegaly is treated w a transphenoidal hypophysectomy. Postoperatively, the nurse a. ensures that any clear nasal drainage is tested for glucose b. maintains the patient flat in bed to prevent cerebrospinal fluid leak c. assists the patient with toothbrushing Q4H to keep the surgical area clean d. encourages deep breathing and coughing to prevent respiratory complications

a. ensures any clear nasal drainage is tested for glucose (R- a transphenoidal hypophysectomy involves entry into the sella turcica through an incision in the upper lip and gingiva into the floor of the nose and the sphenoid sinuses. Postoperative clear nasal drainage with glucose content indicates CSF leakage from an open connection to the brain, putting the pt at risk for meningitis. After surgery, the pt is positioned with head elevated to avoid pressure on the sella turcica, coughing and straining are avoided to prevent increased ICP and CSF leakage, and although mouth care is required Q4H toothbrushing should not be performed for 7-10post sx.)

the 68 year old client diagnosed with hyperthyroidism is being treated with radioactive iodine therapy. Which intervention should the nurse discuss with the client? a. explain it will take up to a month for symptoms of hyperthyroidism to subside b. teach the iodine therapy will have to be tapered slowing over one week c. discuss the client will have to be hospitalized during the radioactive therapy d. inform the client after therapy the client will not have to take any meds

a. explain it will take up to a month for symptoms of hyperthyroidism to subside

Which clinical manifestations may be seen in a client experiencing a hypoglycemic (insulin) reaction? (Select all that apply.) a. headache b. nervousness c. tremor d. excessive perspiration e. tachycardia f. abdominal pain

a. headache b. nervousness c. tremor d. excessive perspiration e. tachycardia

the patient with diabetes has a blood glucose level of 248 mg/dL. Which manifestations in the patient would the nurse understand as being related to this blood glucose level? (select all that apply) a. headache b. unsteady gait c. abdominal cramps d. emotional changes e. increase in urination f. weakness and fatigue

a. headache c. abdominal cramps e. increase in urination f. weakness and fatigue

Preoperative instructions for the patient scheduled for a subtotal thyroidectomy includes teaching the patient a. how to support the head w hands when moving b. that coughing should due avoided to prevent pressure on the incision c. that the head and neck will need to remain immobile until the incision heals d. that any tingling around the lips or in the fingers after surgery is expected and temporary

a. how to support the head with the hands when moving (R- to prevent strain on suture line postoperatively, head must be manually supported while turning and moving in bed, but range-of-motion exercise for the head and neck are also taught preoperatively to be gradually implemented after surgery. There is no contraindication for coughing and deep breathing, and they should be carrier out postoperatively. Tingling around the lips or fingers is a sign of hypocalcemia, which may occur if the parathyroid glands are inadvertently removed during surgery, and should be reported immediately.)

The nurse is performing discharge teaching for a client diagnosed with cushings disease. Which statement by the client demonstrates an understanding of the instructions? a. i will be sure to notify my HCP if i start to run a fever b. before i stop taking the prednisone, i will be taught how to taper it off c. if i get weak and shaky, i need to eat some hard candy or drink some juice d. it is fine if i continue to participate in weekend games of tackle football

a. i will be sure to notify my HCP if i start to run a fever

The elderly client is admitted to the intensive care department diagnosed with severe HHNS. which collaborative intervention should the nurse include in the plan of care? a. infuse 0.9% normal saline IV b. administer intermediate-acting insulin c. perform a blood glucometer cheeks daily d. monitor arterial blood gas results

a. infuse 0.9% normal saline IV

A patient is admitted to the hospital with thyrotoxicosis. On physical assessment of the patient, what should the nurse expect to find? a. hoarsness and laryngeal stridor b. bulging eyeball and dysrhythmias c. elevated temp and signs of HF d. lethargy progressing suddenly to impairment of consciouness

c. elevated temp and signs of HF

A 72 year old woman is diagnosed with diabetes. What does the nurse recognize about the management of diabetes in the older adult? a. it is more difficult to achieve strict glucose control than in younger patients b. it usually is not treated unless the patient becomes severely hyperglycemic c. it does not include treatment with insulin because of limited dexterity and vision d. it usually requires that a younger family member be responsible for care of the patient

a. it is more difficult to achieve strict glucose control than in younger patients

The nurse is teaching the patient with prediabetes ways to prevent or delay the development of type two diabetes. What information should be included (select all that apply). a. maintain a healthy weight b. exercise for 60 minutes each day c. have BP checked regularly d. assess for visual changes on monthly basis e. monitor for polyuria, polyphagia, and polydipsia

a. maintain a healthy weight e. monitor for polyuria, polyphagia, and polydipsia

the client is admitted to the ICU diagnosed with DKA. Which intervention should the nurse implement? select all that apply a. maintain adequate ventilation b. assess fluid volume status c. administer IV potassium d. check for urinary ketones e. monitor intake and output

a. maintain adequate ventilation b. assess fluid volume status c. administer IV potassium d. check for urinary ketones e. monitor intake and output

During care of a patient with syndrome of inappropriate ADH (SIADH), the nurse should a. monitor neurologic status Q2H or more often if needed b. keep the HOB elevated to prevent ADH release c. teach the patient receiving treatment with diuretics to restrict sodium intake d. notify the physician if the p's blood pressure decreases more than 20mmHg from baseline

a. monitor neurologic status Q2H or more often if needed R- the pt with SIADH has marked dilution hyponatremia and should be monitored for decreased neurologic function and convulsions every 2 hours. ADH release is reduced by keeping the HOB flat to increase left atrial filling pressure, and sodium intake is supplemented because of hyponatremia and sodium loss caused by diuretics. A reduction in BP indicates a reduction in total fluid vo and is an expected outcome of treatment.)

the nurse is teaching the client diagnosed with hyperthyroidism. which information should be taught to the client? select all that apply a. notify the HCP if a 3 pound weight loss occurs in 2 weeks b. discuss ways to cope with the emotional liability c. notify the HCP if taking OTC meds d. carry a medical information card or bracelet e. teach how to take thyroid meds correctly

a. notify the HCP if a 3 pound weight loss occurs in 2 weeks b. discuss ways to cope with the emotional liability c. notify the HCP if taking OTC meds d. carry a medical information card or bracelet

What are the major symptoms that characterize diabetes? (Select all that apply.) a. polyuria b. polyphagia c. polyposia d. polydipsia e. polyrrhea

a. polyuria b. polyphagia d. polydipsia

Which drugs may cause hyperglycemia? (Select all that apply.) a. prednisone b. epinephrine c. levothyroxine d. hydrochlorothiazide

a. prednisone b. epinephrine d. hydrocholorothiazide

The client diagnosed with addisons disease is admitted to the ED after a day at the lake. The client is lethargic, forgetful, and weak. Which intervention should the nurse implement? a. start an IV with an 18 gauge needle and infuse NS rapidly b. have the client wait in the waiting room until a bed is available c. obtain a permit for the client to receive a blood transfusion d. collect urinalysis and blood samples for a CBC and calcium level

a. start an IV with an 18 gauge needle and infuse NS rapidly

The patient with type 2 diabetes is being put on acarbose (precose) and wants to know why she is taking it. What should the nurse include in this patients teaching (select all that apply) a. take it with the first bite of each meal b. it is not used in patients with HF c. endogenous glucose production is decreased d. effectiveness is measured by 2 hour postprandial glucose e. it delays glucose absorption from the GI tract

a. take it with the first bite of each meal d. effectiveness is measured by 2 hour postprandial glucose e. it delays glucose absorption from the GI tract

the diabetic educator is teach a class on diabetes type 1 and is discussing sick-day rules. Which intervention should the diabetes educator include in the discussion? select all that apply a. take the diabetic medication even if unable to eat the clients normal diabetic diet b. if unable to eat, drink liquids equal to the clients normal caloric diet c. if is not necessary to notify the HCP if ketones are in the urine d. test blood glucose levels and test urine ketones once a day and keep a record e. call the HCP if glucose levels are higher than 180 mg/dL

a. take the diabetic medication even if unable to eat the clients normal diabetic diet b. if unable to eat, drink liquids equal to the clients normal caloric diet e. call the HCP if glucose levels are higher than 180 mg/dL

The nurse is developing a care plan for the client diagnosed with type 1 diabetes. The nurse identifies the problem "high risk for hyperglycemia related to noncompliance with the medication regimen." What statement is an appropriate short-term goal for the client a. the client will have a blood glucose level between 90 and 140 mg/dL b. the client will demonstrate appropriate insulin injection technique c. the nurse will monitor the clients blood glucose levels 4 time a day d. the client will maintain normal kidney function with 30 ml/hr urine output

a. the client will have a blood glucose level between 90 and 140 mg/dL

The nurse determines that the pt in acute adrenal insufficiency is responding favorably to trtmt when a. the patient appears alert and oriented b. the patient's urinary output has increased c. pulmonary edema is reduced as evidenced by clear lung sounds d. laboratory tests reveal serum elevations of K and glucose and a decrease in sodium

a. the patient appears alert and oriented (R- confusion, irritability, disorientation, or depressioni s often present in pt with Addison's dz, and a (+) response to therapy would be indicated by a return to alertness and orientation. Other indication of response to therapy would be a decreased urinary output, decreased serum potassium, and increased serum sodium and glucose. The pt with Addison's would be very dehydrated and volume-depleted and would not have pulmonary edema.)

What are manifestations of DKA (select all that apply) a. thirst b. ketonuria c. dehydration d. metabolic acidosis e. Kussmaul respirations f. sweet, fruity breath odor

a. thirst b. ketonuria c. dehydration d. metabolic acidosis e. Kussmaul respirations f. sweet, fruity breath odor

when teaching the patient with Type 1 diabetes, what should the nurse emphasize as the major disadvantage of using an insulin pump? a. tight glycemic control can be maintained b. errors in insulin dosing are less likely to occur c. complications of insulin therapy are prevented d. frequent blood glucose monitoring is unnecessary

a. tight glycemic control can be maintained

A patient with diabetes is learning to mix regular insulin and NPH insulin in the same syringe. The nurse determines that additional teaching is needed when the patient does what? a. withdraws the NPH dose into the syringe first b. infects air equal to the NPH dose into the NPH vial first c. removes any air bubbles after withdrawing the first insulin d. adds air equal to the insulin dose into the regular vial and withdraws the dose

a. withdraws the NPH dose into the syringe first

During routine health screening, a patient is found to have a fasting plasma glucose (FPG) of 132 mg/dL (7.33. mmol/L). At a follow-up visit, a diagnostic of diabetes would be made based on (select all that apply) a. glucosuria of 3+. b. an A1C of 7.5%. c. a FPG of ≥126mg/dL (6.9 mmol/L). d. random blood glucose of 126 mg/dL (7.0 mmol/L). e. a 2-hour oral glucose tolerance test (OGTT) of 190 mg/dL (10.5 mmol/L).

b,c. Rationale: The patient has one prior test result that meets criteria for a diagnosis of diabetes, but on a subsequent day must again have results from on of the three tests that meet the criteria for diabetes diagnosis. These criteria include a fasting plasma glucose level of ≥126mg/dL (7.0 mmol/L), or A1C ≥6.5% or a 2 hour OGTT level ≥200mg/dL (11.1 mmol/L). Both the fasting plasma glucose (FPG) and A1C would confirm a diagnosis of diabetes in this patient.

Analyze the following diagnostic findings for your patient with type 2 diabetes. Which result will need further assessment? a. BP 126/80 mm Hg b. A1C 9% c. FBG 130mg/dL (7.2 mmol/L) d. LDL cholesterol 100 mg/dL (2.6 mmol/L)

b. A1C 9%

The nurse has identified the nursing diagnosis of fatigue for a patient who is hypothyroid. What should the nurse do while caring for this patient? a. Monitor for changes in orientation, cognition, and behavior. b. Monitor for vital signs and cardiac rhythm response to activity. c. Monitor bowel movement frequency, consistency, shape, volume, and color. d. Assist in developing well-balanced meal plans consistent with level of energy expenditure.

b. Cardiorespiratory response to activity is important to monitor in this patient to determine the effect of activities and plan activity increases. Monitoring changes in orientation, cognition, and behavior are interventions for impaired memory. Monitoring bowels is needed to plan care for the patient with constipation. Assisting with meal planning will help the patient with imbalanced nutrition: more than body requirements to lose weight if needed.

A client is newly diagnosed with type 1 diabetes mellitus and requires daily insulin injections. Which instruction should the nurse include in the teaching of insulin administration? a. Teach the family members to administer glucagon by injection if the client has a hyperglycemic reaction. b. Instruct the client about the necessity for compliance with prescribed insulin therapy. c. Teach the client that hypoglycemic reactions more likely to occur at the onset of action time. d. Instruct the client in the care of insulin container and syringe handling.

b. Instruct the client about the necessity for compliance with prescribed insulin therapy.

A patient with type 1 diabetes uses 20U of 70/30 neutral protamine Hagedorn (NPH/regular) in the morning and at 6:00pm. When teaching the patient about this regimen, the nurse stresses that a. hypoglycemia is most likely to occur before the noon meal. b. a set meal pattern with a bedtime snack is necessary to prevent hypoglycemia. c. flexibility in food intake is possible because insulin is available 24 hours/day. d. pre-meal glucose checks are required to determine needed changes in daily dosing.

b. Rationale: A split-mixed dose of insulin requires that the patient adhere to a set meal pattern to provide glucose for the action of the insulins, and a bedtime snack is usually required when patients take a long-acting insulin late in the day to prevent nocturnal hypoglycemia. Hypoglycemia is most likely to occur with this dose late in the afternoon and during the night. When premixed formulas are used, flexible dosing based on glucose levels is not recommended.

The following interventions are planned for a diabetic patient. Which intervention can the nurse delegate to nursing assistive personnel (NAP)? a. Discuss complications of diabetes. b. Check that the bath water is not too hot. c. Check the patient's technique for drawing up insulin. d. Teach the patient to use the glucometer for in-home glucose monitoring.

b. Rationale: Checking the temperature of the bath water is part of assisting with activities of daily living (ADLs) and within the scope of care for the nursing assistive personnel (NAP). Discussion of complications, teaching, and assessing learning are appropriate for RNs.

To prevent hyperglycemia or hypoglycemia with exercise, the nurse teaches the patient using glucose-lowering agents that exercise should be undertaken a. only after a 10- to 15-g carbohydrate snack is eaten. b. about 1 hour after a eating, when blood glucose levels are rising. c. when glucose monitoring reveals that the blood glucose is in the normal range. d. when blood glucose levels are high because exercise always has a hypoglycemic effect.

b. Rationale: During exercise, a diabetic person needs both adequate glucose to prevent exercise-induced hypoglycemia and adequate insulin because conterregulatory hormones are produced during the stress of exercise and may cause hyperglycemia. Exercise after meals is best, but a 10- to 15-g carbohydrate snack may be taken if exercise is performed before meals or is prolonged. Blood glucose levels should be monitored before, during, and after exercise to determine the effect of exercise on the levels.

Goals of nutritional therapy for the patient with type 2 diabetes include maintenance of a. ideal body weight. b. normal serum glucose and lipid levels. c. a special diet using diabetic foods. d. five small meals per day with a bedtime snack.

b. Rationale: Maintenance of as near-normal blood glucose levels as possible and a achievement of optimal serum lipid levels with dietary modification are believed to be the most important factors in preventing both short- and long-term complications of diabetes. There is no specific "diabetic diet," and use of dietetic foods is not necessary for diabetes control. Most diabetics eat three meals a day, and some require a bedtime snack for control of nighttime hypoglycemia. A reasonable weight, which may or may not be an ideal body weight, is also a goal of nutritional therapy.

A nurse working in an outpatient clinic plans a screening program for diabetes. Recommendations for screening would include a. OGTT fro all minority populations every year. b. FPG for all individuals at age 45 and then every 3 years. c. testing all people under the age of 21 for islet cell antibodies. d. testing for type 2 diabetes only in overweight or obese individuals.

b. Rationale: The American Diabetes Association recommends that testing for type 2 diabetes with a FPG should be considered for all individuals at the age of 45 and above and, if normal, repeated every 3 years. Testing for immune markers of type 1 diabetes is not recommended. Testing of a younger age or more frequently should be done for members of a high-risk ethnic population, including African Americans, Hispanics, Native Americans, Asians Americans, and Pacific Islanders.

In diabets, atherosclerotic disease affecting the cerebrovascular, cardiovascular, and peripheral vascular systems a. can be prevent by tight glucose control. b. occurs with a higher frequency and earlier onset than in the nondiabetic population. c. is cause by the hyperinsulinemia related to insulin resistance common in type 2 diabetes. d. cannot be modified by reduction of risk factors such as smoking, obesity, and high fat intake.

b. Rationale: The development of atherosclerotic vessel disease seems to be promoted by the altered lipid metabolism common to diabetes, and although tight glucose control may help delay the process, it does not prevent it completely. Atherosclerosis in diabetic patients does respond somewhat to a reduction in general risk factors, as it does in nondiabetics, and reduction in fat intake, control of hypertension, abstention from smoking, maintenance of normal weight, and regular exercise should be carried out by diabetic patients.

When the client is prescribed glipizide (Glucotrol), the nurse knows that which side effects/adverse effects may be expected? (Select all that apply.) a. Tachypnea b. Tachycardia c. Increase alertness d. Increased weight gain e. Visual disturbances f. Hunger

b. Tachycardia e. Visual disturbances f. Hunger

an 18 year old female client, 5'4" tall, weighting 113 kg, comes to the clinic for a nonhealing wound on her lower leg, which she has had for two weeks. Which disease process should the nurse suspect the client has developed? a. Type 1 diabetes b. Type 2 diabetes c. gestational diabetes d. acanthosis nigricans

b. Type 2 diabetes

The nurse at a freestanding health care clinic is caring for a 56 year old male client who is homeless and is a type 2 diabetic controlled with insulin. Which action is an example of client advocacy? a. ask the client if he has somewhere he can go and live b. arrange for someone to give him insulin at a local homeless shelter c. notify adult protective services about the clients situation d. ask the HCP to take the client off insulin because he is homeless

b. arrange for someone to give him insulin at a local homeless shelter

what disorders and diseases are related to macrovascular complications of diabetes? select all that apply a. chronic kidney disease b. coronary artery disease c. microaneurysms and destruction or retinal vessels d. ulceration and amputation of the lower extremities e. capillary and arteriole membrane thickening specific to diabetes

b. coronary artery disease d. ulceration and amputation of the lower extremities

The pt with diabetes has been diagnosed with autnomic neurupathy. What problems should the nurse expect to find in the patient (select all that apply) a. painless foot ulcers b. erectile dysfunction c. burning foot pain at night d. loss of fine motor control e. vomiting undigested food f. painless myocardial infarction

b. erectile dysfunction e. vomiting undigested food f. painless myocardial infarction

When the patient with parathyroid disease experiences symptoms of hypocalcemia, a measure that can be used to temporarily raise serum calcium levels is to a. administer IV normal saline b. have the patient rebreathe in a paper bag c. administer Lasix as ordered d. administer oral phosphorous supplements

b. have the patient rebreathe in a paper bag (R- rebreathing in a paper bag promotes CO2 retention in blood, which lowers pH and creates an acidosis. An academia enhances solubility and ionization of calcium, increasing the proportion of total body Ca available in physiologically active form and relieving the sx of hypocalcemia. Saline promotes calcium excretion, as does Lasix. Phosphate levels in blood are reciprocal to calcium and an increase in phosphate promotes calcium excretion.)

the client with type 2 diabetes controlled with biguanide oral diabetic medication is scheduled for a CT scan with contrast of the abdomen to evaluate pancreatic function. Which intervention should the nurse implement? a. provide a high-fat diet 24 hour prior to test b. hold the biguanide med for 48 hours prior to test c. obtain an informed consent form for the test d. administer pancreatic enzymes prior to the test

b. hold the biguanide med for 48 hours prior to test

An appropriate nursing intervention for the patient with hyperparathyroidism is to a. pad side rails as a seizure precaution b. increase fluid intake to 3000 to 4000ml/day c. maintain bed rest to prevent pathologic frxturs d. monitor the patient for Trousseau's phenomenon or Chvostek's sign

b. increase fluid intake to 3000 to 4000ml/day (R-high fluid intake is indicated in hyperparathyroidism to dilute hypercalcemia and flush the kidneys so that calcium stone formation is reduced.)

A patient with SIADH is treated with water restriction and administration of IV fluids. The nurses evaluates that treatment has been effective when the patient experiences a. increased urine output, decreased serum sodium, and increased urine specific gravity b. increased urine output, increased serum sodium, and decreased urine specific gravity c. decreased urine output, increased serum sodium, and decreased urine specific gravity d. decreased urine output, decreased serum sodium, and increased urine specific gravity

b. increased urine output, increased serum sodium, and decreased urine specific gravity (rationale- the patient with SIADH has water retention with hyponatremia, decreased urine output and concentrated urine with high specific gravity. improvement in the patient's condition reflected by increased urine output, normalization of serum sodium, and more water in the urine, decreasing the specific gravity.)

which statement best describes atherosclerotic disease affected the cerebrovascular, cardiovascular, and peripheral vascular systems in patients with diabetes? a. it can be prevented by tight glucose control b. it occurs with a higher frequency and earlier onset than in the nondiabetic population c. it is caused by the hyperinsulinemia related to insulin resistance common in type 2 diabetes d. it cannot be modified by reduction of risk factors sch as smoking, obesity, and high fat intake

b. it occurs with a higher frequency and earlier onset than in the nondiabetic population

A client is to receive insulin before breakfast, and the time of breakfast tray delivery is variable. The nurse knows that which insulin should not be administered until the breakfast tray has arrived and the client is ready to eat? a. Humulin N b. lispro (Humalog) c. glargine (Lantus) d. Humulin R

b. lispro (Humalog)

The most important nursing intervention during the medical and surgical treatment of the patient with a pheochromocytoma is a. administering IV fluids b. monitoring blood pressure c. monitoring I&O and daily weights d. administering B-adrenergic blocking agents

b. monitoring blood pressure (R- a pheochromocytoma is a catecholamine-producing tumor of the adrenal medulla, which may cause severe, episodic HTN; severe, pounding headache; and profuse sweating. Monitoring for dangerously high BP before surgery is critical, as is monitoring for BP fluctuation during medical and surgical tx.)

What should the goals of nutrition therapy for the patient with type two diabetes include? a. ideal body weight b. normal serum glucose and lipid levels c. a special diabetic diet using dietetic foods d. five small meals per day with a bedtime snack

b. normal serum glucose and lipid levels

The UAP on the medical floor tells the nurse the client diagnosed with DKA wants something else to eat for lunch. Which intervention should the nurse implement? a. instruct the UAP to get the client additional food b. notify the dietician about the clients request c. request the HCP increase the clients caloric intake d. tell the UAP the client cannot have anything else

b. notify the dietician about the clients request

In type 1 diabetes there is an osmotic effect of glucose when insulin deficiency prevents the use of glucose for energy. Which classic symptom is caused by the osmotic effect of glucose? a. fatigue b. polydipsia c. polyphagia d. recurrent infections

b. polydipsia

Which electrolyte replacement should the nurse anticipate being ordered by the HCP in the client diagnosed with DKA who has been admitted to the ICU? a. glucose b. potassium c. calcium d. sodium

b. potassium

A patient with hypothyroidism is treated h Synthroid. When teaching the pt about the therapy, the nurse a. explains that caloric intake must be reduced when drug therapy is started b. provides written instruction for all information related to the medication therapy c. assures the patient that a return to normal function will occur with replacement therapy d. informs the patient that medications must be taken until hormone balance is reestablished

b. provides written instruction for all information related to the medication therapy (R- bc of mental sluggishness, inattentiveness, and memory loss that occur with hypothyroidism, it is important to provide written instructions and repeat information when teaching pt. Caloric intake can be increased when drug therapy is started, because of an increased metabolic rate, and replacement therapy must be taken for life. Although most pts return to a normal state w treatment, cardiovascular conditions and psychoses may persist.)

why are the hormones cortisol, glucagon, epinephrine, and growth hormone referred to as counter regulatory hormones? a. decrease glucose production b. stimulate glucose output by the liver c. increase glucose transport into the cells d. independently regulate glucose level in the blood

b. stimulate glucose output by the liver

The client has developed cushings disease. What statement is the scientific rationale for the development of this diagnosis? a. the client has an autoimmune problem causing the destruction of the adrenal cortex b. the client has been taking steroid meds for an extended period for another diseases process c. the client has a pituitary gland tumor causing the adrenal glands to produce too much cortisol d. the client has developed an adrenal gland problem for which the HCP does not have an explanation

b. the client has been taking steroid meds for an extended period for another diseases process

Which assessment data indicate the client diagnosed with DKA is responding to the medical treatment? a. the client has tended skin turgor and dry mucous membranes b. the client is alert and oriented times three c. the clients ABG results are pH 7.29, PaCO2 44, HCO3 15 d. the clients serum potassium level is 3.3 mEq/L

b. the client is alert and oriented times three

When caring for a patient with nephrogenic diabetes insipidus, the nurse would expect treatment to include a. fluid restriction b. thiazide diuretics c. a high sodium diet d. chlorprpamide (Diabinese)

b. thiazide diuretics Rationale- in nephrogenic Di the kidney is unable to respond to ADH, so vasopressin or hormone analogs are not effective.

Polydipsia and polyuria related to diabetes mellitus are primarily due to a.the release of ketones from cells during fat metabolism. b.fluid shifts resulting from the osmotic effect of hyperglycemia. c.damage to the kidneys from exposure to high levels of glucose. d.changes in RBCs resulting from attachment of excessive glucose to hemoglobin.

b.fluid shifts resulting from the osmotic effect of hyperglycemia.

A client is receiving a daily dose of Humulin N insulin at 7:30 am. The nurse expects the peak effect of this drug to occur at which time? a. 8:15 am b. 10:30 am c. 5:00 pm d. 11:00 pm

c. 5:00 pm

What describes the primary difference in treatment of DKA and hyperosmolar hyperglycemia syndrome (HHS) a. DKA requires administration of bicarbonate to correct acidosis b. potassium replacement is not necessary in management of HHS c. HHS requires greater fluid replacement to correct the dehydration d. administration of glucose is withheld in HHS until the blood glucose reaches a normal level

c. HHS requires greater fluid replacement to correct the dehydration

A client is diagnosed with type 2 diabetes mellitus. The nurse is aware that which statement is true? a. Client is most likely a teenager. b. Client is most likely a child younger than 10 years. c. Heredity is a major causative factor. d. Viral infections contribute most to disease development.

c. Heredity is a major causative factor.

A patient with Addison's disease comes to the emergency department with complaints of N/V/D, and fever. The nurse would expect collaborative care to include a. parenteral injections of ACTH b. IV administration of vasopressors c. IV administration of hydrocortisone d. IV administration of D5W with 20mEq of KCl

c. IV administration of hydrocortisone (R- vom and dia are early indicators of addisonian crisis and fever indicates an infection, which s causing additional stress for pt. trtmt of crisis requires immediate glucocorticoid replacement, and IV hydrocortisone, fluids, Na and glucose are necessary for 24hours. Addison's disease is a primary insufficiency of adrenal gland, and ACTH is not effective, nor would vasopressors be effective w fluid deficiency of Addison's. Potassium levels are incd in Addison's dz, and KCl would be contraindicated.)

The nurse assesses the diabetic patient's technique of self-monitoring of blood glucose (SMBG) 3 months after initial instruction. An error in the performance of SMBG noted by the nurse that requires intervention is a. doing the SMBG before and after exercising. b. puncturing the finger on the side of the finger pad. c. cleaning the puncture site with alcohol before the puncture. d. holding the hand down for a few minutes before the puncture.

c. Rationale: Cleaning the puncture site with alcohol is not necessary and may interfere with test results and lead to drying and splitting of the fingertips. Washing the hands with warm water is adequate cleaning and promotes blood flow to the fingers. Blood flow is also increase by holding the hand down. Puncture on the side of the finger pad are less painful. Self-monitored blood glucose (SMBG) should be performed before and after exercise.

The nurse determines that a patient with a 2-hour OGTT of 152mg/dL has a. diabetes. b. impaired fasting glucose. c. impaired glucose tolerance. d. elevated glycosylated hemoglobin (Hb)

c. Rationale: Impaired glucose tolerance exists when a 2-hour plasma glucose level is higher than normal but lower than the level diagnostic for diabetes (i.e., 140-199mg/dL). Impaired fasting glucose exists when fasting glucose levels are greater than the normal of 100mg/dL but less than the 126mg/dL diagnostic of diabetes. Both conditions represent a condition known as pre diabetes.

The nurse is assessing a newly admitted diabetic patient. Which of these observations should be addressed as a priority by the nurse? a. Bilateral numbness of both hands b. Stage II pressure ulcer on the right heel c. Rapid respirations with deep inspiration d. Areas of lumps and dents on the abdomen.

c. Rationale: Rapid deep respirations are symptoms of diabetic ketoacidosis (DKA). Stage II ulcers and bilateral numbness are chronic complications of diabetes. The lumps and dents on the abdomen indicate a need to teach the patient about site rotation.

The treatment for DKA and HHS differs primarily in that a. DKA requires administration of bicarbonate to correct acidosis. b. potassium replacement is not necessary in management of HHS. c. HHS require greater fluid replacement to correct the dehydration. d. administration of glucose is withheld in HHS until the blood glucose reaches a normal level.

c. Rationale: The management of DKA is similar to that of HHS except HHS requires greater fluid replacement because of the severe hyperosmolar state. Bicarbonate is not usually given in DKA to correct acidosis unless the pH is <7.0 because administration of insulin will reverse the abnormal fat metabolism. Total body potassium deficit is high in both conditions, requiring potassium administration, and, in both conditions, glucose is added to IV fluids when blood glucose levels fall to 250mg/dL (13.9 mmol/L).

A patient taking insulin has recorded fasting glucose levels above 200mg/dL (11.1 mmol/L) on awakening for the last five mornings. The nurse advises the patient to a. increase the evening insulin dose to prevent the dawn phenomenon. b. use a single dose insulin regimen with an intermediate-acting insulin. c. monitor the glucose level at bedtime, between 2:00 and 4:00 am, and on arising. d. decrease the evening insulin dosage to prevent night hypoglycemia and the Somogyi effect.

c. Rationale: The patient's elevated glucose on arising may be the result of either dawn phenomenon or Somogyi effect, and the best way to determine whether the patient needs more or less insulin is by monitoring the glucose at bedtime, between 2:00 and 4:00 am, and on arising. If predawn levels are below 60mg/dL, the insulin dose should be reduced, but if the 2:00 to 4:00 am blood glucose is high, the insulin should be increased.

When teaching the patient with diabetes about insulin administration, the nurse instructs the patient to a. pull back on the plunger after inserting the needle to check for blood. b. clean the skin at the injection site with an alcohol swab before each injection. c. consistently use the same size of the appropriate strength insulin syringe to avoid dosing errors. d. rotate injection sites from arms to thighs to abdomen with each injection to prevent lipodystrophies.

c. Rationale: U100 insulin must be used with a U100 syringe, but for those using low doses of insulin, syringes are available that have increments of 1 unit instead of 2 units. Errors can be made in dosing if patients switch back and forth between different sizes of syringes. Aspiration before injection of the insulin is not recommended, nor is the use of alcohol to clean the skin. Because the rate of peak serum concentrations varies with the site selected for injection, injections should be rotated within a particular area, such as the abdomen.

A patient is admitted to the hospital in thyrotoxic crisis. On physical assessment of the patient, the nurse would expect to find a. hoarseness and laryngeal stridor b. bulging eyeballs and arrhythmias c. elevated temperature and signs of heart failure d. lethargy progressing suddenly to impairment of consciousness

c. elevated temperature and signs of heart failure (R- a hyperthyroid crisis results in marked manifs of hyperthyroidism, w fever tachycardia, heart failure, shock, hyperthermia, agitation, N/V/D, delirium, and coma. Although exophthalmos may be present in pt w Gravs', it is not a signif factor in hyperthyroid crisis. Hoarsness and laryngeal stridor are ch of tetany of hypoparathyroidsm, and lethargy progressing to coma is characteristic of myxedema coma, a complication of hypothyroidism.

Ketoacidosis occurs as a complication of diabetes when a. illnesses causing nausea and vomiting lead to bicarbonate loss with body fluids. b. the glucose level becomes so high that osmotic diuresis promotes fluid and electrolyte loss. c. an insulin deficit causes the body to metabolize large amounts of fatty acids rather than glucose for energy. d. the patient skips meals after taking insulin, leading to rapid metabolism of glucose and breakdown of fats for energy.

c. Rationale: When insulin is insufficient and glucose cannot be used for cellular energy, the body releasees and breaks down stored fats and protein to meet energy needs. Free fatty acids from stored triglycerides are released and metabolized in the liver in such large quantities that ketones are formed. Ketones are acidic and alter the pH of the blood, causing acidosis. Osmotic diuresis occurs as a result of loss of both glucose and ketones in the urine.

A patient with hypoparathyroidism resulting from surgical treatment of hyperparathyroidism is preparing for discharge. What should the nurse teach the patient? a. Milk and milk products should be increased in the diet. b. Parenteral replacement of parathyroid hormone will be required for life. c. Calcium supplements with vitamin D can effectively maintain calcium balance. d. Bran and whole-grain foods should be used to prevent GI effects of replacement therapy.

c. The hypocalcemia that results from PTH deficiency is controlled with calcium and vitamin D supplementation and possibly oral phosphate binders. Replacement with PTH is not used because of antibody formation to PTH, the need for parenteral administration, and cost. Milk products, although good sources of calcium, also have high levels of phosphate, which reduce calcium absorption. Whole grains and foods containing oxalic acid also impair calcium absorption.

When instructing a patient regarding a urine study for free cortisol, what is most important for the nurse to tell the patient? a. Save the first voided urine in the morning. b. Maintain a high-sodium diet 3 days before collection. c. Try to avoid stressful situations during the collection period. d. Complete at least 30 minutes of exercise before collecting the urine sample.

c. Try to avoid stressful situations during the collection period. A urine study for free cortisol requires a 24-hour urine collection. The patient should be instructed to avoid stressful situations and excessive physical exercise that could unduly increase cortisol levels. The patient should also maintain a low-sodium diet before and during the urine collection period.

Which patient should the nurse plan to teach how to prevent or delay the development of diabetes? a. an obese 50 year old hispanic woman b. a child whose father has type 1 diabetes c. a 34 year old woman whose parents both have type 2 diabetes d. a 12 year old boy whose father has maturity onset diabetes of the young (MODY)

c. a 34 year old woman whose parents both have type 2 diabetes

A patient seen at the clinic for an upper respiratory infection reports receiving subcutaneous somatotropin (Genotropin) when asked by the nurse about his current medications. The nurse questions the patient further about a history of 1. adrenal disease. 2. a pituitary tumor. 3. diabetes insipidus. 4. untreated acromegaly

c. a pituitary tumor.

A patient with DI is treated with nasal desmopression. The nurse recognize that the drug is not having an adequate therapeutic effect the the patient experiences a. headache and weight gain b. nasal irritation and nausea c. a urine specific gravity of 1.002 d. an oral intake greater than urinary output

c. a urine specific gravity of 1.002 (rationale- normal urine specific gravity is 1.003 to 1.030, and urine with a specific gravity of 1.002 is very dilute, indicating that there continues to be excessive loss of water and that treatment of DI is inadequate. H/A, weight gain, and oral intake greater the urinary output are signs of volume excess that occur with overmedication. Nasal irritation & nausea may also indicate overmedication.)

What tissues require insulin to enable movement of glucose into the tissue cells (select all that apply) a. liver b. brain c. adipose d. blood cells e. skeletal muscles

c. adipose e. skeletal muscles

A patient who is on corticosteroid therapy for treatment of an autoimmune disorder has the following additional drugs ordered. Which on is used to prevent corticosteroid-induced osteoporosis a. potassium b. fursemide (Lasix) c. alendronate (Foxamax) d. pantoprazole (Protonix)

c. alendronate (Foxamax)

The nurse should observe the patient for symptoms of ketoacidosis when a. illness causing nausea and vomiting lead to bicarbonate loss with body fluids b. glucose levels become so high that osmotic diuresis promotes fluid and electrolyte loss c. an insulin deficit causes the body to metabolize large amounts of fatty acids rather than glucose for energy d. the patient skips meals after taking insulin, leading to rapid metabolism of glucose and breakdown of fats for energy

c. an insulin deficit causes the body to metabolize large amounts of fatty acids rather than glucose for energy

When providing discharge instructions to a patient following a subtotal thyroidectomy, the nurse advises the patient to a. never miss a daily dose of thyroid replacement therapy b. avoid regular exercise until thyroid function is normalized c. avoid eating foods such as soybeans, turnips, and rutabagas d. use warm salt water gargles several times a day to relieve throat pain

c. avoid eating foods such as soybeans, turnips, and rutabagas Rationale- the patient should avoid goitrogens, foods that inhibit thyroid, such as soybeans, turnips, rutabagas, and peanut skins.

the nurse is admitting a client diagnosed with primary adrenal cortex insuffeciency (Addisons disease). Which clinical manifestations should the nurse expect to assess? a. moon face, buffalo hump, and hyperglycemia b. hirsutism, fever, and irritability c. bronze pigmentation, hypotension, and anorexia d. tachycardia, bulging eyes, and goiter

c. bronze pigmentation, hypotension, and anorexia

To prevent complications in the patient with Cushing syndrome, the nurse monitors the patient for a. hypotension b. hypoglycemia c. cardiac arrhythmias d. decreased cardiac output

c. cardiac arrhythmias rationale- electrolyte changes that occur in Cushing syndrome include sodium retention and potassium excretion by the kidney, resulting in hypokalemia

The client diagnosed with type 2 diabetes is admitted to the intensive care unit with hyperosmolar hyperglycemia nonketonic syndrome (HHNS) coma. Which assessment data should the nurse expect the client to exhibit? a. Kussmauls respiration b. kiarrhea and epigastric pain c. dry mucous membranes d. ketones breath odor

c. dry mucous membranes

The client with cushings disease has undergone a unilateral adrenalectomy. Which discharge instructions should the nurse discuss with the client? a. instruct the client to take the glucocorticoid and mineralocorticoid meds as prescribed b. teach the client regarding sexual functioning and androgen replacement therapy c. explain the s/s of infection and when to call the HCP d. demonstrate turn, cough, and deep breathing exercises the client should perform q2h

c. explain the s/s of infection and when to call the HCP

The client received 10 units of Humulin R, a fast actign insulin, at 0700. At 1030 the UAP tells the nurse the client has a headache and is really acting "funny." Which intervention should the nurse implement first? a. Instruct the UAP to obtain the blood glucose level b. Have the client drink 8 oz of orange juice c. go to the clients room and assess the client for hypoglycemia d. prepare to administer 1 ampule 50% dextrose IV

c. go to the clients room and assess the client for hypoglycemia

The emergency department nurse is caring for a client diagnosed with HHNS who has a blood glucose of 680 mg.dL. Which question should the nurse ask the client to determine the cause of this acute complication a. when is the last time you took your insulin b. when did you have your last meal c. have you had some type of infection lately d. how long have you had diabetes

c. have you had some type of infection lately

The patient with newly diagnosed diabetes is displaying shakiness, confusion, irritability, and slurred speech. What should the nurse suspect is happening a. DKA b. HHS c. hypoglycemia d. hyperglycemia

c. hypoglycemia

A patient is scheduled for bilateral adrenalectomy. During the postoperative period, the nurse would expect administration of corticosteroids to be a. reduced to promote wound healing b. withheld until sx of hypocortisolism appear c. increased to promote an adequate response to the stress of surgery d. reduced bc excessive hormones are released during surgical manipulation of the glands

c. increased to promote an adequate response to the stress of surgery (R- although pt with Cushing syndrome has excess corticosteroids, removal of the glands and the stress of surgery require that high doses of cortisone be administered postoperatively for several days. The nurse should monitor the pt postoperatively to detect whether large amounts of hormones were released during surgical manipulation and to ensure the healing is satisfactory.)

A patient with mild iatrogenic cushing syndrome is on an alternate-day regimen of corticosteroid therapy. What does the nurse explain to the patient about this regimen? a. it maintains normal adrenal hormone balance b. it prevents ACTH release from the pituitary gland c. it minimizes hypothalmic pituitary adrenal supression d. it provides a more effective therapeutic effect of the drug

c. it minimizes hypothalmic pituitary adrenal supression

A diabetic patient has a serum glucose level of 824 mg/dL (45.7 mmol/L) and is unresponsive. Following assessment of the patient, the nurse suspects diabetic ketoacidosis rather than hyperosmolar hyperglycemic syndrome based on the findings of a. polyuria. b. severe dehydration. c. rapid, deep respirations. d. decreased serum potassium.

c. rapid, deep respirations.

Which med order should the nurse question in the client diagnosed with untreated hypothyroidism a. thyroid hormones b. oxygen c. sedatives d. laxatives

c. sedatives

the nurse is discussing ways to prevent DKA with the client diagnosed with type 1 diabetes. Which instruction is the most import to discuss with the client? a. refer the client to the American Diabetes Association b. do not take an OTC meds c. take the prescribed insulin even when unable to eat because of illness d. explain the need to get the annual flu and pneumonia vaccines

c. take the prescribed insulin even when unable to eat because of illness

The nurse is assessing the feet of a client with long term type 2 diabetes. Which assessment data warrant immediate intervention by the nurse? a. the client has crumbling toe nails b. the client has athletes foot c. the client has a necrotic big toe d.the client has thickened toenails

c. the client has a necrotic big toe

The charge nurse is making client assignments in the ICU. Which client should be assigned to the most experienced nurse? a. the client with type 2 diabetes who has a blood glucose level of 348 mg/dL b. the client diagnosed with type 1 diabetes who is experiencing hypoglycemia c. the client with DKA who has multifocal premature ventricular contractions d. the client with HHNS who has a plasma osmolarity of 290 mOsm/L

c. the client with DKA who has multifocal premature ventricular contractions

The nurse is preparing to administer the following meds. Which med should she question a. the thyroid hormone to the client who does not have a T3 or T4 level b. the regular insulin to the client with a blood glucose level of 210 mg/dL c. the loop diuretic with a potassium level of 3.3 mEq/L d. the cardiac glycoside to the client who has a digoxin level of 1.4 mg/dL

c. the loop diuretic with a potassium level of 3.3 mEq/L

The client diagnosed with Type 1 diabetes has a glycosylated hemoglobin (A1C) of 8.1%. which interpretation should the nurse make based on this result? a. this result is below normal levels b. this result is within acceptable levels c. this result is above recommended levels d. this result is dangerously high

c. this result is above recommended levels

During assessment of the pt with acromegaly, the nurse would expect the patient to report a. infertility b. dry, irritated skin c. undesirable changes in appearance d. an increase in height of 2 to 3 inches per year

c. undesirable changes in appearance (R- the increased production of GH in acromegaly causes an increase in thickness and width of bones and enlargement of soft tissues, resulting in marked changes in facial features, oily and coarse skin, and speech difficulties. Height is not increased in adults w GH excess bc the epiphyses of the bones are closed, and infertility is not a common finding because growth hormone is usually the only pituitary hormone involved in acromegaly.)

The nurse writes a problem of "altered body image" for a 34 year old client diagnosed with cushings disease. Which intervention should be implemented? a. monitor blood glucose levels prior to meals and at bedtime b. perform a head to toe assessment on the client every shift c. use therapeutic communication to allow the client to discus feelings d. assess bowel sounds and temp q4h

c. use therapeutic communication to allow the client to discus feelings

When replacement therapy is started for a patient with long-standing hypothyroidism, what is most important for the nurse to monitor the patient for? a. Insomnia b. Weight loss c. Nervousness d. Dysrhythmias

d. All these manifestations may occur with treatment of hypothyroidism. However, as a result of the effects of hypothyroidism on the cardiovascular system, when thyroid replacement therapy is started myocardial oxygen consumption is increased and the resultant oxygen demand may cause angina, cardiac dysrhythmias, and heart failure, so monitoring for dysrhythmias is most important.

What is a cause of primary hypothyroidism in adults? a. Malignant or benign thyroid nodules b. Surgical removal or failure of the pituitary gland c. Surgical removal or radiation of the thyroid gland d. Autoimmune-induced atrophy of the thyroid gland

d. Both Graves' disease and Hashimoto's thyroiditis are autoimmune disorders that eventually destroy the thyroid gland, leading to primary hypothyroidism. Thyroid tumors most often result in hyperthyroidism. Secondary hypothyroidism occurs as a result of pituitary failure and iatrogenic hypothyroidism results from thyroidectomy or radiation of the thyroid gland.

A patient with Graves' disease asks the nurse what caused the disorder. What is the best response by the nurse? a. "The cause of Graves' disease is not known, although it is thought to be genetic." b. "It is usually associated with goiter formation from an iodine deficiency over a long period of time." c. "Antibodies develop against thyroid tissue and destroy it, causing a deficiency of thyroid hormones." d. "In genetically susceptible persons, antibodies are formed that cause excessive thyroid hormone secretion."

d. In Graves' disease, antibodies to the TSH receptor are formed, attach to the receptors, and stimulate the thyroid gland to release triiodothyronine (T3), thyroxine (T4), or both, creating hyperthyroidism. The disease is not directly genetic but individuals appear to have a genetic susceptibility to develop autoimmune antibodies. Goiter formation from insufficient iodine intake is usually associated with hypothyroidism.

What characteristic is related to Hashimoto's thyroiditis? a. Enlarged thyroid gland b. Viral-induced hyperthyroidism c. Bacterial or fungal infection of thyroid gland d. Chronic autoimmune thyroiditis with antibody destruction of thyroid tissue

d. In Hashimoto's thyroiditis, thyroid tissue is destroyed by autoimmune antibodies. An enlarged thyroid gland is a goiter. Viral-induced hyperthyroidism is subacute granulomatous thyroiditis. Acute thyroiditis is caused by bacterial or fungal infection.

Following the teaching of foot care to a diabetic patient, the nurse determines that additional instruction is needed when the patient says, a. "I should wash my feet daily with soap and warm water." b. "I should always wear shoes to protect my feet from injury." c. "If my feet are cold, I should wear socks instead of using a heating pad." d. "I'll know if I have sores or lesions on my feet because they will be painful."

d. Rationale: Complete or partial loss of sensitivity of the feet is common with peripheral neuropathy of diabetes, and diabetics may suffer foot injury and ulceration without ever having pain. Feet must be inspected during daily care for any cuts, blisters, swelling, or reddened areas.

A diabetic patient is found unconscious at home, and a family member calls the clinic. After determining that no glucometer is available, the nurse advise the family member to a. try to around the patient to drink some orange juice. b administer 10 U or regular insulin subcutaneously. c. call for an ambulance to transport the patient to a medical facility. d. administer glucagon 1mg intramuscularly (IM) or subcutaneously.

d. Rationale: If a diabetic patient is unconscious, immediate treatment for hypoglycemia must be given to prevent brain damage, and IM or subcutaneous administration of 1mg of glucagon should be done. If that unconsciousness has another cause, such as ketosis, the rise in glucose cause by the glucagon is not as dangerous as the low glucose level. Following administration of the glucagon, the patient should be transported to a medical facility for further treatment and evaluation. Insulin is contraindicated without knowledge of the patient's glucose level, and oral carbohydrate cannot be given when patients are unconscious.

The home care nurse should intervene to correct a patient whose insulin administration includes a. warming a pre filled refrigerated syringe in the hands before administration. b. storing syringes pre filled with NPH and regular insulin needle-up in the refrigerator. c. placing the insulin bottle currently in use in a small container on the bathroom countertop. d. mixing an evening dose of regular insulin with insulin glargine in one syringe for administration.

d. Rationale: Insulin glargine (Lantus), a long-acting insulin that is continuously released with no peak of action, cannot be diluted or mixed with any other insulin or solution. Mixed insulins should be stored needle-up in the refrigerator and warmed before administration. Currently used bottles of insulin can be kept at room temperature.

In addition to promoting the transport of glucose from the blood into the cell, insulin also a. enhances the breakdown of adipose tissue for energy. b. stimulates hepatic glycogenolysis and gluconeogenesis. c. prevents the transport of triglycerides into adipose tissue. d. accelerates the transport of amino acids into cells and their synthesis into protein.

d. Rationale: Insulin is an anabolic hormone, responsible for growth, repair, and storage, and it facilitates movement of amino acids into cells, synthesis of protein, storage of glucose as glycogen, and deposition of triglycerides and lipids as fat into adipose tissue. Glucagon is responsible for hepatic glycogenolysis and gluconeogenesis, and fat is used for energy when glucose levels are depleted.

Lispro insulin (Humalog) with NPH insulin is ordered for a patient with newly diagnosed type 1 diabetes. The nurse knows that when lispro insulin is used, it should be administered a. only once a day b. 1 hour before meals c. 30-45 minutes before meals d. at mealtime or within 15 minutes of meals

d. Rationale: Lispro is a rapid-acting insulin that has an onset of action of 5 to 15 minutes and should be injected at the time of the meal to within 15 minutes of eating. Regular insulin is short acting with an onset of action in 30 to 60 minutes following administration and should be given 30 to 45 minutes before meals.

Which of the following patients would a nurse plan to teach how to prevent or delay the development of diabetes? a. A 62 year-old obese white woman. b. An obese 50 year-old Hispanic woman. c. A child whose father has type 1 diabetes. d. A 34 year-old woman whose parents both have type 2 diabetes.

d. Rationale: Type 2 diabetes has a strong genetic influence, and offspring of parents who both have type 2 diabetes have an increased chance of developing it. Whereas type 1 diabetes is associated with genetic susceptibility related to human leukocyte antigens (HLAs), offspring of parents who both have type 1 diabetes have only a 6% to 10% chance of developing the disease. Lower risk factors for type 2 diabetes include obesity; being a Native American, Hispanic, or African-American; and being 55 years or older.

What is the only type of insulin that may be administered IV? a. NPH b. Detemir c. Lantus d. Regular

d. Regular

When comparing the pathophysiology of type 1 and type 2 diabetes, which statement would be correct for a patient with type 2 diabetes who was admitted to the hospital with pneumonia? a. The patient must receive insulin therapy to prevent the development of ketoacidosis. b. The patient has islet cell antibodies that have destroyed the ability of the pancreas to produce insulin. c. The patient has minimal or absent endogenous insulin secretion and requires daily insulin injections. d. The patient may have sufficient endogenous insulin secretion to prevent ketosis but is at risk for development of hyperosmolar hyperglycemic syndrome.

d. The patient may have sufficient endogenous insulin secretion to prevent ketosis but is at risk for development of hyperosmolar hyperglycemic syndrome.

A patient's recent medical history is indicative of diabetes insipidus. The nurse would perform patient teaching related to which diagnostic test? a. Thyroid scan b. Fasting glucose test c. Oral glucose tolerance d. Water deprivation test

d. Water deprivation test A water deprivation test is used to diagnose the polyuria that accompanies diabetes insipidus. Glucose tests and thyroid tests are not directly related to the diagnosis of diabetes insipidus.

When providing discharge instructions to a patient who had a subtotal thyroidectomy for hyperthyroidism, what should the nurse teach the patient? a. Never miss a daily dose of thyroid replacement therapy. b. Avoid regular exercise until thyroid function is normalized. c. Use warm saltwater gargles several times a day to relieve throat pain. d. Substantially reduce caloric intake compared to what was eaten before surgery.

d. With the decrease in thyroid hormone postoperatively, calories need to be reduced substantially to prevent weight gain. When a patient has had a subtotal thyroidectomy, thyroid replacement therapy is not given because exogenous hormone inhibits pituitary production of TSH and delays or prevents the restoration of thyroid tissue regeneration. Regular exercise stimulates the thyroid gland and is encouraged. Saltwater gargles are used for dryness and irritation of the mouth and throat following radioactive iodine therapy.

A patient suspected of having acromegaly has an elevated plasma growth hormone level. In acromegaly, the nurse would also expect the pt's diagnostic results to include a. hyperinsulinemia b. a plasma glucose of less than 70 c. decreased growth hormone levels with an oral glucose challenge test d. a serum sometomedin C (insulin-like growth-factor) of more than 300

d. a serum somatomedin C (Insulin-like-growth-factor) of more than 300 (R- a normal response to growth hormone secretion is stimulation of the liver to produce somatomedin C which stimulates growth of bones and soft tissue. The increased levels of somatomedin C normally inhibit growth hormone, but in acromegaly the pituitary gland secretes GH despite elevated somatomedin C levels.)

In addition to promoting the transport of glucose from the blood into the cell, what does insulin do? a. enhances the breakdown of adipose tissue for energy b. stimulates hepatic glycogenolysis and gluconeogenesis c. prevents the transport of triglycerides into adipose tissue d. accelerates the transport of amino acids into cells and their synthesis into protein

d. accelerates the transport of amino acids into cells and their synthesis into protein

Causes of primary hypothyroidism in adults include a. malignant or benign thyroid nodules b. surgical removal or failure of the pituitary gland c. surgical removal or radiation of thyroid gland d. autoimmune-induced atrophy of the gland

d. autoimmune-induced atrophy of the gland (R- both Graves disease and Hasimotos thyroiditis are autoimmune disorders that eventually destroy thyroid gland, leading to primary hypothyroidism. Thyroid tumors most often result in hyperthyroidism. Secondary hypothyroidism occurs as a result of pituitary failure, and iatrogenic hypothyroidism results from thyroidectomy or radiation of the thyroid gland.)

what is a cause of primary hypothyroidism in adults? a. malignant or benign thyroid modules b. surgical removal or failure of the pituitary gland c. surgical removal or radiation of the thyroid gland d. autoimmune-induced atrophy of the thyroid gland

d. autoimmune-induced atrophy of the thyroid gland

Physical changes of hypothyroidism that must be monitored when replacement therapy is started include a. achlorhydria and constipation b. slowed mental processes and lethargy c. anemia and increased capillary fragility d. decreased cardiac contractility and coronary atherosclerosis

d. decreased cardiac contractility and coronary atherosclerosis (R- hypothyroidism affects the heart in many ways, causing cardiomyopathy, coronary atherosclerosis, bradycardia, pericardial effusions, and weakened cardiac contractility. when thyroid replacement therapy is started, myocardial oxygen consumption is increased and resultant oxygen demand may cause angina, cardiac arrhythmias, and HFs. It is important to monitor pts with compromised cardiac status when starting replacement therapy.)

when replacement therapy is started for a patient with long standing hypothyroidism, what is the most important for the nurse to monitor the patient for? a. insomnia b. weight loss c. nervousness d. dysrhythmias

d. dysrhythmias

which lab results would indicate the patient has prediabetes? a. glucose tolerance result of 132mg/dL b. glucose tolerance result of 240 mg/dL c. fasting blood glucose result of 80 mg/dL d. fasting blood glucose result of 120 mg/dL

d. fasting blood glucose result of 120 mg/dL

Which statement made by the client makes the nurse suspect the client is experiencing hyperthyroidism a. i just don't seem to have any appetite anymore b. i have a BM about every 3-4 hours c. my skin is really becoming dry and coarse d. i have noticed all my collars are getting tigher

d. i have noticed all my collars are getting tigher

The nurse determines that a patient with a 2 hour OGTT of 152 mg/dL has a. diabetes b. elevated A1C c. impaired fasting glucose d. impaired glucose tolerance

d. impaired glucose tolerance

In a patient with central diabetes insipidus, administration of aqueous vasopressin during a water deprivation test will result in a a. decrease in body weight b. increase in urinary output c. decrease in blood pressure d. increase in urine osmolality

d. increase in urine osmolality (R- pt with DI has a deficiency of ADH with excessive loss of water from the kidney, hypovolemia, hypernatreamia, and dilute urine with a low specific gravity. When vasopressin is administered, the symptoms are reversed, with water retention, decreased urinary output that increases urine osmolality, and an increase in BP.)

The client diagnosed with HHNS was admitted yesterday with a blood glucose level of 780 mg/dL. The clients blood glucose level is now 300 mg/dL. Which intervention should the nurse implement? a. increase the regular insulin IV drip b. check the patients urine for ketones c. provide the client with a therapeutic diabetic meal d. notify the HCP to obtain an order to decrease insulin

d. notify the HCP to obtain an order to decrease insulin

which arterial blood gas results should.d the nurse expect in the client diagnosed with DKA a. pH 7.34, PaO2 99, PaCO2 48, HCO3 24 b. pH 7.38, PaO2 95, PaCO2 40, HCO3 22 c. pH 7.46, PaO2 85, PaCO2 30, HCO3 18 d. pH 7.30, PaO2 90, PaCO2 30, HCO3 18

d. pH 7.30, PaO2 90, PaCO2 30, HCO3 18

the nurse is discussing the importance of exercising with a client diagnosed with type 2 diabetes who diabetes is well controlled with diet and exercise. Which information should the nurse include in the teaching about diabetes? a. eat a simple carb snack before exercising b. carry peanut butter crackers when exercising c. encourage the client to walk 20 minutes three times a week d. perform warm up and cool down exercises

d. perform warm up and cool down exercises

What is the rationale for rotation of insulin injection sites? a. prevents polyuria b. prevents rejection of insulin c. prevents an allergic reaction d. prevents lipodystrophy

d. prevents lipodystrophy

Which nursing intervention should be included in the plan of care for the client diagnosed with hyperthyrodism a. increase the amount of fiber in the diet b. encourage low cal, low protein diet c. decrease the clients fluid intake to 1000 mL/day d. provide 6 small, well balanced meals per day

d. provide 6 small, well balanced meals per day

Which statement would be correct for a patient with type 2 diabetes who was admitted to the hospital with pneumonia? a.The patient must receive insulin therapy to prevent ketoacidosis. b.The patient has islet cell antibodies that have destroyed the pancreas's ability to produce insulin. c.The patient has minimal or absent endogenous insulin secretion and requires daily insulin injections. d.The patient may have sufficient endogenous insulin to prevent ketosis but is at risk for hyperosmolar hyperglycemic syndrome.

d.The patient may have sufficient endogenous insulin to prevent ketosis but is at risk for hyperosmolar hyperglycemic syndrome.


Ensembles d'études connexes

Apple's Premium Pricing Strategy

View Set

Chap 21: cardiovascular assessment

View Set

Heimler's History AP World U2 Review

View Set

Essentials of Meteorology by Ahrens Chapter 1

View Set

Reading Plus: FROM HUMAN TO INSECT PART 2

View Set

Overview of the cytoskeleton., Actin, 8 types of actin proteins, Intermediate filaments, Microtubules, Myosin, Kinesin and Dynein, Cell Cycle 1, CDKs, Some Checkpoint info, G1/S transition and S phase, Mitosis and completion of division

View Set